TỔNG HỢP ĐỀ CHÍNH THỨC VÀ ĐỀ XUẤT KÌ THI HSG KHU VỰC DUYÊN HẢI VÀ ĐỒNG BẰNG BẮC BỘ MÔN TIẾNG ANH 10

Page 1

ĐỀ THI DUYÊN HẢI BẮC BỘ MÔN TIẾNG ANH

vectorstock.com/10212099

Ths Nguyễn Thanh Tú eBook Collection DẠY KÈM QUY NHƠN TEST PREP PHÁT TRIỂN NỘI DUNG

TỔNG HỢP ĐỀ CHÍNH THỨC VÀ ĐỀ XUẤT KÌ THI HSG KHU VỰC DUYÊN HẢI VÀ ĐỒNG BẰNG BẮC BỘ MÔN TIẾNG ANH KHỐI 10 NĂM 2019 CÓ FILE NGHE VÀ ĐẦY ĐỦ ĐÁP ÁN WORD VERSION | 2020 EDITION ORDER NOW / CHUYỂN GIAO QUA EMAIL TAILIEUCHUANTHAMKHAO@GMAIL.COM Tài liệu chuẩn tham khảo Phát triển kênh bởi Ths Nguyễn Thanh Tú Đơn vị tài trợ / phát hành / chia sẻ học thuật : Nguyen Thanh Tu Group Hỗ trợ trực tuyến Fb www.facebook.com/DayKemQuyNhon Mobi/Zalo 0905779594


ĐỀ ĐỀ XUẤT

KỲ THI HỌC SINH GIỎI CÁC TRƯỜNG THPT CHUYÊN KHU VỰC DUYÊN HẢI VÀ ĐỒNG BẰNG BẮC BỘ LẦN THỨ XII , NĂM HỌC 2018 – 2019

(Đề thi gồm 12 trang)

ĐỀ THI MÔN: TIẾNG ANH 10 Thời gian: 180 phút (Không kể thời gian giao đề) \

LISTENING Part 1. You’ll hear a recording and complete the notes below with ONE WORD AND/OR A NUMBER for each answer. HIRING A PUBLIC ROOM Example: •

the Main Hall – seats ……..200……….

Room and cost •

the (1) __________ Room – seats 100

Cost of Main Hall for Saturday evening: £115 + £250 deposit cash ( payment is required)

Cost includes use of tables and chairs and also (2) ___________

Additional charge for use of the kitchen: £25

Before the event •

Will need a music licence

Need to contact caretaker (Mr Evans) in advance to arrange (3) ________

During the event •

The building is no smoking

The band should use the stage door at the back

Don’t touch the system that controls the volume

For microphones, contact the caretaker

After the event •

Need to know the (4) ___________ for the cleaning cupboard

The floor must be washed and rubbish placed in black bags 1


All (5) _______ must be taken down

Chairs and tables must be piled up

Part 2. Listen choose the best answer to each of the following questions . (6.9 – Intensive listening) 1. The English word “biology”________ A. is actually a Greek word

B. originates from two separate words

C. means “to study words”

C. originates from three separate words

2. Biology used to be ________ A. seen as less challenging than other scientists B. an easy, soft option at school C. the study of animals and other living things D. easier than physics or chemistry 3. Nowadays, the science of biology________ A. affects our life more than physics or chemistry does B. is popular with Hollywood actors. C. is recognized by more of the general public D. is often featured on TV, radio, and in the newspaper. 4. Advances in biological science are________ A. universally welcomed B. essential to solving all global problems C. attracting young people to the science D. not a cause for worry 5. The global issue which the speaker does NOT mention is ________ A. population growth B. disease C. energy D. natural habitat destruction 2


Part 3. Decide whether the following statements are TRUE / FALSE or NOT GIVEN (6.06) 1. Isambard Kingdom Brunel was born in the east of England 2. Isambard Kingdom Brunel worked a great deal with his father. 3. Isambard Kingdom Brunel never saw the completed Clifton Suspension. 4. Isambard Kingdom Brunel designed a huge range of projects including bridges, tunnels, ships, and docks. 5. Isambard Kingdom Brunel was also interested in early photographic techniques Part 4. Listen and complete the passage – No more than THREE words in each blank. Words connected with meat are used in quite a large number of (1) __________. For example, if someone from Britain or the USA calls you (2.) __________, it means that you are a (3.) ________, you chicken out. Sometimes, if a film is a failure, some people call it a turkey. Also, if a theater shows bombs, it’s turkey. In Britain, if you are beefy, you are (4.) ___________ , strong, or a hard man. Because the English (5) _________ is roast beef, the French sometimes call an Englishman a rosbif, roast beef. If you are sheepish, you are not (6) __________, and you don’t walk around windy grass fields, but you are considered timid, or shy. If you can’t stand someone eating very loudly and untidy, you may say that he or she eats like a pig. That might be (7) ____________. Some pigs I have seen are (8) __________ when they eat. Anyway, if someone (9) __________ , you might say that they wolf it down. However, if you are very hungry, you might say: “I could (10) ____________ ,” although horse meal is not generally eaten in the English speaking world. LEXICO – GRAMMAR Part 1. Choose the best answer to complete each sentence. 1. As a full story _________, the public reacted with shock. A. Uncovered B. unfolded C. unwrapped D. undid 2. The song is not _________ as original as their previous ones. A. almost B. virtually C. nearly D. practically 3. I left the company by _________, not because I was forced to. A. choice B. option C. selection D. preference. 4. I can’t understand why you have to make such a _________ about something so unimportant. A. mess B. stir C. fuss D. bother 5. If you don’t want that wardrobe, I am sure I could put it to good _________ A. value B. use C. benefit D. worth 6. _________ it or not, I’ve just been given a totally unexpected pay rise! A. Believe B. Accept C. Presume D. Allow 7. I was pretty _________ then; I had a lot to learn. A. white B. green C. blue D. black 8. _________ appears considerably larger at the horizon than it does overhead is merely an optical. A. The moon B. That the moon C. When the moon D. The moon which 3


9. The leaves of the white mulberry provide food for silkworms, _________ silk fabrics are woven. A. whose cocoons B. from cocoons C. whose cocoons

D. from whose cocoons

10. _________ business managers plan the tasks that their employees are to carry out. A. It is the organizing process

B. They process the organizing

C. While the organizing process

D. Through the organizing process

11. Their home is _________ designed to suit their needs. A. prestigiously

B. respectfully

C. ideally

D. valuably

12. It has been established _________ dispute that this disease can be cured. A. past

B. over

C. beyond

D. outside

13. He was _________ with emotion at seeing his team win the championship. A. carried over

B. carried forward

C. carried out

D. carried away

14. Mr. Lightfinger was charged with _________ the funds of several charities. A. misappropriating

B. mislaying

C. mistaking

D. misplacing

C. assent

D. consent

15. All visitors are requested to _________ with the regulation. A. agree

B. comply

16. The postal services were _________ for several weeks by the strike. A. disrupted

B. perturbed

C. disarrayed

D. deranged

17. He believed that promotion should be awarded on _________, not on length of service. A. equality

B. merit

C. characteristics

D. purpose

18. He _________ the cart before the horse by buying the ring before he had proposed to her. A. fastened

B. tied

C. put

D. coupled

19. After congratulating his team, the coach left, allowing the players to let their _________ down for a while. A. hair

B. heads

C. hearts

D. souls

20. Grandma says there wasn’t a _________ in that story Granddad told last night about being a war hero. A. speck

B. crumb

C. dot

D. grain

Part 2. There are TEN mistakes in the following passage, find them then correct them. 4


Edwin Hubble was an American astronomer who research led to discoveries about galaxies and the nature of the universe. He settled a long debate by demonstrating that the Andromeda nebula located outside our galaxy, established the islands universe theory, which states that galaxies exit outside of our own. His study of the distribution of galaxies resulted from Hubble’s Constant, a standard relationship between a galaxy distance from the earth and its speed of recession. In 1925, Hubble had devised a classification system for the structure of galaxies and provided conclusively observational evidence for the expansion of the universe. His work pushed the one-hundred- inch Mount Wilson telescope beyond its capability and provided strong impetus for the construction of an instrument twice its size at Mount Palomar, where Hubble used during his last years of research. The telescope that bears his name was launching on a space shuttle in 1990 and orbits the earth, collecting datum about the size of the universe. Part 3. Complete the following sentences with a suitable preposition 1. When the offer of the free trip to Holland was made, John was very quick _________ the mark, and managed to get the first ticket. 2. If trains always leave_________ schedule, why do so many of them arrive late at the other end? 3. My mother never gives anyone a tip _________ principle. 4. I met him at the party and he asked _________ you. 5. “I am Polish _________ birth, but I have French nationality”. 6. He went _________ his own accord: nobody forced him to go. 7. She was annoyed with Angela_________ ignoring their agreement. 8. Children should not talk _________ to their parents. 9. A shortage of money has forced them to scale _________ the project. 10. He accepted my advice _________ reservation. Part 4.Use the word given in capitals to form a word that fits in the space Lichens are a unique group of complex, (1. FLOWER) _________ plant growing on rock and trees. There are thousands of kinds of lichen, which come in a wide variety of colours. They are composed of algae and fungi which (2. UNITY)_________ to satisfy the needs of the lichens. The autotrophic green algae (3. PRODUCTION) _________ all their food through a process called photosynthesis and provide the lichen with (4. NUTRITION) _________ element. On the other hand, the heterotrophic fungus which (5. DEPENDENCE) _________ on other elements to provide its food, not only (6. ABSORPTION) _________ and stores water for the plant but also helps protect it. This (7. UNITE) _________ by which two (8. SIMILARITY) _________ organisms live together is called “Symbiosis”. This (9. SHARE) _________ enables lichens to resist the most advert environmental conditions found on earth. They can be in some very (10. LIKE) _________ places such as the polar ice caps as well as in tropical zones, in dry areas as much as wet ones, on mountain peaks and along coastal areas. READING Part 1. Read the text then decide which word best fits each space. 5


KIWI SURPRISE When a work project offered me the opportunity to return to New Zealand, I spent several weeks (1) ___________ a country I had left in my early twenties. I’d forgotten about the petrol stations where men in smart uniforms attend to you. They fill your tank, (2) ______________ your oil and still (3) __________ you less than one third of the British price for fuel. And the people rush to your assistance if they see you (4) _____________ over a map. Or the blissful absence of tips. Locals simply cannot understand why anybody should (5) __________ to pay extra for friendly efficient service. Given that New Zealand has about 3,000 kilometres of coastline, it should come as no surprise that social life (6) ___________ around the sea. When Auckland office workers leave their desks at the end of the working day, they don’t (7) ___________ home. Instead they (8) __________ a beeline for marina and spend the evening under sail on the Hauraki Gulf. There are more yachts in Auckland than in any other city in the world – no wonder it’s called the City of Sails. Even those who can’t afford a vessel of their own will always know someone who has one, or at the (9) _________ least, will windsurf the offshore breezes at speeds that make the commuter ferries appear to stand (10) ___________ 1. A. regaining

B. recapturing

C. refamiliarising

D. rediscovering

2. A. control

B. measure

C. check

D. calculate

3. A. charge

B. ask

C. require

D. demand

4. A. pointing

B. doubting

C. clamoring

D. puzzling

5. A. accept

B. insist

C. expect

D. respond

6. A. centres

B. revolves

C. turns

D. gathers

7. A. move

B. aim

C. head

D. divert

8. A. have

B. do

C. get

D. make

9. A. simple

B. single

C. hardly

D. very

10. A. still

B. dead

C. afloat

D. upright

Part 2. Fill each of the numbered blanks in the following passage with ONE suitable word. We do not know how art began any more than we know how language started. If we take art to mean (1) __________ activities as building temples and houses, making pictures and sculptures, or weaving patterns, there are (2) ___________people in all the world without art. If, on the other hand, we mean by art some kind of beautiful luxury, we must realize that (3) __________ use of the word is a very recent development and that many of the outstanding artists of the past never thought of it. We can best understand this difference if we think of architecture. We all (4) __________ that there are beautiful buildings and that some of them are true (5) ___________ of art. But there is (6) ____________ any building in the world which was not erected for a particular purpose. Those who use these buildings as places of worship or entertainment, or as dwellings, judge them first and foremost by the standards of utility. But apart from this, they may like or (7) ______________ 6


the deign or the proportion of the structure. In the (8) __________ the attitude to paintings and statues was often similar. We are not (9) ___________ to understand the art of the past if we are quite ignorant of the (10) ________ it had to serve.

Part 3. Read the passage and answer the question below In the late eighteenth century, battle raged in almost every corner of Europe, as well as in the Middle East, South Africa, the West Indies, and Latin American. In reality, however, there was one major war during this time, the war between Britain and France. All other battles were ancillary to this large conflict, and were often at least partially related to its antagonist’s goals and strategies. France sought total domination of Europe. This goal was constructed by British independence and Britain’s efforts throughout the continent to thwart Napoleon; through treaties, Britain built coalitions (not dissimilar in concept to today’s NATO) guaranteeing British participation in all major European conflicts. These two antagonists were poorly matched, insofar as they had very unequal strengths: France was predominant on land, Britain at sea. The French knew that, short of defeating the British navy, their only hope of victory was to close all the ports of Europe to British ships. Accordingly, France set out to overcome Britain by extending its military domination from Moscow to Lisbon, from Jutland to Calabria. All of this entailed tremendous risk, because France did not have the military resources to control this much territory and still protect itself and maintain order at home. French strategies calculated that a navy of 150 ships would provide the force necessary to defeat the British navy. Such a force would give France a three – to- two advantage over Britain. This advantage was deemed necessary because of Britain’s superior sea skills and technology, and also because Britain would be fighting a defensive war, allowing it to win with fewer forces. Napoleon never lost sight of his goal, because Britain represented the last substantial impediment to his control of Europe. As his force neared that goal, Napoleon grew increasingly impatient and began planning an immediate attack. However, Britain anticipated Napoleon’s imminent invasion and decided that its only hope was to take the offensive immediately. “Napoleon” the British general Wellington once observed, “never in his life had the patience for a defensive war”. Indeed, in all ventures Napoleon tried to take the offensive on all fronts, and at all costs. Had he earlier postponed the Russian expedition, for example, he might not only have conquered Russian, but might also have conserved enough power to command the continent. In the end, the British gambit succeeded, and British’s sea power overwhelmed Napoleon’s forces. 1. The main purpose of this passage is to ________________ A. summarize the prelude to the naval conflict between France and England. B. prove that Britain’s superior naval power was effective in conquering Europe. 7


C. demonstrate the futility of a land power trying to block ports effectively D. trace the history of social relations between England and France 2. The word “ancillary” in line 3 is closest in meaning ________________ A. secondary

B. central

C. subsequent

D. harmful

3. The word “antagonists” in line 4 is closest meaning to ________________ A. people

B. enemies

C. navies

D. kings

4. According to the passage, France’s ultimate goal during the wars was to ________________ A. secure its borders, which were contested by neighbors. B. protect Europe from British domination C. develop a stronger navy D. take control of the entire continent 5. According to the passage, the British army fought France ________________ A. only at sea B. in almost all of France’s European offensives C. only when Britain’s borders were attacked D. in the West Indies and South African, but nowhere else 6. It can be inferred that France’s efforts to close the continent’s ports ________________ A. guaranteed its later victory over Britain B. was entirely unattainable C. increased the chances of a successful rebellion at home D. required it to conquer South Africa as well 7. The word “impediment” in line 20 is closest in meaning to ________________ A. comment

B. residue

C. dismemberment

D. obstacle

8. According to the passage, France believed a large advantage was necessary to defeat the British navy for all of the following reasons EXCEPTS: ________________ A. The British navy had superior sailors B. The British were engaged in several other wars at the time 8


C. The British would be fighting defensively D. The British possessed better ships and weaponry than did the French 9. It can be inferred from the passage the British general Wellington ________________ A. had no respect for Napoleon’s skill as a general B. enjoyed gambling C. expected to gain an advantage by forcing Napoleon to fight defensively D. opposed Britain’s participation in defense treaties with other European countries 10. The word “gambit” in line 25 is closest in meaning to ________________ A. calculated risk

B. virtually reality

C. refreshing change D. dubious victory

Question 4: Read the following passage and do the tasks below Weakness of the school system A. By attempting to fit in as much as possible, the school day is continually being added to. In many ways, this would appear to be a good idea, as our knowledge and understanding of the world is always growing and it would seem logical to incorporate this into schools. The reality, however, has some decided drawbacks. There is a growing feeling amongst many that the modern school curriculum, in an effort to teach as many varied subjects as possible, is actually teaching students less. It seems that by constantly adding to what should be taught in the classroom, the classes are less focused, not offering the deeper learning that institutions perhaps should. B. With classes sometimes only 30 minutes long, the overwhelming amount of information teachers are required to present often only gives students time to learn facts, not to think in any great detail about what they are being presented with. The problem is that students are not getting the opportunity to absorb what they are being taught as the curriculum expands in order to keep what has already been taught and supplement it with everything new that comes along. The weaknesses of such a system are clear – well informed though such students may be, there is the risk of an increasing number of graduates who have no real creative or intellectual ability. By denying students the opportunity to sit and think their way through problems, or even consider their own opinion, some schools are not always providing a truly educational atmosphere. There are, of course, certain aspects of education which need to be taught by simply inputting the information, basic mathematics, for example. But there are many other subjects which could be best learned by having an opportunity to think and discuss what is being taught. Literature, writing and the social sciences are good examples of subjects which cannot be considered as ‘covered’ by a mass of information without the opportunity to discuss, debate or consider meaning or implications. There are also important social skills to be learned during such periods of open discussion, skills which are not addressed by an endless flow of teacher-centred information. C. Teachers themselves have also voiced concerns about the amount of information they are required to impress upon their students. There is a feeling in many educational establishments that students are no longer being educated, but taught how to pass tests. In a world where academic success is too often measured by examination results, this is a serious concern. If there is too much information to simply be memorised and not enough time to truly assimilate it, what happens to students who fail to meet the grade? By current standards, they are failures, yet they may have great potential in areas not covered by the test and there are many students 9


who, despite clear intellectual ability, simply do not perform well in tests. Again, the problem is one of focus, as education authorities are looking at the outcome of schooling rather than the content presented in the class. D. It is here that many teachers feel the situation could be addressed at a local level. By giving more discretion to teachers, school courses could be tailored to suit the students rather than tailoring students to meet ever-expanding course requirements. In addition, by running a curriculum that gives options rather than defines an entire course, considerably more freedom would be possible. As it is, progression through most primary and secondary schools is regimented, and there is little room for students to identify and develop their own skills and strengths. If material could be chosen on the basis of its merits rather than simply because it has been put in the curriculum, then what is selected may be taught to a depth that would serve some purpose. There is, of course, a counter-argument, which claims that such open guidelines could lead to vast differences in standards between schools. What one teacher may see as essential for a student’s education, another may see as irrelevant, and this will result in students with widely different educational strengths. E. With such a high-pressure learning environment, there are also a number of social aspects to schooling which need to be considered. The increased student workload cannot be covered in the classroom alone for the simple reason that there is not enough time in the average school week, and much of this extra workload has been pushed into the realm of homework. At its best, homework should be the opportunity to look in greater detail at what has been studied. In other words, to actually think about it and its relevance. The reality, however, is often very different. Concerned parents and overextended students are finding that homework is taking an increasingly large part of a student’s evening, cutting into time many feel should be spent as part of a child’s social education. Other social pressures have compounded the situation, as many of the areas of educating a young child which should be the responsibility of the parents have ill-advisedly become the school’s responsibility. Drug awareness and health issues, for example, are occupying an increasingly large part of the school day. F. Many people believe that we should be teaching less, but teaching it better, and it is here that they think a solution can be found. Yet the process of rewriting a curriculum to incorporate only that which is essential but can be well learned would take far longer than most educational authorities have, and would be considered by many to be a ‘regressive’ step. Changes in the curriculum have largely been motivated by changes in the nature of employment, as job mobility demands that people know something about considerably more areas than were traditionally necessary. A little about a lot allows for the job mobility which has become so common. No matter what the final verdict may be, one thing is for sure – change will be slow, and not always for the best. Choose the most suitable headings for sections A–F from the list below. i. A question of time ii. Lack of teacher training iii. Student success iv. The argument for flexibility v. Importance of teaching experience vi. Extra-curricular pressures vii. The benefits of a varied curriculum viii. Imbalanced focus ix. Over-reliance on examinations x. Quality of quantity?

1. Paragraph A ________ 2. Paragraph B ________ 3. Paragraph C ________ 4. Paragraph D ________ 5. Paragraph E ________ 6. Paragraph F ________

Do the following statements agree with the views of the writer? Write 10


YES if the statement agrees with the writer NO if the statement does not agree with the writer NOT GIVEN if there is no information about this in the passage 7. No subjects can be comprehensively learned without time to discuss and debate the facts. 8. Tests are a fair measure of ability. 9. Schools are trying to be responsible for too many aspects of a child’s education. 10. Future changes in the curriculum will improve the situation. WRITING Part 1. Finish each of the following sentences in such a way that it means exactly the same as the sentence before it. 1. Skyscrapers in the USA are on average taller than anywhere else in the world. The average……………………………………………………………………............ 2. Even though I admire his courage, I think he is foolish. Much…………………………………………………………………………………… 3. They had to wait for twelve hours before their flight left. Only after a ……………………………………………………………………………. 4. I write to him almost everyday. Hardly…………………………………………………………………………………. 5. The house collapsed because of faulty building work. It was…………………………………………………………………………………… Part 2. Rewrite the following sentences using the words in brackets. Do not alter the words given. 1. Don’t tell the boss anything about this. (BREATHE) …………………………………………………………………………………………. 2. She wasn’t speaking seriously. (TONGUE) …………………………………………………………………………………………. 3. She does not want to be involved in the scandal caused by her husband’s remarks. (DISTANCE). …………………………………………………………………………………………….. 4. It is my opinion that there is no advantage in further discussion. (SEE) 11


…………………………………………………………………………………………….. 5. Please excuse Jane’s poor typing as she’s only been learning for a month. (ALLOWANCE) ……………………………………………………………………………………………….. Part 3. Essay writing Nowadays the crime rate is increasing, especially among teenagers. What are the reasons behind it? How can we reverse this trend? Use specific reasons and examples to support your answer, your writing should be of about 250 words in length. ……………………………………………………………………………………………………………………… ……………………………………………………………………………………………………………………… ……………………………………………………………………………………………………………………… ……………………………………………………………………………………………………………………… ……………………………………………………………………………………………………………………… ……………………………………………………………………………………………………………………… ……………………………………………………………………………………………………………………… ……………………………………………………………………………………………………………………… ……………………………………………………………………………………………………………………… ……………………………………………………………………………………………………………………… ……………………………………………………………………………………………………………………… ……………………………………………………………………………………………………………………… ……………………………………………………………………………………………………………………… ……………………………………………………………………………………………………………………… ……………………………………………………………………………………………………………………… ……………………………………………………………………………………………………………………… ……………………………………………………………………………………………………………………… ……………………………………………………………………………………………………………………… ……………………………………………………………………………………………………………………… ……………………………………………………………………………………………………………………… ……………………………………………………………………………………………………………………… ……………………………………………………………………………………………………………………… ……………………………………………………………………………………………………………………… ………………………………………………………………………………………………………………………

12


HƯỚNG DẪN CHẤM ĐỀ THI DUYÊN HẢI LỚP 10 – TRƯỜNG THPT CHUYÊN NGUYỄN TẤT THÀNH YÊN BÁI LISTENING Part 1. 1. Charlton

2. parking

3. entry

4. code

5. decoration/ decorations

Part 2 1B

2A

3D

4C

2T

3T 4T

5D

Part 3 1T

5NG

Part 4 1. colloquial English phrases 4. physically well built 7. unfair to pigs

2. A chicken

3. Coward

5. national dish

6. covered in wool

8. quite delicate

9. eats especially quickly

10. eat a horse LEXICO – GRAMMAR Part 1. 1.B

2.C

13. D 14.A

3.A

4.C

5.B

6.A

7.B

8.B

15.B

16.A

17.B

18.C

19.A

20.D

9.D

10.D

11.C

12.C

Part 2. There are TEN mistakes in the following passage, find them then correct them. Edwin Hubble was an American astronomer who research led to discoveries about galaxies and the nature of the universe. He settled a long debate by demonstrating that the Andromeda nebula located outside our galaxy, established the islands universe theory, which states that galaxies exit outside of our own. His study of the distribution of galaxies resulted from Hubble’s Constant, a standard relationship between a galaxy distance from the earth and its speed of recession. In 1925, Hubble had devised a classification system for the structure of galaxies and provided conclusively observational evidence for the expansion of the universe. His work pushed the one-hundred- inch Mount Wilson telescope beyond its capability and provided strong impetus for the construction of an instrument twice its size at Mount Palomar, where Hubble


used during his last years of research. The telescope that bears his name was launching on a space shuttle in 1990 and orbits the earth, collecting datum about the size of the universe. Mistakes 1. Who 2. Located 3. Established 4. From 5. Galaxy 6. In 7. Conclusively 8. Where 9. Launching 10. datum

Correction 1. Whose 2. Was located 3. Establishing 4. In 5. Galaxy’s 6. By 7. Conclusive 8. Which 9. Launched 10. data

Part 3. Complete the following sentences with a suitable preposition 1. off

2. On 7. For

3. On 8. Back

4. After 9. Down

5. By

6. Of 10. Without

Part 4. 1. Flowerless 5. Depends

2. unite3. Produce 6. Absorbs

7. Union

4. Nutritional / nutrient 8. Dissimilar 9. sharing

10. Unlike

READING Part 1. Read the text then decide which word best fits each space. 1D

2C

3A

4D

5C

6B

7C

8D

9D

10A

Part 2. Fill each of the numbered blanks in the following passage with ONE suitable word. 1. such; 2. no / few; 3. this ; 4. know/ realize/agree ; 5. works/examples; 6. scarcely / hardly; 7. dislike ; 8. past; 9. likely / able / ready; 10. aim / purpose Part 3. 1.A

2.A

3.

4.D

5.B

6.A

7.D

8.B

9.C

10.A

Part 4 : Read the following passage and do the tasks below 1.x

2. viii

3. ix

4. iv

5. vi

6. i

7. NO

8. NO

9. YES

10. NO


WRITING Part 1. Finish each of the following sentences in such a way that it means exactly the same as the sentence before it. 1. The average skyscraper in the USA is taller/ higher / bigger than anywhere else in the world / The average height / size of skyscrapers in the USA is greater than anywhere else in the world. 2. Much as I admire his courage, I think he is foolish. 3. Only after a twelve hour wait / a delay of twelve hours did their flight leave. 4. Hardly a day goes by / passes without me/ my writing to him. 5. It was faulty building work that caused the building to collapse / the collapse of the building. Part 2. Rewrite the following sentences using the words in brackets. Do not alter the words given. 1. 2. 3. 4.

Don’t breathe a word of this to the boss. She was speaking with her tongue in her cheek. She wants to distance herself from the scandal caused by her husband’s remark. I do not / cannot see any advantage / point / sense in further discussion. As far as I can see, there is no advantage / point / sense in further discussion. As I see it, there is no advantage /point / sense in further discussion. 5. Please make allowances for Jane’s poor typing as she’s only been learning for a month. TAPESCRIPTS Part 1 Official: Hello? Woman: Oh, hello. I wanted to enquire about hiring a room in the Village Hall, for the evening of September the first. Official: Let me just see … Yes, we have both rooms available that evening. There’s our Main Hall – That’s got seating for 200 people. Or there’s the Charlton Room … Woman: Sorry? Official: The Charlton Room – C-H-A-R-L-T-O-N. That’s got up to one hundred. Woman: Well, we’re organising a dinner to raise money for a charity, and we’re hoping for at least 150 people, so I think we’ll go for the Main Hall. How much would that cost? Official: Let’s see. You wanted it for the evening of September 1st? Woman: Yes, that’s a Saturday.


Official: So from six p.m to midnight that’d be £115 – That’s the weekend price, it’s £75 on weekdays. Woman: That’s all right. Official: And I have to tell you there’s also a deposit of £250, which is refundable of course as long as there’s no damage. But we do insist that this is paid in cash, we don’t take cards for that. You can pay the actual rent of the room however you like though – cash, credit card, cheque. Woman: Oh, well I suppose that’s Ok. So does the charge include use of tables and chairs and so on? Official: Oh, yes. Woman: And what about parking? Official: Yeah, that’s all included. The only thing that isn’t included is … you said you were organising a dinner? Woman: Yeah Official: Well, you’ll have to pay extra for the kitchen if you want to use that. It’s £25. It’s got very good facilities – good quality cookers and fridges and so on. Woman: Ok, well I suppose that’s all right. We can cover the cost in our entry charges. Official: Right. So I’ll make a note of that. Now there are just one or two things you need to think about before the event. For example, you’ll have to see about getting a licence if you’re planning to have any music during the meal. Woman: Oh, really? Official: It’s quite straightforward. I’ll give you the details later on. And about a week or ten days before your event you’ll need to contact the caretaker, that’s Mr Evans, to make the arrangements for entry – he’ll sort that out with you. Woman: And do I give him the payment as well? Official: No, you do that directly with me. Woman: Right. Now is there anything I need to know about what happens during the event? Official: Well, as you’ll be aware, of course the building is no smoking throughout. Woman: Of course. Official: Now, are you having a band? Woman: Yes,


Official: Well, they’ll have a lot of equipment, so rather than using the front door they should park their van round the back and use the stage door there. You can open that from inside but don’t forget to lock it at the end. Woman: Ok Official: And talking of bands. I’m sure I don’t need to tell you this, but you must make sure that no one fiddles about with the black box by the fire door – that’s a system that cuts in when the volume reaches a certain level. It’s a legal requirement. Woman: Sure. Anyway, we want people to be able to talk to one another so we don’t want anything too loud. Oh, that reminds me, we’ll be having speeches – are there any microphones available? Official: Yeah. Just let the caretaker know, he’ll get those for you. Right, now when the event is over we do ask that the premises are left in good condition. So there’s a locked cupboard and you’ll be informed of the code you need to open that. It’s got all the cleaning equipment, brushed and detergent and so on. Woman: Right. So what do we need to do after everyone’s gone? Sweep the floors I suppose? Official: Well, actually they have to be washed, not just swept. Then you’ll be provided with black plastic bags, so all the rubbish must be collected up and left outside the door. Woman: Of course. We’ll make sure everything’s left tidy. Oh, and I forgot to ask, I presume we can have decorations in the room? Official: Yes, but you must take them down afterwards. Woman: Sure. Official: And the chairs and tables should be stacked up neatly at the back of the room. Woman: I’ll make sure I’ve got a few people to help me.

Part 4 Words connected with meat are used in quite a large number of colloquial English phrases. For example, if someone from Britain or the USA calls you a chicken, it means that you are a coward, you chicken out. Sometimes, if a film is a failure, some people call it a turkey. Also, if a theater shows bombs, it’s turkey. In Britain, if you are beefy, you are physically well built, strong, or a hard man. Because the English national dish is roast beef, the French sometimes call an Englishman a rosbif, roast beef. If you are sheepish, you are not covered in wool, and you don’t walk around windy grass fields,


but you are considered timid, or shy. If you can’t stand someone eating very loudly and untidy, you may say that he or she eats like a pig. That might be unfair to pigs. Some pigs I have seen are quite delicate when they eat. Anyway, if someone eats especially quickly, you might say that they wolf it down. However, if you are very hungry, you might say: “I could eat a horse,” although horse meal is not generally eaten in the English speaking world.


HỘI CÁC TRƯỜNG CHUYÊN VÙNG

ĐỀ THI CHỌN HỌC SINH GIỎI LẦN THỨ XII MÔN: TIẾNG ANH - KHỐI 10

DUYÊN HẢI VÀ ĐỒNG BẰNG BẮC BỘ TRƯỜNG THPT CHUYÊN NGUYỄN TRÃI

Thời gian: 180 phút

TỈNH HẢI DƯƠNG

Đề thi gồm: 15 trang

ĐỀ ĐỀ XUẤT

SECTION 1: LISTENING (50 points) I. Complete the following information about the cars available for rental, using NO MORE THAN ONE WORD OR NUMBER for each gap. (10 points) Name

Size

Miles per gallon

Price*

IOTA

Small

20

£23

COMBI

Medium

18

(3). __________

ROADSTER

(1). __________

2. _____________

(4). __________

*Prices include tax and (5). ________________ II. Listen to a conversation on a university campus and choose the correct option. (10 points) 1. What is the woman’s status at the university? A. She is a senior.

B. She is a junior.

C. She’s a transfer student.

D. She’s a graduate student.

2. What is the man’s status at the university? A. He is a senior.

B. He is a tutor.

C. He’s a transfer student.

D. He’s a graduate student.

3. What does the man want to learn from the woman? A. How to transfer to a junior college. B. How to find his way around campus. C. The course requirements for a literature major. D. Who won the campus election. 4. How many total courses must a student take for a literature major? 1


A. Three

B. Five

C. Eight

D. Ten

5. The man will probably take his elective courses in which area? A. American literature

B. World literature

C. Literary analysis

D. Surveying

III. You will hear an explorer called Richard Livingstone talking about a trip he made in the rainforest of South America. Listen and indicate true (T) or false (F) statements. (10 points) Statements

T

F

1. They went all the way by boat. 2. Richard say that during the walk, they were always both cold and wet. 3. In a deserted camp, they found some soup made from unusual meat and vegetables. 4. After the meal, they began to feel worried about what they have done. 5. Before leaving the camp, they left the sum of 50 dollars to thank the host. IV. You will hear a man called Alec Gardiner talking about the things he collects. Listen and complete the sentences. Write NO MORE THAN THREE WORDS taken from the recording for each answer in the spaces provided. (20 points) Alec isn’t exactly (1) ______________ why he collects things. He started collecting when he was (2) _______________. As a child Alec kept the things he collected in (3) ______________. Alec says it’s a wonderful feeling when he completes a (4) __________ of something. Alec likes collecting cartoon figures because they look (5) ____________ and they amuse him. It’s possible to collect so many Mickey Mouse figures because Mickey is the (6) ___________ cartoon character. He keeps most of his cartoon figures in the living room on small (7) ___________. Alec used to buy things at collectors’ (8) ______________ and antique shops.

2


Alec doesn’t like to leave the house for too long because he’s afraid of (9) _______________. In order to complete a set of something, Alec sometimes has to pay out (10) _______________ hundred pounds. SECTION 2: LEXICO - GRAMMAR (50 points) I. Choose the word or phrase that best fits the gap in each sentence (20 points) 1. Nathalie seems very tough at work. She’s a different person at home, _________. A. though

B. although

C. as though

D. even though

2. I kept out of the conversation because it _________ me. A. wasn’t concerned B. wasn’t concerning

C. didn’t concern D. didn’t concern to

3. The entire city was _________ electricity last night- it was chaotic. A. no

B. almost no

C. hardly any

D. without

4. Henry was overweight, so he went on a strict diet and _________ twenty kilos. A. missed

B. lost

C. failed

D. fell

5. He was arrested because he answered to the description of the _________ man. A. searched

B. pursued

C. wanted

D. hunted

6. Humanity has done great damage to the environment in its search for _________ materials. A. live

B. raw

C. crude

D. rude

7. _________, the balcony chairs will be ruined in this weather. A. Leaving uncovered C. Left uncovered

B. Having left uncovered D. Been left uncovered

8. One way to let off _________ after a stressful day is to take some vigorous exercise. A. cloud

B. tension

C. steam

D. sweat

9. Their research into the causes of cancer promises to break the new _________ in the field and possibly lead to a cure. A. earth

B. ground

C. soil

D. land

10.After three days in the desert, his mind began to play _________ on him. A. games

B. jokes

C. tricks

D. fun

11.The match will be screened on ITV with _________ commentary by Any Gray. 3


A. lively

B. live

C. alive

D. living

12.I know you didn’t want to upset me but I’ sooner you _________ me the whole truth yesterday. A. could have told B. told

C. have told

D. had told

13.As the drug took _________, the patient became quieter. A. effect

B. force

C. influence

D. action

14.The dawn redwood appears ____ some 100 million years ago in northern forests around the world. A. was flourished

B. having to flourish

C. to have flourished

D.

have flourished 15.His comments _________ little or no relation to the facts and the figures of the case. A. reflect

B. bear

C. give

D. possess

16.All _____ is a continuous supply of the basic necessities of life. A. what is needed

B. for our needs

C. the thing needed

D. that is needed

17.It is urgent that this letter _____ immediately. A. was posted

B. posted

C. be posted

D. be post

18.John: This grammar test is the hardest one we’ve ever had this semester! Mary: _____ but I think it’s quite easy. A. I couldn’t agree more

B. I understand what you’re saying

C. You’re wrong

D. I don’t see in that way

19.It is only recently that ballets have been based on the themes _____ American life. A. reflecting

B. reflects

C. is reflecting

D. reflected

20.I wish you’d do the accounts. I don’t have ________ for numbers. A. a head

B. a mind

C. the heart

D. the nerve

II. The passage below contains 10 errors. IDENTIFY and CORRECT them. Write your answers in the space provided in the column on the right. (10 points) Britain has a general mild temperate climate. The weather, however, tends to be changed (though not necessarily unpredictable) as a result of the constant influence of different air mass. The prevailing winds are south-westerly, which bring warm air in from across the 4


Atlantic. There are a few extremes in temperature, which rarely goes above 320C or below -100C. In summer, southern Britain is warmer than northern Britain because of its latitude, but in winter the North Atlantic Drift – a warm sea current - keeps the west mild than the east. Consequently, Wales and the south-west Peninsula has the most moderate climate and eastern England the most extremely. These differences, are not great however, and local variations arise from factors such as altitude and pollution are often greater. Annual rainfall is fairly evenly distribute, but ranges from more than 1,600 mm in the mountainous areas of the west and north far less than 800 mm over central and eastern parts. This is because depression from the Atlantic bring frontal rainfall first to the west and because western Britain is higher and so gets more relief rain. Your answer: Mistakes

Corrections

Mistakes

1.

6.

2.

7.

3.

8.

4.

9.

5.

10.

Corrections

III. Complete the following sentences with a suitable particle/preposition. (10 points) 1. My wife backed me ______ over my decision to quit my job. 2. My mum and dad always find a way to work well together and are not critical ________ each other. 3. At first, residents opposed plans for a new city skate park, but the skaters eventually won them _______. 4. Traditionally, Amish weddings are held on Tuesdays and Thursdays, so there is time in ________ to get ready for and clean up after each. 5. According to a government study, Korean elderly women are much more vulnerable ________ social isolation and illnesses compared to their male counterparts. 5


6. Let’s make ________ that island and wait for the storm to pass. 7. Could you please refrain ________ smoking - this is a hospital! 8. The company pulled ________ despite the economic crisis. 9. We were very sorry to hear that your grandfather passed ________ last week. 10. When Mr. Spendthrift ran out of money, he fell ________ on his mother for help. IV. Supply the correct form of the verbs in block capitals in brackets to complete the passage. (10 pts) Mount Mulanje Mount Mulanje in Malawi is the highest mountain in central Africa, part of a range which comprises no fewer than twenty peaks over 2,500 metres. The range is readily (1)________ by road and a ACCESS day’s drive allows a (2)_________ circumnavigation. More LEISURE energetic visitors, particularly walkers and climbers, are rewarded FORGET

with an experience that is (3)__________.

Mulanje is a (4)________ sight, visible for miles around. The BREATH giant slab of rock appears to protrude almost vertically from the plain. This impression is borne out by the existence of the longest sheer rockface in Africa, demanding for even the most skilled (5)___________ . The explanation for this dramatic geography lies MOUNTAIN in the rock: a hard granite, very resistant to (6)_________ , which ERODE

contrasts with the softer rocks of the plains. Most visitors remain on the lower, gentler slopes, making use of forest huts for overnight accommodation. The trek up the foothills, along clearly defined paths, is not overly (7) ________ but

may take up to a week. As the climate cools gradually, almost CHALLENGE (8)___________ , with every few metres of altitude gained, so the PERCEPTIBLE full (9)___________ of fauna and flora is revealed in all its DIVERSE 6


SPLENDID

(10)_____________. SECTION 3: READING (50 points)

I. Read the text below and decide which answer (A, B, C or D) best fits each gap. (10 points) AS OLD AS YOU FEEL It might after all be true that you are only as old as you feel. A British clinic is carrying out new high-tech tests to calculate the “real” biological age of patients (1)_____ (on the rate of physical deterioration. Information on every (2)_____ of a patient’s health, fitness, lifestyle and family medical history is (3)_____ in to a computer to work out whether they are older or younger than their calendar age suggests. The availability and increasing accuracy of the tests has (4)_____ one leading British gerontologist to call for biological age to be used to determine when workers should retire. He (5)_____ that if an employee’s biological or “real” age is shown, for example, to be 55 when he reaches his 65th birthday, he should be (6)_____ to work for another decade. Apparently most employers only take into (7)_____ a person’s calendar years, and the two may differ considerably. Some of those prepared to pay a substantial sum of money for the examinations will be able to smugly walk away with medical (8)_____ showing that they really are as young as they feel, giving them the confidence to act and dress as if they were younger. Dr Lynette Yong, resident doctor at the clinic where the tests are offered claims that the purpose of these tests will be to motivate people to (9)_____ their health. The concept of “real” age is set to become big (10)_____ in the USA with books and websites helping people work out whether their body is older or younger than their years. Others firmly believe that looks will always be the best indicator of age. 1. A. prospect

B. arranged

C. based

D. established

2. A. attitude

B. position

C. decided

D. aspect

3. A. planned

B. provided

C. supplied

D. fed

4. A. prodded

B. prompted

C. projected

D. provoked

7


5. A. argues

B. discusses

C. enquires

D. debates

6. A. incited

B. encouraged

C. supported

D. promoted

7. A. detail

B. account

C. interest

D. importance

8. A. evidence

B. grounds

C. signs

D. demonstration

9. A. increase

B. gain

C. improve

D. progress

10. A. pursuit

B. concern

C. trade

D. business

II. Fill each gap in the passage below with ONE appropriate word in the space provided. (15 points) INTRODUCTION TO A NOVEL Some years ago, I received a letter from a stranger, Joanna King. It seemed at first to be one of those pleasant fan letters that authors are occasionally cheered (1)________, but which then turn out to be something else. Joanna had an aunt, aged ninety-eight, who had kept a diary from the age of thirteen until she was ninety-four. (2)_______ Joanna nor her husband had ever been allowed to read any of these diaries, but because their relative was a woman (3)_______ strong opinions, they thought they would be interesting. The point of writing to me was to ask my advice. Joanna had read a memoir I’d written about my own mother and grandmother, two ordinary women with (4)______ claim to fame, and it had made her wonder if there was some value in the diaries (5)_______ a social document. Could I suggest (6)________ might be done with them? I suggested that a university might be interested and enclosed various names and addresses. I said the thought of someone keeping a diary over such (7)______ length of time, so neatly covering most of a century, was (8)________ itself extraordinary, and I would love to read them myself. Joanna replied saying that this was what she had hoped. (9)________ is, that I myself might be intrigued enough to want to (10)_______ something of them. I hadn’t, in fact, mean that, but once it had been suggested I began to toy with the prospect. III. Read the following passage and choose the best answer to each question. (10 points) Madison Square Garden, a world-famous sporting venue in New York City, has actually been a series of buildings in varied locations rather than a single building in one 8


spot. In 1873. P. T. Barnum built Barnum's Monster Classical and Geological Hippodrome at the corner of Madison Avenue and 26th Street, across from Madison Square Park. Two years later bandleader Patrick Gilmore bought the property, added statues and fountains and renamed it Gilmore's Gardens. When Cornelius Vanderbilt bought the property in 1879, it was renamed Madison Square Garden. A second very lavish Madison Square Garden was built at the same location in 1890, with a ballroom. a restaurant, a theater, a rooftop garden, and a main arena with seating for 15,000. However, this elaborate Madison Square Garden lasted only until 1924, when it was torn down to make way for a forty-story skyscraper. When the second Madison Square Garden had been replaced in its location across from Madison Square Park. boxing promoter Tex Rickard raised six million dollars to build a new Madison Square Garden. This new Madison Square Garden was constructed in a different location on 8th Avenue and 50th Street and quite some distance from Madison Square Park and Madison Avenue. Rickard's Madison Square Garden served primarily as an arena for boxing prizefights and circus events until it outgrew its usefulness by the late 1950s. A new location was found for a fourth Madison Square Garden. atop Pennsylvania Railroad Station, and plans were announced for its construction in 1960. This current edifice, which includes a huge sports arena. a bowling center. a 5,000-seat amphitheater, and a twenty-nine-story office building, does retain the traditional name Madison Square Garden. However, the name is actually quite a misnomer. The building is not located near Madison Square, nor does it have the flowery gardens that contributed to the original name

1. The main point of this passage is that Madison Square Garden (A) has had a varied history in various locations

(B) was P. T. Barnum's

major accomplishment (C) is home to many different sporting events

(D). was named after an adjacent park

2. Which paragraph discusses the third incarnation of Madison Square Garden? 9


(A) The first paragraph

(B) The second paragraph

(C) The third paragraph

(D). The last paragraph

3. What is a "venue"? (A) A place where people come together

(B) An event in a competition

(C) An exhibit of various products

(D). An invitation to a

program 4. According to the passage, Patrick Gilmore did all of the following EXCEPT that he (A) purchased the property at the corner' of Madison Avenue and 26th Street (B) made improvements to the property that he bought (C) named the property that he bought Madison Square Garden (D) sold the property to Cornelius Vanderbilt 5. The word "lavish" is closest in meaning to (A) simple

(B) modern

(C) elaborate

(D)

outlandish 6. How long did the second Madison Square Garden last? (A) 11 years

(B) 34 years

(C) 45 years

(D) 60years

7. The word “it� in the third paragraph refers to (A). circus events

(B). arena

(C). Square Garden

(D). Square Park

8. Which of the following would most likely have taken place at Rickard's Madison Square Garden? (A) A ballroom dance

(B) A theater production

(C) A basketball game

(D) A tiger show

9. The word "edifice" is most likely (A) an address

(B) an association

(C) a component

(D) a building

10. What can be inferred about the current Madison Square Garden? (A) It is on Madison Avenue.

(B) It is across from

Madison Square Park. (C) It has incredible gardens

(D) It is above a transportation center. 10


IV. Read the following passage then do the tasks that follow. (15 pts) HOW DOES THE BIOLOGICAL CLOCK TICK? A

Our life span is restricted. Everyone accepts this as 'biologically' obvious. ‘Nothing

lives for ever!’ However, in this statement we think of artificially produced, technical objects, products which are subjected to natural wear and tear during use. This leads to the result that at some time or other the object stops working and is unusable ('death' in the biological sense). But are the wear and tear and loss of function of technical objects and the death of living organisms really similar or comparable? B

Our ‘dead’ products are ‘static’, closed systems. It is always the basic material

which constitutes the object and which, in the natural course of things, is worn down and becomes 'older’. Ageing in this case must occur according to the laws of physical chemistry and of thermodynamics. Although the same law holds for a living organism, the result of this law is not inexorable in the same way. At least as long as a biological system has the ability to renew itself it could actually become older without ageing; an organism is an open, dynamic system through which new material continuously flows. Destruction of old material and formation of new material are thus in permanent dynamic equilibrium. The material of which the organism is formed changes continuously. Thus our bodies continuously exchange old substance for new, just like a spring which more or less maintains its form and movement, but in which the water molecules are always different. C

Thus ageing and death should not be seen as inevitable, particularly as the organism

possesses many mechanisms for repair. It is not, in principle, necessary for a biological system to age and die. Nevertheless, a restricted life span, ageing, and then death are basic characteristics of life. The reason for this is easy to recognise: in nature, the existent organisms either adapt or are regularly replaced by new types. Because of changes in the genetic material (mutations) these have new characteristics and in the course of their individual lives they are tested for optimal or better adaptation to the environmental conditions. Immortality would disturb this system - it needs room for new and better life. This is the basic problem of evolution 11


D

Every organism has a life span which is highly characteristic. There are striking

differences in life span between different species, but within one species the parameter is relatively constant. For example, the average duration of human life has hardly changed in thousands of years. Although more and more people attain an advanced age as a result of developments in medical care and better nutrition, the characteristic upper limit for most remains 80 years. A further argument against the simple wear and tear theory is the observation that the time within which organisms age lies between a few days (even a few hours for unicellular organisms) and several thousand years, as with mammoth trees. E

If a lifespan is a genetically determined biological characteristic, it is logically

necessary to propose the existence of an internal clock, which in some way measures and controls the aging process and which finally determines death as the last step in a fixed programme. Like the fife span, the metabolic rate has for different organisms a fixed mathematical relationship to the body mass. In comparison to the life span this relationship is ‘inverted’: the larger the organism the lower its metabolic rate. Again this relationship is valid not only for birds, but also, similarly on average within the systematic unit, for all other organisms (plants, animals, unicellular organisms). F

Animals which behave ‘frugally’ with energy become particularly old for example,

crocodiles and tortoises. Parrots and birds of prey are often held chained up. Thus they are not able to ‘experience life’ and so they attain a high life span in captivity. Animals which save energy by hibernation or lethargy (e.g. bats or hedgehogs) live much longer than those which are always active, The metabolic rate of mice can be reduced by a very low consumption of food (hunger diet) They then may live twice as long as their well fed comrades. Women become distinctly (about 10 per cent) older than men. If you examine the metabolic rates of the two sexes you establish that the higher male metabolic rate roughly accounts for the lower male life span. That means that they live life ‘energetically’ - more intensively, but not for as long. G

It follows from the above that sparing use of energy reserves should tend to extend

life. Extreme high performance sports may lead to optimal cardiovascular performance, but they quite certainly do not prolong life. Relaxation lowers metabolic rate, as does 12


adequate sleep and in general an equable and balanced personality. Each of us can develop his or her own ‘energy saving programme’ with a little self observation, critical selfcontrol and, above all, logical consistency. Experience will show that to live in this way not only increases the life span but is also very healthy. This final aspect should not be forgotten. The Reading Passage has seven paragraphs, A-G, For question 1-6, choose the correct heading for paragraphs A-G from the list of headings below. Write the correct number, i-x, in the corresponding numbered boxes. LIST OF HEADINGS

i The biological clock ii Why dying is beneficial iii The ageing process of men and women iv Prolonging your life v Limitations of life span vi Modes of development of different species vii A stable life span despite improvements viii Energy consumption ix Fundamental differences in ageing of objects and x organisms Repair of genetic material

Example answer: Paragraph A: v Your answers 1. Paragraph B

2. Paragraph C

3. Paragraph D 13


….............

….............

….............

4. Paragraph E

5. Paragraph F

6. Paragraph G

….............

….............

….............

Questions 7-10, complete the notes below Choose NO MORE THAN TWO WORDS from the passage for each answer. Write your answers in boxes 7-10 • Objects age in accordance with principles of (7) __________ and of (8) _________ • Through mutations, organisms can (9) __________ better to the environment • (10) __________ would pose a serious problem for the theory of evolution SECTION 4: WRITING (60 points) I. Write the new sentences using the words in brackets. Do not alter the given words in any way. (10 points) 1. “I think the whole idea is ridiculous.”, he said. (being) He dismissed ____________________________________________________ 2. The disagreement was a lot of a fuss about nothing. (teacup) The disagreement ________________________________________________ 3. Sorry, you can’t do whatever you want. (pick) Sorry, you are not in _______________________________________________ 4. You should not open the door for any reason. (no) Under ___________________________________________________________ 5. Her lateness made the boss angry. (bananas) It was because of _________________________________________________ II. Rewrite the following sentences, keeping their meaning unchanged, beginning with the words given. (10 points) 1. You must never mention this to him Under ……………………………………………………………………………. 2. His condition improved so rapidly that he went home four days after the operation. 14


There ……………………………………………………………………………... 3. Children learn a lot about how to behave in a situation like this. Only .................................................................................................... …… 4. It wasn’t my fault that she lost the money. Through ................................................ ……………………………………………. 5. The chances are that the whole thing will have been forgotten by next term. In all ……………………………………………………………………………... IV. Essay writing: (30 points) Write an essay of about 250 words to express your opinion on the following topic: With the development of online communication, people will never be alone and will always be able to make new friends. To what extent do you agree?

Người ra đề: Phạm Thị Diệu Thu Chữ kí: ………………………… SĐT: 0985 791 820

15


HỘI CÁC TRƯỜNG CHUYÊN VÙNG

ĐỀ THI CHỌN HỌC SINH GIỎI LẦN THỨ XII MÔN: TIẾNG ANH - KHỐI 10

DUYÊN HẢI VÀ ĐỒNG BẰNG BẮC BỘ TRƯỜNG THPT CHUYÊN NGUYỄN TRÃI

Thời gian: 180 phút

TỈNH HẢI DƯƠNG

Đề thi gồm: 15 trang

ĐÁP ÁN

SECTION 1: LISTENING (50 points) I. Complete the following information about the cars available for rental, using NO MORE THAN ONE WORD OR NUMBER for each gap. (10 points) 1. medium 2. 17 3. £28.75 4. £31.05 5. insurance II. Listen to a conversation on a university campus and choose the correct option. (10 points) 1. A

2. C

3. C

4. C

5. A

III. You will hear an explorer called Richard Livingstone talking about a trip he made in the rainforest of South America. Listen and indicate true (T) or false (F) statements. (10 points) 1. F

2. F

3. T 4. T 5. F

IV. You will hear a man called Alec Gardiner talking about the things he collects. Listen and complete the sentences. Write NO MORE THAN THREE WORDS taken from the recording for each answer in the spaces provided. (20 points) 1 sure

2 a small child

3 his bedroom

4 set

5 cute

6 oldest

7 shelves

8 markets

9 burglaries

10 a couple of 1


SECTION 2: LEXICO - GRAMMAR (50 points) I. Choose the word or phrase that best fits the gap in each sentence (20 points) 1.A

2. C

3. D

4. B

5. C

6. B

7. C

8. C

9. B

10. C

11. B

12. D

13. A

14. C

15. B

16. D

17. C

18. B

19. A

20. A

II. The passage below contains 10 errors. IDENTIFY and CORRECT them. Write your answers in the space provided in the column on the right. (10 points) Mistakes

Corrections

Mistakes

Corrections

1. changed

Changeable

6. extremely

extreme

2. mass

Masses

7. arise

arising

3. a few

Few

8.distribute

distributed

4. mild

Milder

9. far

to

5. has

Have

10. depression

depressions

III. Complete the following sentences with a suitable particle/preposition. (10 points) Answers: 1. up

2. of

3. over/round

4. between

5. to

6. for

7. from

8. through

9. away

10. back

IV. Supply the correct form of the verbs in block capitals in brackets to complete the passage. (10 pts) 1. accessible

6. erosion

2. leisurely

7. challenging

3. unforgettable

8. imperceptibly

4. breath(-)taking

9. diversity/biodiversity 2


5. mountaineer(s)

10. splendour/splendor

SECTION 3: READING (50 points) I. Read the text below and decide which answer (A, B, C or D) best fits each gap. (10 points) 1. C 2. D 3. D 4. B 5. A 6. A 7. B 8. A 9. C 10. D II. Fill each gap in the passage below with ONE appropriate word in the space provided. (15 points) 1. by

2. Neither

3. of/with

4. no/little

5. as

6. what

7. a

8. in

9. That

10. make

III. Read the following passage and choose the best answer to each question. (10 points) 1.A

2.C

3.A

4.C

5.C

6.B

7.C

8.D

9.D

10.D

IV. Read the following passage then do the tasks that follow. (15 points – 1.5 points/ correct answer) 1. ix

7. physical chemistry

2. ii

8. thermodynamics

3. vii

9. adapt

4. i

10. immortality

5. viii

(7 and 8 can be in either order)

6. iv

SECTION 4: WRITING (60 points) I. Write the new sentences using the words in brackets. Do not alter the given words in any way. (10 points) 1. the whole idea as being ridiculous. 2. made a storm in the teacup. 3


3. the position to pick and choose. 4. no circumstances should you open the door. 5. her lateness that the boss went bananas. II. Rewrite the following sentences, keeping their meaning unchanged, beginning with the words given. (10 points) 1. Under no circumstances must you mention this to him. 2. There was such a rapid improvement in his condition that he went home four days after the operation. 3. Only in a situation like this do children learn a lot about how to behave. 4. Through no fault of mine that she lost the money. 5. In all probabilities, the whole thing will have been forgotten by next term Transcript Part 1: Saleswoman: Good morning sir. How can I help you? Man: I’d like to rent a car, please. Saleswoman: Certainly. What kind of car were you thinking of? Man: Oh, a small and medium-sized one. Could you show me a brochure of leaflet with the different available types? Saleswoman: Of course. We have one type of small car and two types of medium-sized ones. As you can see, the small type is called the Iota and the medium ones are called the Combi and the Roadster. Man: thank you. I see the Iota offers 20 miles to the gallon in urban areas. Saleswoman: Yes. It’s certainly very economical. The Combi is almost as good at 18 miles to the gallon. The Roadster offer 17 – Is that right? Man: You are right. Saleswoman: Of course, those mileages go up substantially when driving on motorways. Man: I will be driving around town. The only time I will be using the motorway is from here, the airport, to the centre. 4


Saleswoman: I see. Well, I’d recommend the small Iota. Parking can’t be difficult in town and it’s a touch easier with a small vehicle. Man: Yes, of course. Let me just look at the prices before making a final decision. I’m here on business and will need the car for four days. The daily rate for the Iota is … Saleswoman: £20 plus tax. The Combi is £25 and the Roadster is £27. Again, those prices are below tax. Man: The tax is 15%, right? Saleswoman: Yes. So the total costs are £23, £28.75 and £31.05 per day, respectively. Man: Do those prices include insurance? Saleswoman: Yes, they do. However, the insurance only covers one driver – the one who signs the rental agreement. We can cover other drivers for an additional fee. Man: that’s OK. I’m here alone. OK, I’ll take the Iota. Oh, what about fuel costs? Part 2: Listen to a conversation on a university campus. Man: You seem to know your way around campus. Have you been here long? Woman: I’m a senior literature major. I’ll be graduating next June. Man: Your major is literature? Mine is, too. But I’m just beginning my work in my major. I just transferred to this university from a junior college. Perhaps you could tell me about the courses you’ve got to take for a literature major. Woman: Well, for a literature major, you need to take eight courses, and it’ll take two semesters, and it’s required for all literature majors is “Introduction to Literary Analysis.” Man: You mean, if you want to specialize in American literature, I still must take two semesters of world literature? Woman: Yes, because the two semesters are required for all literature majors. Man: But I only want to study American literature. Woman: At least you can take all of your five elective courses in the area that you want. Man: That’s what I’ll do then. Part 3: 5


You will hear an explorer called Richard Livingstone talking about a trip he made in the rainforest of South America. Listen and indicate true (T) or false (F) statements. (10 points)

6


Part 4 Hello, good evening. Welcome to Private Passions. My name is Alec Gardiner and I’m here to tell you about my passion: collecting. Collecting everything: key rings, matchboxes, cartoons characters, beer mats, cake decorations – you name it, I collect it. Now, of course, the question I get asked most often is: why? Why do I do it? What is the point? To be honest, I’m not sure I can answer that question, other than I’ve been doing it for years and it gives me a lot of pleasure. I started collecting when I was a small child and I’ve never stopped. I was never bored as a child because there was always something new to collect. Then I had to organize and arrange them in my bedroom. I enjoyed that. I started with the small toys out of cereal packets, then football cards, toy cars. And I’ve still got them. The most pleasing thing is when I find the last object to complete a set of something. That’s really satisfying. It’s so great to know that you have got absolutely all of them! Of all the things I collect, cartoon figures are my favourite because they are so cute and funny. I’ve got about 6,000 little models just of Mickey Mouse alone … from all over the world. You’d be amazed where Mickey turns up! I have thousands of other cartoon characters, too, especially Disney ones. That’s my largest collection, simply because there are so many to collect. Recently, my main problem has been where to keep everything. Most of my collections are in my living room. The walls are covered with little shelves. I have a thousand key rings, as well, hanging on the walls. In the past, I used to spend a lot of time visiting collectors’ markets and little antique shops. That was my main source of new additions for my collections. But now I mostly buy on the net – eBay for example. I’m a computer consultant, anyway, so I seem to spend most of my life online in front of the computer these days, whether it’s work or pleasure. Anyway, I don’t like to leave the house for very long. Everything in there is precious, and I’m worried about burglaries. So online shopping is perfect for me. I suppose I must have spent thousands and thousands of pounds over the years. Sometimes I have to spend a couple of hundred pounds just on one little item, if I need it to complete a set of something. It’s a lot of money for a Disney model – but it’s worth it to me. I guess that makes me an addict! 7


8


ĐỀ THI CHỌN HỌC SINH GIỎI

TRƯỜNG THPT CHU VĂN ANHÀNỘI

DUYÊN HẢI ĐỒNG BẰNG BẮC BỘ 2019 MÔN: TIẾNG ANH - KHỐI 10 Thời gian: 180 phút Đềthigồm: 18trang

ĐỀ XUẤT

A. LISTENING (50 points) Part 1. You will hear a telephone conversation between an operator and a caller. The caller is inquiring car insurance. Write NO MORE THAN THREE WORDS AND/OR A NUMBER for each answer. (10 points) CAR INSURANCE INQUIRY FORM

Name:

LIAM BYRD

Car model:

Ford Fiesta 2002

Registration:

3R1 (1) ________________________________

Current mileage:

(2) _______________________________miles

Estimated yearly mileage:

(3) ______________________________a year

Overnight parking:

(4) ____________________________________

Cover:

Third party (5) __________________________

No claims bonus:

6years

Part 2. You will hear people talking in five different situations. Choose the best answer A, B or C. (10 points). 1. You are in a shop when you overhear this man answering the telephone. What does the caller want to buy? A. a book about playing a guitar B. a book about guitar music C. a cassette of guitar music

1 / 18


2. You are listening to the radio when you hear this man speaking. What is he talking about? A. history

B. shipbuilding

C. politics

3. You are sitting in a cafĂŠ when you hear this woman speaking. She is telling her friend about ________. A. the weather

B. buying a new coat

C. new windows

4. Listen to this woman introducing a college lecture. The visiting lecturer ________. A. has recently changed career B. has made a new discovery C. was late for the lecture 5. You will hear someone talking about soap operas. What does the speaker think about them? A. They are boring and meaningless to everyone. B. The plot is very exciting and unpredictable. C. People become addicted to them without realizing it.

Your answers 1.

2.

3.

4.

5.

Part 3. You will hear a conversation which takes place in an open market between three people, Bob and Andrea Jones and Bob’s friend - Carl. Decide whether the statements are true (T) or false (F). (10 points) 1. When Bob spots his old colleague, he does not want to talk to him. 2. Bob is indifferent towards Irish folk music. 3. Acres was shut down. 4. Bob spent most of his redundancy money on a car. 5. When Andrea suggests that Bob sell his silverware he seems enthusiastic.

Your answers: 2 / 18


1.

2.

3.

4.

5.

Part 4: You will hear a psychologist speaking on British radio on the subject of astrology. Complete the sentences with NO MORE THAN THREE WORDS for each gap. (20 points) Over 60% of (1) ________ admit to being interested in astrology. Less than 3% of people would consult the stars before making (2) ________. Psychologists now believe that time of birth can affect a person’s (3) ________ development. Time of birth is not the only factor, but it is the (4) ________ for future changes. To test his idea, the speaker decided to compare people’s (5) ________ and zodiac sign. An amazing number of (6) ________ were born around mid July to mid August. Other connections found by the study were (7) ________. A fair number of serious sports players were born in the (8) ________ months. The speaker thinks that the lack of professionals dominated by mostly onestar sign is a little (9) ________. The connections may be weak because people are removed from the (10) ________.

Your answers: 1.

6.

2.

7.

3.

8.

4.

9.

5.

10.

B. LEXICO - GRAMMAR (50 points) Part 1. Choose one of the words marked A, B, C, or D which best completes each of the following sentences. (20 points) 1. I was kept awake for most of the night by the _________ of a mosquito in my ear. A. whine

B. moan

C. groan

D. screech

2. The police took him into custody _________ he disembarked from the plane. A. as long as

B. while 3 / 18


C. the moment when

D. the instant

3. I think we ought to see the rest of the exhibition as quickly as we can, _________ that it closes in half an hour. A. granted

B. assuming

C. given

D. knowing

4. After a fall in profits, the company decided _________ the hotel business. A. to pull out of

B. to back off from

C. to take out of

D. to keep away from

5. In spite of working their fingers to the _________, all the staff were made redundant. A. nail

B. edge

C. flesh

D. bone

6. Oil spills will _________ even the healthiest of marine ecosystem. A. play havoc on

B. break ground with

C. pay the consequences for

D. take their toll on

7. The doctor thought he had got over the worst, but his condition suddenly _________. A. deteriorated

B. dismantled

C. dissolved

D. disintegrated

8. _________ a small creature that defends itself with lobster-like claws and a poisonous sting. A. Scorpions are

B. Many a scorpion is

C. A scorpion, which is

D. The scorpion is

9. The library is _________ people who lose their books. A. cracking down on

B. stepping up

C. going down with

D. coming up against

10. They turned down the proposal _________ that it didn’t fulfill their requirement. A. by reason

B. on the grounds

C. as a cause

D. allowing

11. Going down white-water rapids in a canoe must be extremely _________! Does your heart start beating really fast? A. trivial

B. mundane

C. sedentary

12. If _________ by hunger, you might consider eating a frog. 4 / 18

D. exhilarating


A. they are driven to desperation

B. driving to desperate

C. you are driving to desperation

D. driven to desperation

13. In a situation like this, there are no _________ rules. You just have to use your own discretion. A. clear and cut

B. hard and fast

C. up-and-coming

D. out-and-out

14. She demanded to be promoted to a post of responsibility, otherwise she would _________ about her affair with the Prime Minister. A. reveal the cake

B. spill the beans

C. let loose the pussycat

D. spit out the bit

15. When reading the top of the hill, _________. A. we suddenly caught sight of the sea B. it was the sea that extended below us C. we extended the sea below us D. the sea came into view 16. His English was roughly _________ with my Greek, so communication was rather difficult. A. level

B. on a par

C. equal

D. in tune

17. I didn’t want to make a decision _________ so I said I’d like to think about it. A. on and off

B. in one go

C. there and then D. at a stroke

18. If only the neighbor _________ a pet fish, not a dog which always barks. A. is having

B. were having

C. had had

D. would have

19. _________, we couldn’t make out the gist of the passage. A. Carefully as Jack explained

B. As Jack carefully explained

C. Due to Jack’s carefully explanations D. Though carefully explained by Jack 20. Surgeons now can do miracles to their physical _________ which used to be unbeatable. A. conditions

B. wrongdoings

C. malfunctions

5 / 18

D. abnormalities


Your answers: 1.

2.

3.

4.

5.

6.

7.

8.

9.

10.

11.

12.

13.

14.

15.

16.

17.

18.

19.

20.

Part 2. Read the following text which contains 10 mistakes. Identify the mistakes and write the corrections in the corresponding numbered boxes.(10 points) MOVIES

LINE 0

Even before the turn of the century, movies began to develop in two major

1

directions: the realistic and the formalistic. Realism and formalism are merely

2

general, rather than absoluteterms. When using to suggest a tendency toward

3

either polarity, such labels can be helpful, but at the end they are still just

4

labels. Few films are exclusive formalist in style, and fewer yet are

5

completely realist. There is also an important difference between realism and

6

reality, although this distinct is often forgotten. Realism is a particular style,

7

where physical reality is the source of all the raw materials of film, both

8

realistic and formalistic. Virtually all movie directors go to the

9

photographable world for their subject matter, but what they do with this

10

material - how they shape and manipulate it - determines their stylistic

11

emphasis.

12

Generally speaking, realistic films attempt to reproduce the surface of

13

concrete reality with a minimum of distortion. In photographing objects and

14

events, the filmmaker tries to suggest the copiousness of life himself. Both

15

realist and formalist film directors must select (and hence emphasize) certain

16

details from the chaotic sprawl of reality. But the element of selectivity in

17

realistic films is less obvious. Realists, in short, try to preserve the illusion

18

that their film world is unmanipulated, an objective mirror of the actual world.

19

Formalists, on the other hand, make no such pretense. They deliberately 6 / 18


20

stylize and distort their crude materials so that only the very naive should mistake a manipulated image of an object or event for the real thing.

Your answers: Line

Mistakes

Corrections

1. 2. 3. 4. 5. 6. 7. 8. 9. 10.

Part 3. Complete the following sentences with suitable prepositions or particles. (10 points) 1. He’s sometimes bad tempered but he’s a good fellow _________ heart. 2.Don’t quote me. What I am about to say is _________ the record. 3. I was taken _________ when Michaela said I’d hurt her feelings. 4. If it starts to rain, make _________ a nearby cave to wait for it to pass. 5. Capital punishment was done ________ _________ in Britain nearly half a century ago. 6. When the funds finally petered _________, they had to abandon the scheme. 7. The dentist told me that when the effect of the anaesthetic wore _________, I might feel a little pain. But it was more than a little. 8. Everyone approved of the scheme but when we asked for volunteers they all hung _________. 7 / 18


9. Their predictions were not borne _________ by subsequent events. In fact, the very opposite occurred. 10. To prevent soldiers from rebelling, the commander splits them into groups to play them ________ _________.

Your answers: 1.

2.

3.

4.

5.

6.

7.

8.

9.

10.

Part 4. Read the text below. Use the word given in capitals to form a word that fits in the gap. (10 points) THE COOLING OCEANS The upper layers of Earth’s oceans have cooled (1) _______ SIGNIFY over the past two years, even though the planet as a whole is warming up. While this may just be part of the natural (2) VARY _______ of oceans, climatologists are still confounded by the massive unaccountable (3) _______ of heat. Scientists have been LOSE (4) _______ concerned by rising sea temperatures over the last 50 INCREASE years but those new (5) _______ tell a different story.

FIND

Generally speaking, the (6) _______ of heat by the oceans ABSORB reduces atmospheric warning. Now (7) _______ taken by the MEASURE National Oceanic and Atmospheric Administration have put a wrinkle in the trend. The researchers used data from 3000 floating buoys which monitor the oceans (8) _______.

WORLD

They found out that the oceans dropped in temperature by an (9) _______ 0.02 degrees centigrade between 2011 and 2013. Now, BELIEVE that may not seem like much, but trying to account for the missing energy is proving to be enormously (10) _______. It is possible PROBLEM that volcanic eruptions are one main cause of the phenomenon, but 8 / 18


no firm answers have yet been provided.

Your answers: 1.

6.

2.

7.

3.

8.

4.

9.

5.

10.

C. READING (50 points) Part 1. Read the following passage and decide which answer (A, B, C, or D) best fits each gap. (10 pts) THE VACUUM CLEANER Until about 250 years ago, households did not take dirt as seriously as they do now - it was a fact of life, and that was that. Cleaning often consisted of an annual (1) ______ called 'spring cleaning' when the furniture was moved aside, and all the linen products in the house were cleaned. Carpets and rugs were taken outside, hung on ropes and had the dust (2) ______ out of them - an exhausting and messy process. The industrial revolution brought about a major change - as new products became available to make homes cleaner, a corresponding interest in 'domestic hygiene' appeared in households. This in turn led to the (3) ______ of further products, one of which was the vacuum cleaner. (4) ______ has it that when one of the first vacuum cleaners was demonstrated, a kindly scientist took the proud inventor (5) ______ and offered a bit of advice that was to become crucial to the future evolution of the product - 'make it suck, not blow'. The first vacuum cleaners appeared in the 1860s in the United States. They were operated by hand pumps and were almost as (6) ______ as spring cleaning. It was only when electric motors had become sufficiently (7) ______ to become portable that vacuum cleaners became common household items. Most of today's major (8) ______ - including Electrolux and Hoover - were born in the 1920s. 9 / 18


The household dirt that vacuum cleaners suck up is mostly dead skin cells humans (9) ______ millions of cells every day. A much smaller proportion comes from dust and soil carried into the house from (10) ______. 1. A. ritual

B. result

C. resolution

D. scrub

2. A. cleaned

B. taken

C. beaten

D. sucked

3. A. fabrication

B. appearing

C. recreation

D. development

4. A. Story

B. Epics

C. Legend

D. Tale

5. A. away

B. aside

C. aback

D. along

6. A. laborious

B. hard

C. nefarious

D. straining

7. A. scientific

B. forward

C. technological

D. advanced

8. A. brand

B. marks

C. makes

D. trademarks

9. A. lose

B. outgrow

C. omit

D. shed

10. A. external

B. outside

C. beyond

D. indoors

Your answers:

1.

2.

3.

4.

5.

6.

7.

8.

9.

10.

Part 2. Read the text below and think of the word which best fits each space. Use only ONE word in each space. (15 points) THE LIMITS OF MAGIC IN HARRY PORTER “Harry Porter” is a series of novels written by British author J.K. Rowling. The novels chronicle the (1) _______ of a young wizard, Harry Porter and his friends Hermione Granger and Ron Weasley, all of whom are students at Hogwarts School of Witchcraft and Wizardry. Before publishing the first “Harry Porter” novel, J.K. Rowling spent five years establishing the limitations of magic; determining it could and could (2) ________ do. “The most important thing to decide when you’re creating a fantasy world”, she said in 2000, “is what the characters CAN’T do.” (3) ________ instance, while it is possible to conjure things out (4) ________ thin air, it is far more tricky to create something that fits an exact specification (5) ________ than a general 10 / 18


one; moreover, any objects so conjured tend not to last. It is also impossible to resurrect the dead. Dead bodies can be enchanted to (6) ________ a living wizard’s bidding, but these bodies remain mere puppets and have no souls. It is also possible via the rare Priori Incantatem effect to converse with ghost-like “shadows” of magically murdered people. The Resurrection Stone also allows one to talk to the dead, but (7) ________ brought back by the Stone are not corporeal, (8) ________ do they wish to be disturbed from their peaceful rest. Likewise, it is impossible to make oneself immortal unless one makes (9) ________ of a mystical object of great power to sustain life. If one were to possess the three Deathly Hallows, it is fabled that they would possess the tools to become the “master of death”. However, (10) ________ a true “master of death” is to be willing to accept that death is inevitable.

Your answers: 1.

6.

2.

7.

3.

8.

4.

9.

5.

10.

Part 3. Read the passage and choose the best option A, B, C, or D to answer the questions. (10 points) BRINGING UP CHILDREN Where one stage of child development has been left out, or not sufficiently experienced, the child may have to go back and capture the experience of it. A good home makes this possible - for example, by providing the opportunity for the child to play with a clockwork car or toy railway train up to any age if he still needs to do so. This principle, in fact, underlies all psychological treatment of children in difficulties with their development, and is the basic of work in child clinics. 11 / 18


The beginnings of discipline are in the nursery. Even the youngest baby is taught by gradual stages to wait for food, to sleep and wake at regular intervals and so on. If the child feels the world around him is a warm and friendly one, he slowly accepts its rhythm and accustoms himself to conforming to its demands. Learning to wait for things, particularly for food, is a very important element in upbringing, and is achieved successfully only if too great demands are not made before the child can understand them. Every parent watches eagerly the child's acquisition of each new skill: the first spoken words, the first independent steps, or the beginning of reading and writing. It is often tempting to hurry the child beyond his natural learning rate, but this can set up dangerous feelings of failure and states of anxiety in the child. This might happen at any stage. A baby might be forced to use a toilet too early, a young child might be encouraged to learn to read before he knows the meaning of the words he reads. On the other hand, though, if a child is left alone too much, or without any learning opportunities, he loses his natural zest for life and his desire to find out new things for himself. Learning together is a fruitful source of relationship between children and parents. By playing together, parents learn more about their children and children learn more from their parents. Toys and games which both parents and children can share are an important means of achieving this co-operation. Building-block toys, jigsaw puzzles and crosswords are good examples. Parents vary greatly in their degree of strictness or indulgence towards their children. Some may be especially strict in money matters; others are severe over times of coming home at night, punctuality for meals or personal cleanliness. In general, the controls imposed represent the needs of the parents and the values of the community as much as the child's own happiness and well-being. With regard to the development of moral standards in the growing child, consistency is very important in parental teaching. To forbid a thing one day and excuse it the next is no foundation for morality. Also, parents should realize that "example is

12 / 18


better than precept". If they are hypocritical and do not practise what they preach, their children may grow confused and emotionally insecure when they grow old enough to think for themselves, and realize they have been, to some extent, deceived. A sudden awareness of a marked difference between their parents' ethics and their morals can be a dangerous disillusion. 1. The principle underlying all treatment of developmental difficulties in children ________. A. is to capture them before they are sufficiently experienced B. is to send them to clinics C. is in the provision of clockwork toys and trains D. offers recapture of earlier experiences 2. Learning to wait for things is successfully taught ________. A. is achieved successfully by all children B. only if excessive demands are avoided C. because excessive demands are not advisable D. in spite of excessive demands being made 3. The encouragement of children to achieve new skills ________. A. can never be taken too far B. should be balanced and moderate C. should be focused on only at school D. will always assist their development 4. Parental controls and discipline ________. A. serve a dual purpose B. reflect only the values of the community C. are designed to promote the child’s happiness D. should be avoided as far as possible 5. The practice of the rule "Example is better than precept� _______. A. only works when the children grow old enough to think for themselves B. would help avoid the necessity for ethics and morals C. will free a child from disillusion when he grows up 13 / 18


D. is too difficult for all parents to exercise 6. In the 1st paragraph, the author lays some emphasis on the role of the ________ in helping the child in trouble. A. psychiatrists

B. community

C. family

D. nursery

7. The phrase “conforming to” in the 2nd paragraph means ________. A. agreeing with

B. following

C. adapting to

D. accepting

8. The word “zest” in the 2nd paragraph can be best replaced by ________. A. enthusiasm

B. enjoyment

C. excitement

D. appetite

9. The word “imposed” in the 3rd paragraph is closest in meaning to ________. A. introduced

B. made

C. excepted

D. constrained

10. Hypocrisy on the part of the parents may ________. A. disqualify their teachings altogether B. make their children lose faith in them C. result in their children’s wrong behaviour D. impair their children’s mind

Your answers: 1.

2.

3.

4.

5

6.

7.

8.

9.

10.

Part 4. Read the text and do the following tasks. (15 points) HOW DOES THE BIOLOGICAL CLOCK TICK? A. Our life span is restricted. Everyone accepts this as ‘biologically’ obvious. ‘Nothing lives for ever!’ However, in this statement we think of artificially produced, technical objects, products which are subjected to natural wear and tear during use. This leads to the result that at some time or other the object stops working and is unusable (‘death’ in the biological sense). But are the wear and tear and loss of function of technical objects and the death of living organisms really similar or comparable. B. Our ‘dead’ products are ‘static’, closed systems. It is always the basic 14 / 18


material which constitutes the object and which, in the natural course of things, is worn down and becomes ‘older’. Ageing in this case must occur according to the laws of physical chemistry and of thermodynamics. Although the same law holds for a living organism, the result of this law is not inexorable in the same way. At least as long as a biological system has the ability to renew itself it could actually become older without ageing; an organism is an open, dynamic system through which new material continuously flows. Destruction of old material and formation of new material are thus in permanent dynamic equilibrium. The material of which the organism is formed changes continuously. Thus our bodies continuously exchange old substance for new, just like a spring which more or less maintains its form and movement, but in which the water molecules are always different. C. Thus ageing and death should not be seen as inevitable, particularly as the organism possesses many mechanisms for repair. It is not, in principle, necessary for a biological system to age and die. Nevertheless, a restricted life span, ageing, and then death are basic characteristics of life. The reason for this is easy to recognise: in nature, the existent organisms either adapt or are regularly replaced by new types. Because of changes in the genetic material (mutations) these have new characteristics and in the course of their individual lives they are tested for optimal or better adaptation to the environmental conditions. Immortality would disturb this system — it needs room for new and better life. This is the basic problem of evolution. D. Every organism has a life span which is highly characteristic. There are striking differences in life span between different species, but within one species the parameter is relatively constant. For example, the average duration of human life has hardly changed in thousands of years. Although more and more people attain an advanced age as a result of developments in medical care and better nutrition, the characteristic upper limit for most remains 80 years. A further argument against the simple wear and tear theory is the observation that the time within which organisms age lies between a few days (even a few hours for unicellular organisms) and several thousand years, as with mammoth trees. E. If a life span is a genetically determined biological characteristic, it is 15 / 18


logically necessary to propose the existence of an internal clock, which in some way measures and controls the ageing process and which finally determines death as the last step in a fixed programme. Like the life span, the metabolic rate has for different organisms a fixed mathematical relationship to the body mass. In comparison to the life span this relationship is ‘inverted’: the larger the organism the lower its metabolic rate. Again this relationship is valid not only for birds, but also, similarly on average within the systematic unit, for all other organisms (plants, animals, unicellular organisms). F. Animals which behave ‘frugally’ with energy become particularly old, for example, crocodiles and tortoises. Parrots and birds of prey are often held chained up. Thus they are not able to ‘experience life’ and so they attain a high life span in captivity. Animals which save energy by hibernation or lethargy (e.g. bats or hedgehogs) live much longer than those which are always active. The metabolic rate of mice can be reduced by a very low consumption of food (hunger diet). They then may live twice as long as their well-fed comrades. Women become distinctly (about 10 per cent) older than men. If you examine the metabolic rates of the two sexes you establish that the higher male metabolic rate roughly accounts for the lower male life span. That means that they live life ‘energetically’ — more intensively, but not for as long. G. It follows from the above that sparing use of energy reserves should tend to extend life. Extreme high performance sports may lead to optimal cardiovascular performance, but they quite certainly do not prolong life. Relaxation lowers metabolic rate, as does adequate sleep and in general an equable and balanced personality. Each of as can develop his or her own ‘energy saving programme’ with a little selfobservation, critical self-control and, above all, logical consistency. Experience will show that to live in this way not only increases the life span but is also very healthy. This final aspect should not be forgotten.

A. Choose the correct heading for paragraphs B-G from the list of headings below. List of Headings 16 / 18


i. The biological clock ii. Why dying is beneficial iii. The ageing process of men and women iv. Prolonging your life v. Limitations of life span vi. Modes of development of different species vii. A stable life span despite improvements viii. Energy consumption ix. Fundamental differences in ageing of objects and organisms x. Repair of genetic material Example answer: Paragraph A v

B. Complete the notes below. Choose NO MORE THAN TWO WORDS from the passage for each answer. • Objects age in accordance with principles of (7) _________ and of (8) _________. • Through mutations, organisms can (9) _________ better to the environment. • (10) _________ would pose a serious problem for the theory of evolution.

Your answers 1. Paragraph B

____________

6. Paragraph G

2. Paragraph C

____________

7.

3. Paragraph D

____________

8.

4. Paragraph E

____________ 9.

5. Paragraph F

____________

____________

10.

D. WRITING (50 points) Part 1. Complete the second sentence so that it has a similar meaning to the first sentence, using the given word. DO NOT CHANGE THE WORD GIVEN. You must use between THREE and SIX words, including the word given. (10 points) 17 / 18


1. They say this company is one of the most reliable in the country. (REPUTED) This company _____________________________ of the most reliable in the country. 2. We were all surprised when she announced that she was engaged to be married. (ANNOUNCEMENT) The _____________________________________________________ all by surprise. 3. Finding the survivors is our number one priority. (UTMOST) It is of the _________________________________________________ the survivors. 4. Whatever happens, I will never trust him again. (EVER) Under ___________________________________________________ trust him again. 5. Tom didn’t feel like dancing that night. (MOOD) Tom _________________________________________________ dancing that night.

Part 2. Rewrite the sentences below in such a way that their meanings stay the same, starting with the provided word(s). (10 points) 1. Absolute secrecy was crucial to the success of the mission. Without _____________________________________________________________. 2. The two sides never looked likely to reach an agreement. At

no

time

was

________________________________________________________. 3. He is a complete hypocrite; in public he condemns smokers, yet he smokes a packet a day himself. So __________________________________________________________________. 4. Ruth never asks anyone for a loan as she doesn’t like to admit she has financial problems. Ruth is ______________________________________________________________. 5. He loses his temper at all things, even the slightest one. He flies ______________________________________________________________.

Part 3. Essay writing(30 points)

18 / 18


In some countries, people have boycotted fast food chains because they feel that they pose a threat to the national character of their cities. Write an essay in which you either support or oppose the growth of fast food chains around the world.

THE END

Ngườirađề: THÁI THỊ PHƯƠNG NGA (0946061199)

19 / 18


ĐÁP ÁN

TRƯỜNG THPT CHU VĂN AN HÀ NỘI

ĐỀ THI CHỌN HỌC SINH GIỎI DUYÊN HẢI ĐỒNG BẰNG BẮC BỘ 2019 MÔN: TIẾNG ANH - KHỐI 10 Đáp ángồm10trang ĐỀ XUẤT

A. LISTENING (50 points) Part 1. You will hear a telephone conversation between an operator and a caller. The caller is inquiring car insurance. Write NO MORE THAN THREE WORDS AND/OR A NUMBER for each answer. (10 points) CAR INSURANCE INQUIRY FORM

Name:

LIAM BYRD

Car model:

Ford Fiesta 2002

Registration:

3R1 (1) JTW

Current mileage:

(2) 90,000 miles

Estimated yearly mileage:

(3) 6,000 a year

Overnight parking:

(4) on the road

Cover:

Third party(5) fire and theft

No claims bonus:

6 years

Part 2. You will hear people talking in five different situations. Choose the best answer A, B or C. (10 points). 1. A

2. C

3. C

4. B

5. C

Part 3. You will hear a conversation which takes place in an open market between three people, Bob and Andrea Jones and Bob’s friend - Carl. Decide whether the statements are true (T) or false (F). (10 points) 1. T

2. F

3. T

4. F

5. F 1


Part 4: You will hear a psychologist speaking on British radio on the subject of astrology. Complete the sentences with NO MORE THAN THREE WORDS for each gap. (20 points) 1. British adults

6. artists and entertainers

2. business decisions

7. less obvious

3. character

8. winter

4. foundation / base

9. disappointing

5. choice of career

10. effects of nature

B. LEXICO - GRAMMAR (50 points) Part 1. Choose one of the words marked A, B, C, or D which best completes each of the following sentences. (20 points) 1.A

2.D

3.C

4.A

5.D

6.D

7.A

8.D

9.A

10.B

11.D

12.D

13.C

14.B

15.A

16.B

17.C

18.B

19.D

20.D

Part 2. Read the following text which contains 10 mistakes. Identify the mistakes and write the corrections in the corresponding numbered boxes.(10 points) Line

Mistakes

Corrections

1.

2

using

used

2.

3

at the end

in the end

3.

4

exclusive

exclusively

4.

6

distinct

distinction

5.

7

where

whereas/ while

6.

9

what

how

7.

13

himself

itself

8.

19

crude

raw

9.

19

should

would 2


10. 20

to

for

Part 3. Complete the following sentences with suitable prepositions or particles. (10 points) 1. at

2. off

3. aback

4. for

5. away with

6. out

7. off/ away

8. back

9. out

10. off against

Part 4. Read the text below. Use the word given in capitals to form a word that fits in the gap. (10 points) 1. SIGNIFICANTLY

6. ABSORPTION

2. VARIATION

7. MEASUREMENTS

3. INCREASINGLY

8. WORLDWIDE

4. LOSS

9. UNBELIEVABLE

5. FINDINGS

10. PROBLEMATIC

C. READING (50 points) Part 1. Read the following passage and decide which answer (A, B, C, or D) best fits each gap. (10 pts) 1. A

2. C

3. D

4. C

5. B

6. A

7. D

8. C

9. D

10. B

Part 2. Read the text below and think of the word which best fits each space. Use only ONE word in each space. (15 points) 1. lives

6. do

2. not

7. those

3. For

8.nor

4. of

9. use

5. rather

10. being

3


Part 3. Read the passage and choose the best option A, B, C, or D to answer the questions. (10 points) 1. D

2.B

3. D

4. A

5. C

6. C

7. A

8. C

9. D

10. B

Part 4. Read the text and do the following tasks. (15 points) 1. Paragraph B

ix

6. Paragraph G

2. Paragraph C

ii

7. physical chemistry

iv

(7 and 8 can be in either order) 3. Paragraph D

vii

8.thermodynamics

4. Paragraph E

i

9.adapt

5. Paragraph F

viii

10. immortality

D. WRITING (50 points) Part 1. Complete the second sentence so that it has a similar meaning to the first sentence, using the given word. DO NOT CHANGE THE WORD GIVEN. You must use between THREE and SIX words, including the word given. (10 points) 1. This company is reputed to be oneof the most reliable in the country. 2. The announcement of her engagement took us all by surprise. 3. It is of the utmost importance that we find the survivors. 4. Under no circumstances will I ever trust him again. 5. Tom wasn’t in the mood for dancing that night.

Part 2. Rewrite the sentences below in such a way that their meanings stay the same, starting with the provided word(s). (10 points) 1. Without absolute secrecy this mission would not have succeeded/ been successful/ been a success/ would have failed. 2. At no time was there any likelihood of the two sides reaching an agreement. 3. So great a hypocrite is he that in public he condemns smokers, while he smokes a packet a day himself. 4


4. Ruth is too proud to ask anyone for a loan as she doesn’t like to admit she has financial problems. 5. He flies off the handle at all things, even the slightest one.

Part 3. Essay writing(30 points) 1.

Content: (35% of total mark)

a. Providing all main ideas and details as required b. Communicating intentions sufficiently and effectively 2. Organization & Presentation: (30% of total mark) a. Ideas are well organized and presented with coherence, cohesion, and clarity b. The essay is well-structured 3. Language: (30% of total mark) a. Demonstration of a variety of vocabulary and structures appropriate to the level of English language gifted upper-secondary school students b. Good use and control of grammatical structures 4. Punctuation, spelling, and handwriting (5% of total mark) a. Good punctuation and no spelling mistakes b. Legible handwriting

5


TAPESCRIPT A. LISTENING (50 points) Part 1. You will hear a telephone conversation between an operator and a caller. The caller is inquiring car insurance. Write NO MORE THAN THREE WORDS AND/OR A NUMBER for each answer. (10 points) Operator (O): Good afternoon, Harline Car Insurance. Can I help you? Caller (C): Hello. I’d like to speak to someone about car insurance. O: Certainly, sir. Can I take your name please? C: It’s Liam. Liam Byrd. O: Okay Mr. Byrd. Is the policy for you? C: Yes, I bought the car last night. It’s still with the original owner though … until I get the insurance sorted out. Will I be able to do this over the phone now? O: Of course, sir. If you have all the information we need to process the application, we should be able to sort it out for you immediately. Could you tell me the make and model, Mr. Byrd. C: Yes, it’s a Ford Fiesta. O: And can I have the registration number please? C: Yes, It’s 3R1 JTW. O: Okay … I can see it’s a 2002 model, is that right? C: That’s right, yes. O: An what’s the current mileage Mr. Byrd? C: Very high unfortunately! 90,000 miles O: Okay. It’s obviously very reliable! C: Hopefully, yes. O: And how many miles will you be driving per year? C: Erm, probably about 6,000 a year. O: Okay. And where will the car be kept overnight? C: Well, I don’t have a garage or a driveway so it’ll be on the road. O: Okay, nearly there. When would you like the policy to be effective from? Immediately I suppose? 6


C: Yes. I’m hoping to drive the car home this evening so could I be covered from today? O: Yes, that’s fine. Now, do you want fully comprehensive cover or third party fire and theft? C: Just third party fire and theft. It’s only an old car. O: And how much no claims bonus have you got, sir? C: 6 years.

Part 2. You will hear people talking in five different situations. Choose the best answer A, B or C. (10 points). SITUATION 1: You are in a shop when you overhear this man answering the telephone. Hello, Hammerstones, can I help you? Yes, we can take an order. Is it a recent publication? … Okay, then, do you know the name of the author? … Aha, and the title? … “Getting the Most out of Your guitar”. Yes, I am familiar with that title. It is actually available in paperback now … Okay then … If you’d like to leave your name and telephone number, … SITUATION 2: You are listening to the radio when you hear this man speaking. Well Giles, what is currently happening in the Cumberland dry-dock dispute is nothing new. There has been a long history of industrial problems in British shipbuilding and this makes what the minister said wrong on a number of counts. In fact, this statement could be going to cause serious difficulties for the government. Rodwyn Lewis has managed to get a number of backs up in the union, which is not going to help his government solve the dispute. SITUATION 3: You are sitting in a café when you hear this woman speaking.

7


Well, I’m not in the slightest bit bothered about the cold weather this year. Not since Geoff got round to fitting the new windows. I mean the old ones were letting in the rain. Last winter I got so cold that I ended up having to wear a coat in our front living room and what with Samantha being only two-and-a-half well, we just had to do something about them. Anyway, Geoff’s marvelous when it comes to jobs round the house. I mean I had them up and finished in a week – good thing it wasn’t snowing then.

SITUATION 4: Listen to this woman introducing a college lecture. Good evening, everybody. Er, this won’t be very long. Er, I’d just like to say thank you to all of you for managing to turn up at such short notice. I’m sure you won’t be disappointed. I’m sure by now all of you must have heard about Anne’s staggering breakthoughin the field of micro-neurosurgery. And, I can tell you, as a personal friend of hers, that Anne started her career right here. So it’s very fitting that I should be introducing – right now – Anne suiter. SITUATION 5: You will hear some people talking about soap operas. Nowadays our TV’s seem to be showing soaps every day for most of the day. Soaps from America, Australia or the continent. The characters tend to be stereotypes and the plot is often repetitive and predictable. So why then do we rush back to our homes after work to see what has happened to the characters that we love to hate? Maybe because we actually enjoy the lack of challenge that they offer, and although we may protest that we aren’t really interested, they become a habit and we suffer from withdrawal symptoms if we don’t get our daily dose of bad acting. No one believes they are real characters of course – or do they? Many characters actually receive fan mail and if one dies, some fans are unhappy for weeks.

8


Part 3. You will hear a conversation which takes place in an open market between three people, Bob and Andrea Jones and Bob’s friend - Carl. Decide whether the statements are true (T) or false (F). (10 points) A: B: Bob C: Carl A: Look Bob! That’s a new stall we haven’t seen before. B: You mean the one with all those CDs and cassettes … hang on … hey! That’s Carl. A: You mean Carl who used to work with you at “Acres”? What’s he doing here? B: Must have started up a new business – though he’s the last person I’d have expected to … C: Hey, Bob! B: Quick, pretend we haven’t seen him. A: Oh, come on! He’s your friend, you can’t ignore him like that. C: Bo, what a great surprise … haven’t seen you for … must be years. B: Carl. Good to see you mate. Er … This is my fiancée, Andrea. C: Oh, pleased to meet you. Are you interested in country music? I know Bob’s not too keen. A: Is that what all this is? All country music? Have you got any Irish Folk music? B: Oh, God. C: I did have, but it didn’t seem to sell too well. B: I’m not surprised. C: So I got rid of it all – pity really. B: Good thing if you ask me … And, how long have you been doing this? C: You mean the stall? Oh, about two months, after “Acres” closed down, I was out of work for about a year, and then one day, I got talking to my claims officer … er … about my collection … and he suggested that I did this … and so far it’s been quite profitable. A: Do you make a profit? C: Last week I took a bit over 3,000 pounds. B: Three thousand quid! C: Well, some of it has to pay overheads and VAT but I should get to keep at least half of it. 9


A: Was it difficult to get started? What did you have to do? C: No, no, it was quite easy really. I just talked to a “small business advisor” at the bank and luckily, I still had some redundancy money left so there was no need for a loan or anything … anyway what about you? B: Oh, I spent my redundancy money on a holiday. A: And the car! B: Well, yeah, yeah, it didn’t quite cover the car so I had to take out a loan, which I’m still paying off – you see the new job pays even less than “Acres” did. I guess I should have done something like you have. A: You still could. I’m sure the bank could extend you a loan if you asked. Have you seen Bob’s silverware? … It’s really quite unique. C: Er yeah, yes, he did show me some pictures once. Come to think of it, there’s a man who sells similar stuff at a car boot sale in Blackpool, where I do a stall on Sundays. A: There, you see. You could do a stall on Sundays and keep on your job at the bakery. B: Oh, er .. I don’t know about that. I’m not sure all that standing around in the cold would be good for my circulation. The doctor said I need to stay in a warm environment and keep moving. A: You’re just making excuses now. This is a fantastic chance to make some money …

Part 4: You will hear a radio talk for young people about animals communicating with each other. Complete the sentences with NO MORE THAN THREE WORDS for each gap. (20 points) Introducer: We have with us today the psychologist Simon Calvin who has recently created quite a stir with some rather unlikely claims about Astrology. Simon, are you indeed claiming that our lives are ruled by the stars? Simon: No, not at all. I would not for one minute say that all of our lives are affected by the movements of stars in the heavens. My research really concerns things 10


that are far move down to earth. But, before I start, I should just say that by no means do the majority of people in general totally disbelieve the value of astrology. An incredible 62% of British adults say that their stars are of some interest to them although only 3.5% would use them to choose a husband or wife, and only 2.9% of people would refer to them in making business decisions. But anyway, coming back to what I said before, I’m not so interested in the stars themselves, as I am in the time of year somebody is born. Psychologists now widely agree that both early life experiences and time of birth are a great influence on the development of a person’s character. Of course, this does not make the whole of a person’s character, but rather the foundation or base on which later changes are made. However, these later changes in character are not connected with the person’s time of birth. So the question is, then, after all these personality changes have happened, can we look at an adult and see any of this foundation or base as part of their psychological make-up? The technique I have been using to do just this is a very simple one, but I believe, very effective. I have simply made a statistical comparison between people’s choice of career and their zodiac sign. Where somebody has made a strong choice in favour of a particular kind of career, I believe that this indicates something very important in their character. If you look at a particular profession, say entertainers for example, and find that more than 15% of them were born in a certain short period of the year, it suggests that their choice of that profession is in some way connected with their birth at that particular time of the year. And this is, in fact, the case. An astonishing 20% of artists and entertainers were born in the period between the 12th of July and the 20th of August. This, in fact, is the most positive connection we have found so far. But there have been others although they have been less obvious. We looked at keen sports players participating more than five times a week and found a sizable number were born in the winter months of January and February. Another group of people we looked at were frequent travelers, who we discovered were more likely to have been born in early spring while accountants, bankers, executives and people generally in financial professions more frequently have birthdays in late spring. Now, so far, we haven’t found any professions containing large percentages of one particular star sign, which 11


is a little disappointing, but this is, no doubt, because our modern technological world has removed us further and further from the effects of nature, we could be seeing percentages as high as 40 or 50 or even more. This of course would mean that we would have …

THE END

Ngườirađề: THÁI THỊ PHƯƠNG NGA (0946061199)

12


SỞ GD&ĐT BẮC GIANG TRƯỜNG THPT CHUYÊN BẮC GIANG (Đề thi đề xuất)

ĐỀ THI CHỌN HỌC SINH GIỎI VÙNG DUYÊN HẢI - ĐỒNG BẰNG BẮC BỘ NĂM HỌC 2018-2019 Môn: Tiếng Anh – Lớp 10 (Thời gian: 180 phút – không kể thời gian giao đề)

A. LISTENING (50 points) Part 1. Complete the notes below. For question 1-5, write NO MORE THAN TWO WORDS AND/OR A NUMBER for each answer. (10 points) APPLICATION FORM Example:

Answer

Years of membership:

seven

Type of I.D I.D No: Family name: Other name: D.O.B: I.D. expiry date: Address: Class of vehicle: Endorsement: Conviction:

Your answers: 1.

2.

1 ____________ 2. _____________ Black Gavin Raymond 22/01/1973 3. _____________ 4. _____________ Meadowbank motor car None 5. ______________ (1993)

3.

4.

5.

PART 2: You will hear an interview with Alex Mustard, an underwater photographer who has just published a successful book featuring his work. For questions 1- 5, choose the answer (A, B, C or D) which fits best according to what you hear. 1. What does Alex say about filming wildlife underwater? A. It’s impossible to plan what you’ll see. B. It’s fairly easy to set up controlled shots.


C. You have to work closely with other people. D. You have to be quick to exploit opportunities. 2. What does Alex say about his favourite shots in the book? A. They are the ones he had most fun shooting. B. It was hard to get the divers to take them seriously. C. They may embarrass some of the divers who feature in them. D. It's unusual to see shots that show the light-hearted side of diving. 3. How does Alex feel about photographing dangerous creatures underwater? A. He tries to avoid looking aggressive in their company. B. He dislikes the idea of disturbing them in any way. C. He recognises the need to avoid certain species. D. He can tell if an animal is likely to attack him. 4. Alex says that anyone wanting to take up underwater photography should A. choose subjects that are likely to keep relatively still. B. buy the most expensive equipment they can. C. avoid getting too close to their subjects. D. take shots of small things at first. 5. For Alex, the main attraction of going to Sardinia is A. the number of submerged wrecks he can explore. B. the convenient location of the main dive sites. C. the range of underwater life that he can see. D. the temperature and clarity of the water. Your answers: 1. 2. 3. 4. 5. PART 3: Listen to a customer speaking to telephone support. Answer the questions choosing either "True" (T) or "False" (F). ____ 1. Jackie was surprised that David had problems placing his order. ____ 2. David needs to order the software for his office. ____ 3. Jackie gives him the 25% discount even though he's not ordering online. ____ 4. Jackie tells David that the free microphones are usually not very good quality. ____ 5. David buys both versions of the software. Your answers: 1. 2. 3. 4. 5. PART 4: You will hear part of a radio interview with a man who is the director of an Environmental Centre. For questions 1-10, write NO MORE THAN THREE WORDS to complete the sentences. The Environmental Centre has been open for (1) ______________ The Centre has working displays of sun and (2) _____________ School children visit the Centre to carry out (3) _____________ on the environment.


The majority of courses take (4) _____________ apart from ones in the summer. Accommodation is provided in (5) _____________ made of wood. The Centre does not allow anyone (6) __________. The course on garden wildlife and different plants is called (7) _____________. The most popular course shows people how (8) _____________. The cost of a course depends on whether people have (9) _____________ or not. The Centre has (10) _____________to make sure nobody is refused a place on a course. Your answers: 1. 6.

2. 7.

3. 8.

4. 9.

5. 10.

B. LEXICO-GRAMMAR (50 points) Part 1. Choose the best option A, B, C, or D to complete the following sentences and write your answers in the corresponding numbered boxes. (20 points) 1. Several hundred people have signed the petition to put a ______ to the nuclear test in the region. A. stop B. finish C. break D. cease 2. There is little doubt that homocides still continue to be a ______ question in a number of countries. A. obstructing B. nagging C. burdening D. contending 3. I was awfully tired. However, I made up my mind to _____ myself to the tedious task once again. A. involve B. absorb C. engross D. apply 4. Why do you always ask me a favour when I have got my ______ full? A. palms B. brains C. hands D. arms 5. She summed up Henry’s achievements in a few ______ phrases. A. felicitous B. utilitarian C. loquacious D. ominous 6. ______ account should you leave the building without notifying a member of staff. A. Of little B. Of no C. Not on any D. Not on your 7. When he found out that he had been rejected, he cut up ______ A. raw B. rough C. loose D. harsh 8. The demonstrators asked the ______ to join in the demonstration. A. witnesses B. sightseers C. onlookers D. viewers 9. Frank said his brand new Ferrari could do 250 kph and Tony, not to be ______, claimed his Porshe could drive at 300 kph. A. overriden B. outdone C. downgraded D. outspoken 10. It isn’t easy to obtain admission to this university, to say ______ of the costs you have to incur. A. but a few B. nothing more C. not so much D. only a little


11. I think it was completely thoughtless of you to have turned your _____ on the friends asking for your help. A. fist B. neck C. head D. back 12. I know you don’t like Ernest very much, but still you have to ______ it to him that he is conscientious worker. A. return B. hand C. award D. admit 13. After the break-up, her mood ______between hope and despair. A. transfixed B. radiated C. emanated D. vacillated 14. The burglar’s presence was betrayed by a _____floorboard. A. crackling B. crunching C. groaning D. creaking 15. There is a pressing need to make some rarefied subjects of research intelligible to those living beyond the_________tower. A. ivory B. silver C. diamond D. golden 16. Desirous of achieving promotion, she set aside all her personal plans and entered a six-month_________course. A. retraining B. reskill C. refresher D. reconstruction 17. We can’t apply the same theory to this situation – it is__________opposite to the one we encountered last month. A. diabolically B. diagonally C. diametrically D. diachronically 18. After the earthquake, the entrance hall was turned into a _________ casualty ward. A. mainstay B. makeshift C. piecework D. wayside 19. She was on her ______ throughout the interview because she didn’t want to say anything stupid. A. defence B. lookout C. caution D. guard 20. – So, are you going to buy that flat or not? - I’m not sure yet. I’m ______ about it. ‘’ A. between two minds B. between two tools C. in two minds D. in two ideas Your answers: 1. 6. 11. 16.

2. 7. 12. 17.

3. 8. 13. 18.

4. 9. 14. 19.

5. 10. 15. 20.

Part 2. Read the passage below which contains 10 mistakes. Identify the mistakes and write the corrections in the corresponding numbered boxes. (10 points) Line 1 It is an accepted part of everyday nostalgic to assume that in the past 2 food was somehow better than it is today. The fruit and vegetables were more 3 naturally grow and this was not seen as an extra bonus which added ten per 4


5 6 7 8 9 10 11 12 13 14 15 16 17 18 19 20 21 22

sent on to the price. Most food was fresh, not frozen, and you had the chance to examine it to see whether you wanted it. When you went shopping you could ask for exactly what piece of meat you wanted and see the butcher to cut it instead of finding it ready-wrapped in plastic. And your local trademan soon, got to know what you wanted, and provided it for you; otherwise he would have gone out of business. Of course, unless we invent time-travel we shall never know whether this is all true. Survivors from those distant days naturally tend to dislike today's convenient foods, and to prefer the Good Old Days when a joint of beef filled the oven, produced thick red juice instead of water when cooked, and cost the same as a can of Coke. What is always forgotten is that then as now the quantity of your food depended very much upon who you were, how well-off you happened to be, and where you lived. Shopping then demanded considerable skills, and shoppers had to be able to tell the fresh from the not so fresh. Their was no sell-buy date to act as a guide. If you were hard up then frozen meat and canned foods would have been in the menu, just as they are today.

Your answer: Line

Mistake

Correction

Line

Mistake

Correction

Part 3. Complete each sentence with one suitable particle or preposition. Write your answers in the box provided. (10 points) 1. It’s almost impossible to specify how mnay fake banknotes are currently ______ circulation. 2. “Why does he look so depressed?” “He must be ______ his wits end because of his wife’s illness, I suppose” 3. It’s rather ______ my comprehension how they have succeeded in growing fruit trees on this barren soil.


4. “Why didn’t you answer the phone? “Probably, I wasn’t ______ call when it rang. I must have been outside the house” 5. Now that the case is ______ my hands, I don’t need to worry any more. The other barrister has taken it over. 6. Eddie got into serious trouble after he had treated himself ______ too many books and had no money left for the bus back home. 7-8. I’m not convinced Arthur’s comments had much to do _____ the affair. To me, they were rather _____ the point. 9. I know Oete’s conduct was intolerable, but don’t be too hard ______ him. 10. I’m extemely pressed ______ money these days. Could you lend me a few pounds, please? Your answers: 1. 2. 3. 4. 5. 6. 7. 8. 9. 10. Part 4. Write the correct form of the words given in the brackets. Write your answers in the spaces provided below. (10 points) New food labelling system The government may soon forcce restaurants to introduce a “traffic light” labelling system on menus to help slow down rising levels of (0. OBESE) ______ . Agreen circle would show food is safe and (1. ADVICE) ______ to eat, while amber foods should be eaten in moderation and red foods eaten occasionally. Supermarkets already (2. VOLUNTEER) ______ follow a similar code, but minister believe a radical (3. EXTEND) ______ of the system to restaurant chains is necessary. They intend to (4. HARD) ______ government policies on public health as recent statistics show that two thirds of adults are (5. WEIGH) ______ or worse, obese. Abroad, New York city has already forced restaurant chains to list calorie content on mennus, believing this will lead to a dramatic (6. REDUCE) ______ in the number of people who are obesse or who suffer from diabetes. If the government here goes ahead with the “traffic light” system it will no doubt face considerable (7. RESIST) ______ from the food industry, which would be forced to spend money on (8. DATE) ______ menus. This new system would also be (9. CONTROVERSY) ______ for small restaurant chains because menus are (10. SEASON) ______ and constantly change, and many dishes do not contain standardized levels of ingredients. Answers: 1. 6. 2. 7. 3. 8. 4. 9. 5. 10.


C. READING (50 points) Part 1. Read the following passage and decide which answer (A, B, C, or D) best fits each gap. Write your answers in corresponding numbered boxes. (10 points) MASTER THE ART OF BEING EFFECTIVE I suffer from a debilitating condition called "procrastination"- the ability to put off endlessly the things I have to do. Each morning when I wake up, my mind launches into its own decision-making(1) ______ . Shall I get up or shall I press the snooze button? By the time I (2) ______ to what I should wear, the complexity of the decision would need an advisory (3) ______ to solve it. I’m already hours behind (4) ______ , and I haven’t even eaten my breakfast yet. I have always been a procrastinator. I am tortured by menus and holiday brochures, paint color (5) ______ and satellite television. So much so that I decided to invest in a new book on the subject, which (6) ______ it could treat my condition. According to the book, I should identify my weaknesses, then become more productive and develop priorities that (7) ______ my personal goals. My problem, I am told, is that I put off doing something because I fear the outcome. But even filling in a tax form is rarely as dreadful as we think. Apparently, each day I should think of something I don’t want to do, (8) ______ to it, then use a kitchen timer to establish how long it actually took. If I still can’t face the task, I should visualize newspaper headlines (9) ______ my achievement. So when do I start my new (10) ______? That’ a big problem. I just can’t decide. 1. A. course B. process C. development D. case 2. A. lead B. arrive C. get D. approach 3. A. committee B. commission C. community D. communication 4. A. timetable B. schedule C. programme D. agenda 5. A. maps B. diagrams C. graphs D. charts 6. A. claimed B. demanded C. challenged D. pretended 7. A. exhibit B. imitate C. display D. reflect 8. A. stand B. persevere C. persit D. stick 9. A. announcing B. notifying C. informing D. stating 10. A. structure B. control C. regime D. management Your answers: 1. 2. 3. 4. 5. 6. 7. 8. 9. 10. Part 2. Read the following text and fill in the blank with ONE suitable word. Write your answers in corresponding numbered boxes. (15 points) There is still some (1) ______ between research and coaching practice. Sports psychology has been able to develop a relatively significant research (2) ______ in the last 15 years; aided by general experimental researchers often using athletes as an easily


identifiable and obtainable population. Yet, as a practising sports psychologists, I recently observed an athletics coach, whose (3) ______ to a promising middledistance runner losing a winning position on the last lap was to prioritize developing a (4) ______ finish. What he didn’t address was the fact that the runner failed to focus when he got (5) ______ with emotions. Then (6) ______, I also witnessed some baseball coaches doing some work – which I would have taken (7) ______ in my professional capacity – (8) ______ assessing and profiling strengths and weaknesses, and also on performance anxiety. So things vary and some trainers are clearly more knowledgeable than others. But it is not surprising that, as a result, public conceptions are often confused on the issue and even attach (9) ______ connotations to sports psychologists and (10) ______ the nature of seeing them. Your answers: 1. 6.

2. 7.

3. 8.

4. 9.

5. 10.

Part 3. Read the following passage and circle the best answer to each of the following questions. Write your answers in corresponding numbered boxes. (10 points) TO SPANK OR NOT TO SPANK UP - Minneapolis, MN - A father was recently arrested by the police for spanking his child, starting a debate among the American public about spanking. Is spanking, or other types of corporal punishment, an acceptable form of discipline for children? Or is it a form of child abuse? The case that has everyone talking is the arrest of Dale Clover, a thirty-six-year-old father of three, at a shopping mall in St. Louis, Missouri. He was arrested after an employee at the mall saw him spanking his five-year-old son, Donny, and called the police. The father was arrested for child abuse. Mr. Clover admits that he hit his son but says that it wasn’t child abuse. He says it was discipline. Across the country, parents ,disagree on this issue: What is the difference between loving discipline and child abuse? Some parents like Rhonda Moore see a clear difference between spanking and child abuse. Rhonda Moore believes a little bit of pain is necessary to teach a child what is right and wrong. “It’s like burning your hand when you touch a hot stove. Pain is nature’s way of teaching us.” Moore believes that spanking is done out of love, but child abuse is done out of anger, when the parent loses control. “When I spank my children, I always talk to them before and afterward, and explain why they are being spanked. I explain what they did wrong, and they remember not to do it again.” Moore says that her children respect her as a parent and understand that she is spanking them for their own good. In contrast, Taylor Robinson, father of four, feels that parents should never hit their children for any reason. Robinson wants his children to learn right and wrong, but not


because they are afraid of being hit. “Spanking teaches children to fear their parents, not respect them. When a parent spanks a child, what the child learns is that problems should be solved with violence.” Robinson believes that children learn that it is acceptable for parents to hurt their children. “None of these are lessons that I want to teach my children. I want my children to learn to talk about their problems and solve them without violence, but spanking doesn’t teach that.” Parents are split about corporal punishment, and doctors also disagree about the issue. Dr. John Oparah thinks our child abuse laws sometimes go too far: that is, they make it difficult for parents to discipline their children. Oparah says that today many children do not respect their parents. “Children need strong, loving discipline. Sometimes spanking is the best way to get a child’s attention, to make sure the child listens to the parent.” Oparah says that he knows loving parents who have had police officers come to their home because their children have reported being spanked. Oparah believes parents should not be treated like criminals. “As a society, we complain all the time that your young people are getting into more and more trouble, committing crimes - yet when parents try to control their children, they’re punished. Some parents are afraid to discipline their children because their neighbors might call the police.” Most doctors, however, say that there are many harmful effects of spanking. Dr. Beverly Lau is opposed to spanking. Lau argues that spanking can lead to more violent behavior in children. She points to research that shows that children who are spanked are more violent when they grow up. “A child may stop misbehaving for the moment, but over time, children who are spanked actually misbehave more than children who are not spanked.” Lau adds that research shows that if you want a peaceful family, parents should not spank their children. The issue of spanking and corporal punishment will continue to be debated among parents and in the courts. In the meantime, if he is convicted of child abuse, Dale Clover could get up to five years in prison. 1. What is the main idea of this passage? A. Doctors believe that spanking is harmful to children. B. Parents and doctors disagree about the benefits of spanking children. C. Spanking teaches children the difference between right and wrong. D. Child abuse laws make it difficult for parents to discipline their kids. 2. In paragraph 2, who called the police in the Dale Clover case? A. His five-year-old son B. A mall employee C. Dale Clover himself D. Clover’s wife 3. In paragraph 3, why does Rhonda Moore give the example of touching a hot stove? A. To argue that parents need to control their children B. To demonstrate that spanking is wrong , C. To question the laws about child abuse D. To show the benefits of spanking 4. Why does the author include Rhonda Moore and Taylor Robinson in this article?


A. To give examples of parents who spank their children B. To compare the views of a parent and a doctor C. To contrast the opinions of two parents D. To argue against spanking one’s children 5. In paragraph 4, all of the following are lessons Taylor Robinson believes children learn from spanking EXCEPT_______. A. children should fear their parents B. problems could be solved with violence C. it is acceptable for parents to hurt their kids D. people should talk about problems, without violence 6. In paragraph 5, the phrase “go too far” is closest in meaning to _______ . A. are too hard on children B. take control away from parents C. are not strong enough to stop abuse D. teach children to respect for their parents 7. In paragraph 6, why does the author use the word “however” A. To contrast Oparah’s view from other doctors’ B. To question the views of most doctors C. To support Oparah’s opinions D. To introduce a similar opinion 8. In paragraph 6, what is the author’s primary purpose? A. To question Beverly Lau’s opinions about spanking B. To explain why parents should spank their kids C. To compare two different views on spanking D. To show the harmful effects of spanking 9. In paragraph 7, what can be inferred about Dale Clover’s case? A. Dale Clover is guilty of child abuse. B. Dale Clover’s case has not ended yet. C. It is likely that Clover will be convicted. D. Clover might be in jail longer than five years 10.Why does the author use the example of Dale Clover? A. To support the idea that spanking niakes a family stronger B. To support the idea that spanking is a controversial subject C. To support the idea that spanking is necessary for disciplining children D. To support the idea that spanking teaches children to know right from wrong Your answers: 1. 2. 3. 4. 5. 6. 7. 8. 9. 10. Part 4. Read the following extract and answer questions 1-10. (15 points) The history of writing


Paragraph 1 ______ The earliest stage of writing is called pre-writing or proto-literacy, and depends on direct representation of objects, rather than representing them with letters or other symbols. Evidence for this stage, in the form of rock and cave paintings, dates back to about 15,000 years ago, although the exact dates are debatable. This kind of proto-literate cave painting has been found in Europe. With the best known examples in Southwestern Prance, but also in Africa and on parts of the American continent. These petrographs (pictures on rock) show typical scenes of the period, and in clude representations of people, animals and activities. Most are astonishingly beautiful, with a vibrancy and immediacy that we still recognise today. They are painted with pigments made from natural materials including crushed stones and minerals, animal products such as blood, ashes, plant materials of all kinds, and they produce a wide range of colours and hues. Paragraph 2 ______ Why did ancient people put such effort into making them? Various theories have been put forward, but the most compelling include the idea that the pictures were records of heroic deeds pr important events, that theu were part of magical ceremonies, or that they were a form of primitive calendar, recording the changes in the season as they happened. These, then, are all explanation aas to why man started to write. Paragraph 3 ______ A related theory suggests that the need for writing arose thereafter from the transactions and bartering what went on.In parts of what is now Iraq and Iran, small pieces of fired earth-pottery have been found which appear to have been used as tokens to represent bartered objects, much as we use tokens in a casino, or money, today. Eventually, when the tokens themselves became too numerous to handle easily, representations of the tokens were inscribed on clay tablets. Paragraph 4 ______ An early form of writing is the use of pictograms, which are pictures used to communicate. Pictograms have been found from almost every part of the world and every era of development, and are still in use in primitive communities nowadays. They represent objects, ideas or concepts more or less directly. They tend to be simple in the sense that they are not a complex or full picture, although they are impressively difficult to interpret to an outsider unfamiliar with their iconography, which tends to be localized and to differ widely from society to society. They ere never intended to be a detailed testimony which could be interpreted by outsiders, but to serve instead as aside-memoires to the author, rather as we might keep a diary in a persoanl shorthand. However, some modern pictograms are more or less universally recognised, such as the signs which indicate men’s and women’s toilets, or road signs, which tend to be very similar throughout the world. Paragraph 5 ______


The first pictograms that we know of are Sumerian in origin, and date to about 8000 BC. They show how images used to represent concrete objects could be expanded to include abstractions by adding symbols together, or using associated symbols. One Sumerian pictogram, for example, indicates ‘death combining the symbols for “man” and “winter”; another shows “power” with the symbol for a man with the hands enlarged. Paragraph 6 ______ By about 5,000 years ago, Sumerian pictograms had spread to other ares, and the Sumerians had made a major advance towards modern writing with the development of the rebus principle, which meant that symbols could be used to indicate sounds. This was done by using a particular symbol not only for the thng it originally represented, but also for any thing which was pronounced in a similar way. So the pictogram for na (meaning ‘animal’) could also be used to mean ‘old’ (which was also pronounced na). The specific meaning of the pictogram (whether na meant ‘old’ or ‘animal’) could only be decided through its context. Paragraph 7 ______ It is a short step from this to the development of syllabic writing ysing pictograms, and this next development took about another half a century. Now, the Sumerians would add pictograms to each other so that each, prepresenting an individual sound or syllable, formed part of a larger word. This pictograms representing the syllables he, na and mi (‘mother, ‘old’, ‘my’) could be put together to form hemani or ‘grandmother’ (15 days practice for IELTS reading) For questions 1-6, choose correct heading for paragraphs 1-7 from the list of headings. Write your answers in the corresponding numbered boxes. There is an example. List of Headings A. Magic and heroes B. Doing business C. Early developments D. Sounds and symbols E. Image on stone F. Stories and seasons G. From visual to sound H. A peronal record 1. Paragraph 1: ______ 2. Paragraph 2: ______ 3. Paragraph 3: ______ Paragraph4: __G___

4. Paragraph 5: ______ 5. Paragraph 6: ______ 6. Paragraph 7: ______


For questions 7-10, complete the following notes by using ONE or TWO WORDS from the reading passage for each answer. Notes on the development of writing First stage of writing – pre-writing or proto-literacy – very old- 15,000 years. Evidence: cave and rock paintings. Famous example – (7) ______________________. Reasons for development of writing: primitive ceremonies, recording events, seasons, used on pottery to represent (8) ______________________. Next stage: very simple pictograms – pictures used to represent articles and (9) __________________, very simple drawings (but very difficult to understand). Then – 8000 BC – combined (1) __________________ to create new concepts (e.g. man + winter = death). After this – started using same pictogram for different words with same sound, very important step. D. WRITING (50 PTS) Part 1. Rewrite the sentences so that they have the same meaning as the ones before them. 1. All the passengers of the jumbo jet were killed in the crash. (8) None _______________________________________________________________ 2. This matter is so complicated that we don’t know how to deal with it. 11 So __________________________________________________________________ 3. Although she seems to be very courteous, she can also be very impolite. 13 Courteous ____________________________________________________________ 4. Someone suggested that we should stop manufacturing the low-price items. 15 There ________________________________________________________________ 5. Nobody knows what caused the collapse of the building. 16 It’s __________________________________________________________________ Part 2. . Rewrite each sentence using the word in brackets so that the meaning stays the same. You must use between THREE and EIGHT words, including the word given. (10 points) 1. I don’t usually learn anything useful from the television, but last night I did. (CHANGE) I ______________________________ useful from the television last night. 2. Simon doesn’t agree with me about which slogan is best for the campaign. (DIFFERENCE) Simon ________________________ opinion over which slogan is best for the campaign. 3. Senator Smith announced his intention to resign a short time ago. (KNOWN) Senator Smith has just ____________________________ to resign. 4. It was wrong of you not to tell me about that information. (KEPT) You should ______________________________ me. 5. Jepdee needs to arrive soon or we’ll have to go without him. (BEHIND) We’ll have to ______________________________ turns up soon.


Part 3. Writing an essay (about 250 words) People say that nowadays, modern teens are too much reliant on Information Technologies. To what extent, do you agree or disagree with this statement? Nguời ra đề: Thân Thị Ngọc Thủy – Vũ Thị Kim Thủy Số đt: 0982584582


ĐỀ THI CHỌN HỌC SINH GIỎI VÙNG DUYÊN HẢI - ĐỒNG BẰNG BẮC BỘ NĂM HỌC 2018-2019 Môn: Tiếng Anh – Lớp 10 (Thời gian: 180 phút – không kể thời gian giao đề)

SỞ GD&ĐT BẮC GIANG TRƯỜNG THPT CHUYÊN BẮC GIANG (ĐÁP ÁN Đề thi đề xuất) A. LISTENING (50 points) PART 1: 1. Driver licence

4. 29 Sunset street/St

2. AY196310

5. dangerous driving

3.19/05/2012 or 19(th) May 2012 PART 2: Cae-practice test plus- test 2- part 3- p52 1. D

2. D

3. B

4. A

5. B

4. T

5. F

PART 3: 1. F

2. T

3. T

PART 4: Fce- practice test plus 2- test 1- part 2-p12 1. 25 years

6. to smoke inside

2. wind power.

7. natural gardening

3. a project

8. to save water

4. 3/three days

9. a job

5. basic cabins

10. a fund

B. LEXICO-GRAMMAR (50 points) Part 1 (20 pts) 1. A 2. C 3. D 4. C 5. A 6. C 7. B 8. C 9. B 10. D 11. D 12. B 13. D 14. D 15. A 16. C 17. C 18. B 19. D 20. C Part 2 (10 pts) Line

Mistake

Correction

Line

Mistake

Correction

1

Nostalgic

Nostalgia

14

Convenient

convenience

4

Grow

Grown

17

Quantity

quality

5

Sent

cent

21

Their

There

7

To cut

Cutting/ cut

21

Sell-buy

Sell-by


8

Trademan

tradesman

22

In

On

Part 3 (10 pts) 1. in 6. To

2. At 7. With

3. Beyond 8. Off

4. Within 9. On

5. Off 10. for

Part 4 (10 pts) 1. Advisable 6. reduction

2. Voluntarily 7. Resistance

3. Extension 8. Updating

4. Harden 5. Overweight 9. Controversial 10. Seasonal

C. READING (50 points) Part 1 1. B 2. C 3. A 4. B 5. D 6. A 7. D 8. D 9. A 10.C Part 2 1. distance

2. Base

3. Reaction

4. Sprint

5. overcome

6. again

7. Pride

8. On

9. Negative

10 misunderstand

Part 3 1. D

2. B 3. D 4. A 5. D 6. C 7. B 8. A 9. C 10. A

Part 4 1. E

2. F

7. Southwestern France

3. B

4. C 8. Bartered objects

5. D

6. H 9. Concepts/ ideas

10. Symbols

D. WRITING (50 PTS) Part 1 1. None of the passengers of the jumbo jet survived the crash. 2. So complicated is the matter that we don’t know how to deal with it. 3. Courteous though/ as she seems to be, she can also be very impolite. 4. There was a suggestion that we should stop manufaturing the low-price items. 5. It’s unknown/ uncertain what caused the collapse of the building. Part 2 1. I made a change to learn something …. 2. Simon and I have a difference of …. 3. Senator Smith has just made it known that he intends …/ made known his intention 4. You should not have kept that information from me. 5. We’ll have to leave Jepdee behind unless he turns up soon


Part 3 Nguời ra đề: Thân Thị Ngọc Thủy – Vũ Thị Kim Thủy Số đt: 0982584582


SỞ GD&ĐT BẮC NINH TRƯỜNG THPT CHUYÊN BN

ĐỀ THI HỌC SINH GIỎI DHBB NĂM HỌC 2018 - 2019 Môn thi: Tiếng Anh 10 Thời gian: 180 phút (không kể thời gian giao đề)

ĐỀ ĐỀ XUẤT

SECTION A: LISTENING (50 points) HƯỚNG DẪN PHẦN THI NGHE HIỂU • Bài nghe gồm 4 phần, mỗi phần được nghe 2 lần, mỗi lần cách nhau 15 giây, mở đầu và kết thúc mỗi phần nghe có tín hiệu nhạc. • Thí sinh có 3 phút để hoàn chỉnh bài trước tín hiệu nhạc kết thúc bài nghe. • Mọi hướng dẫn cho thí sinh (bằng tiếng Anh) đã có trong bài nghe. Part 1: Listen to a woman calling an insurance company to report an accident. For questions 1-5, write NO MORE THAN THREE WORDS AND/OR A NUMBER for each answer. Write your answers in the corresponding numbered boxes provided. (10pts) (Adapted from https://www.ielts-exam.net/practice_test) TRAVEL SAFE Insurance Plc Department: (1). __________________ Client details: Name: Elisabeth Ricard Date of birth: 8.10.1975 Address: (2)._____________________ (street), Callington (town) Policy number: CZ8809 Accident details: Date: (3). _________________ Time: Approx. 8.30 pm Supporting evidence: (4). __________________ Medical problems (if any): (5). _______________

Your answers: 1.

2.

3.

4.

5.

Part 2: Listen to a discussion on expectations in today’s job market and choose the correct answer A, B, C, or D which fits best according to what your hear. Write your answers in the corresponding numbered boxes provided. (10pts) (Adopted from Cambridge Objective Proficiency)

1/16


1. How does Diane Webber view “job for life”? A. She regrets the fact that this situation is no longer the norm. B. She feels that many long-serving employees fail to make a useful contribution. C. She believes that people should have challenged their employers’ motives more. D. She wishes the workplace had been more secure in the past. 2. According to Diane, younger workers in today’s workplace A. learn all the skills they need early on. B. accept lateral moves if they are attractive. C. expect to receive benefits right from the start. D. change jobs regularly to achieve a higher level. 3. What does Diane say about continuity in companies? A. It is desirable in both junior and senior management. B. It is impossible to achieve in today’s more competitive environment. C. It is unimportant, due to the greater emphasis on teamwork. D. It is necessary, but only up to a point. 4. According to Diane, what is the actual benefit of higher levels of personnel movement? A. Higher levels of output.

B. Better problem-solving.

C. More creativity.

D. Greater efficiency.

5. Diane considers that nowadays companies are at most risk from A. run-of-the-mill employees who play safe. B. successful high-fliers who quickly move on. C. unreliable staff who lack commitment. D. external advisors who have undue power. Your answers: 1.

2.

3.

4.

5.

Part 3: You will hear part of an interview with Norman Cowley, a well-known novelist and biographer. For questions 1-5, decide whether the statement is TRUE (T) or FALSE (F). Write your answers in the corresponding numbered boxes provided. (10 pts) (Adapted from Cambridge Certificate in Advanced English 6) 2/16


1. Norman Cowley is proud of the directness of the writing of his first novel. 2. According to Norman Cowley, the value of book reviews now is that they motivate less committed writers. 3. Norman Cowley thinks that some modern novels don’t analyze the characters sufficiently. 4. Norman Cowley believes that a novel should mainly give beautiful language to the reader. 5. What Norman Cowley likes about writing a biography is making the subject known to a wider audience. Your answers: 1.

2.

3.

4.

5.

Part 4: You will hear an interview with a man called Daren Howarth, who works as a carbon coach. For questions 1-10, complete the sentences with NO MORE THAN THREE WORDS AND/OR A NUMBER for each answer. Write your answers in the corresponding numbered boxes provided. (20 pts) (Adopted from FCE listening test 6) The Carbon Coach 1. Daren says that a carbon coach works full-time as a (1)____________ with various clients. 2. Before becoming a carbon coach, Daren trained to be an (2)___________ . 3. When assessing a family's carbon footprint, Daren looks first at their (3) __________ . 4. Daren uses what's called a (4) ________ to see how much electricity things use. 5. Daren points out that (5) _______ will help pay for roof insulation. 6. Daren feels that using (6) __________ of the old type is the worst waste of energy he sees. 7. Daren helped to reduce a band's carbon footprint at (7) ________ as well as on its CDs. 8. Daren mentions a new type of green home called an (8) ________ . 9. The new green home uses both the sun and (9) _______ to produce electricity.

3/16


10. Daren suggests buying a (10) ________ which gives more information about the new green home. Your answers: 1.

2.

3.

4.

5.

6.

7.

8.

9.

10.

SECTION B: LEXICO AND GRAMMAR (50 pts) Part 1: Choose the best answer to each of the following questions. Write your answers in the corresponding numbered boxes. (20 pts) (WORD PERFECT + OLYMPIC 30-4+ OXFORD WORD SKILLS) 1. Because of the dominance of retail chain-stores, most shopping centers show the same bland _______ and no imagination. A. similarity B. likeness C. equality D. uniformity 2. She wears the most _______ color combinations you could ever imagine. A. hiding B. hideout C. hidebound D. hideous 3. It would help _______ me, if you could go to the Post Office for me. A. totally B. absolutely C. enormously D. largely 4. The plastic surgery must have cost the ________, but there’s no denying she looks younger. A. world B. planet C. universe D. earth 5. A few of the older campers were sent home after a week as they were ________. A. lenient B. erratic C. unruly D. indulgent 6. When they advertised the job, they were ________ with application. A. dense B. filled C. plentiful D. inundated 7. I suppose he could _______ have reached the summit on his own, but I doubt it. A. conceivably B. credibly C. imaginatively D. believably 8. Be _______ as it may, I am afraid any kind of financial compensation is out of the question. A. it B. that C. this D. which 9. In time-honoured _______, the chairman offered a toast to the shop-floor workers. A. protocol B. way C. fashion D. procedure 10. I had to get through a lot of _______ tape, but I finally got the documents I needed. A. red B. blue C. black D. yellow 11. His new play is not only interesting but also unique. It is really off the beaten ________. A. track B. road C. path D. route 12. He tends to act ________ impulse, especially when he is together with his best friends. A. at B. on C. by D. with 13. The river is ________ with increasing load of household rubbish. A. soaked B. burdened C. saturated D. choked 14. The dispute was eventually resolved by a(n) ________ decision of the arbitrator. A. interested B. uninterested C. disinterested D. uninteresting 4/16


15. Being ________ out of $20,000 by his best friend, he swears not to put trust in anybody. A. done B. made C. cheated D. tricked 16. The project manager ______ animatedly as he spoke about his experiences in the jungle. A. advanced B. looked C. noticed D. gestured 17. He seemed very quiet, but it would be a mistake to ______ his intelligence. A. devalue B. depreciate C. undermine D. minimize 18. “Don’t look so worried! You should take the boss’s remarks with a ______ of salt.” A. teaspoon B. pinch C. grain D. dose 19. The ceremony was one hour late as the organisers hadn’t ______ for such an adverse weather condition. A. expected B. bargained C. calculated D. supposed 20. Why do you object to him being taken on – he’ll be a(n) ______ to the company? A. property B. estate C. asset D. material Your answers: 1. 2.

3.

4.

5.

6.

7.

8.

9.

10.

11.

13.

14.

15.

16.

17.

18.

19.

20.

12.

Part 2: The passage below contains 10 mistakes. Underline the mistakes and write the corrections in the corresponding numbered boxes. (10 pts) (United States History Preparing for the advanced placement examination) Line

1 2 3 4 5 6 7 8 9 10 11 12 13 14 15 16 17 18 19 20

Adolescence always has been and always will be (0). the a very difficult time in life. You are lost where between childhood and adulthood, but still, this is the time in life when you have to break freely from the conformity of your peers to find yourself. Some people argue that it’s more difficult to be young today than it used to be. Is this true, and in that case, why? In modern society teenagers pressure to mature much more quickly than one or two hundred years ago. Today, minors on a very early stage have to make decisions regard education, often bearing upon their future careers. In the past, children were expected to follow in the footsteps of their parents, that is to say, the son was supposed to take in the profession of his father, while the daughter was expected to stay at home to take care of domestic duties such as cooking and cleaning. Furthermore, today it’s much more difficult to find your place in society. As cities grow, crime increases, and the anonymity people experiment grows as well. It becomes more difficult to find and cultivate your own ideals and value. On the other hand, the adolescent of today have great opportunities than ever before. In the past, if your father was a blacksmith or a farmer, in ten years, so you would be. Today, teenagers have the possibility to fulfil in all their dreams and ambitions.

5/16


Your Answers: No Line

Mistake

Correction

1 2 3 4 5 6 7 8 9 10 Part 3: Fill each gap in the following sentences with one of the prepositions or particles. Write your answers in the corresponding numbered boxes. (10 pts) 1. We travelled ________ the 6.45 train, which arrived at 8.30. 2. Parents should never dote _________ their children if they do not want to spoil them. 3. If a rebellion flared _________, the authorities would have to take immediate action. 4. She can rattle _______ the names or all the presidents of the US. 5. It was a difficult concept to grasp, but he soon latched _______. 6. Those naughty boys went on making terrible noise in the park even though they had been ticked ______ by the annoyed constable. 7. At first Tom insisted he was right, but then began to back ______. 8. When the cost was set _______ the benefits, the scheme looked good. 9. In a bitter article, he lashed _______ at his critics. 10. I’m sorry to butt _______, but did you happen to mention the name “Fiona”? Your answers: 1.

2.

3.

4.

5.

6.

7.

8.

9.

10.

Part 4: Write the correct form of each bracketed word in the following passage. Write your answers in the corresponding numbered boxes. (10pts) (CAE Reading and Use of English – Test 13). BLACK WIDOW SPIDER The black widow spider’s (0. NOTORIOUS) is not without foundation. However, an element of exaggeration has led to certain (1. CONCEPT) regarding its evil nature. Firstly, this spider is not as dangerous as is often thought. While it is indeed one of 6/16


the most (2. VENOM) species of spider, its venom being fifteen times stronger than that of the prairie rattlesnake, its bite injects such as amount of venom by (3. COMPARE) that it is unlikely to kill humans. In fact, (4. FATAL) are rare. Black widows bite only if they are touched or their web is threatened. Furthermore, only the adult female is poisonous. Those most at risk from the female are the spider’s natural pray-insects-and male black widow spiders. The latter are vulnerable as the female is (5. SOLITUDE) by nature, and has been known to kill and eat the male after mating. Such (6. OCCUR) are rare, but they explain how the spider got its name – and its reputation. Nevertheless, the (7. PLEASE) effects of this spider’s bite should not be (8. ESTIMATE), and if you live in a temperate climate and have a fireplace in your home, it is advisable to take (9. CAUTION). Black widow spiders often inhabit wood piles, so you should wear gloves when handling firewood. Furthermore, since black widow spiders are (10. RESIST) to many insecticides, you should regularly clean out likely hiding places. Example: 0. NOTORIOUS notoriety

Your answers: 1.

2.

3.

4.

5.

6.

7.

8.

9.

10.

C. READING (50 pts) Part 1: For questions 1-10. Read the following passage and decide which answer A, B, C or D best fits each gap. Write your answers in corresponding numbered boxes. (10 pts) (FCE Test) TWO CREATURES OF THE PAST - MAMMOTHS AND MASTODONS The elephant has some distant relatives called mammoths, which lived in the Stone Age. More than 15,000 years ago people painted pictures of them on cave walls. Then, astonishingly, in 1799, a man walking along the banks of the River Lena in Siberia (1) _______ a starting discovery. Peering into a wall of ice, he could just see the shape of a massive, hairy mammoth, apparently (2) _______ at him. He immediately ran away in terror, but several days later he (3) _______ the courage to return and cut the mammoth’s tusks - its two enormous teeth - out of the ice to sell them. Mammoth tusks are the biggest teeth of any known creature, some (4) _______ a length of five meters. These tusks were (5) _______ to good use, protecting the mammoths’ young from other animals, and brushing away snow. To most of us, mammoths are probably the most (6)_______ of the elephant’s extinct relatives, but thousands of years earlier, in the woodlands of eastern North America, lived another of the elephant’s relatives called the mastodon. Like mammoths, 7/16


mastodon may also have had a hairy coat, but while mammoths mainly ate grass on the plains, mastodons (7) _______ to eat twigs and leaves. We do not know why mastodons became extinct. However, computer studies of the decrease in mammoth (8) _______ suggest that it was a particular (9) _______ of overhunting by humans, and changes in the climate at that time which (10)_______ to their disappearance. 1. A. made 2. A. watching out 3. A. grew 4. A. increasing 5. A. put 6. A. usual 7. A. preferred 8. A. quantities 9. A. combination 10. A. guided Your answers 1. 2.

3.

B. took C. did B. watching over C. looking after B. felt C. found B. reaching C. expanding B. held C. set B. frequent C. regular B. enjoyed C. desired B. amounts C. totals B. addition C. attachment B. caused C. led

4.

5.

6.

7.

D. had D. looking out D. experienced D. completing D. kept D. familiar D. selected D. numbers D. connection D. influenced

8.

9.

10.

Part 2: Read the text below and think of the word which best fits each gap. Use only ONE word in each space. Write your answers in corresponding numbered boxes. (15 pts) (Adapted from https://elearn.edu.vn/LessonExercis) The human race is only one species of (1) _________ in the living world. Many other species exist on this planet. However, human beings have a great influence on the rest of the world. They are changing the environment (2) _________ building cities and villages where forests once stood. They are affecting the water supply by using water for industry and agriculture. They are changing weather conditions by cutting down trees in the forests and are destroying the air by adding (3) ________ to it. It can be said that human beings are changing the environment in all respects through their actions and by their habits. This has (4) _________ in two serious consequences. The (5) _________ is that many kinds of animals are killed. The second is that the environment where these animals are living is being destroyed. As the result, the number of (6) _________ animals is decreasing so rapidly that they are in danger of becoming extinct. In order to make sure that these rare animals do not disappear, (7) _________ have been made to protect endangered nature. Scientists have made lists of these species and suggested ways to save them. Many organizations have been set up and (8) _________ have been raised. Thousands of national parks all over the world have been established to protect endangered animals. Laws have been introduced to prohibit (9) _________ endangered animals and destroying the environment where animals are living. If people’s (10.)_________ with the environment decreases, more species will survive and produce offspring. The earth will be a happy planet where human beings, animals and plants peacefully co-exist. 8/16


Your answers: 1.

2.

3.

4.

5.

6.

7.

8.

9.

10.

Part 3: Read the following passage and choose the best answer to each question. Write your answers in the corresponding numbered boxes provided below the passage (10 pts) (Adopted from TOEFL- Reading 6) Anthropology distinguishes itself from the other social sciences by its greater emphasis on fieldwork as the source of new knowledge. The aim of such studies is to develop as intimate an understanding as possible of the phenomena investigated. Although the length of field studies varies from a few weeks to years, it is generally agreed that anthropologists should stay in the field long enough for their presence to be considered ‘natural’ by the permanent residents. Realistically, however, anthropologists may never reach this status. Their foreign mannerisms make them appear clownish, and so they are treated with curiosity and amusement. If they speak the local language at all, they do so with a strange accent and flawed grammar. They ask tactless questions and inadvertently break rules regarding how things are usually done. Arguably this could be an interesting starting point for research, though it is rarely exploited. Otherwise, anthropologists take on the role of the ‘superior expert’, in which case they are treated with deference and respect, only coming into contact with the most high-ranking members of the society. Anthropologists with this role may never witness the gamut of practices which take place in all levels of the society. No matter which role one takes on, anthropologists generally find fieldwork extremely demanding. Anthropological texts may read like an exciting journey of exploration, but rarely is this so. Long periods of time spent in the field are generally characterised by boredom, illness and frustration. Anthropologists in the field encounter unfamiliar climates, strange food and low standards of hygiene. It is often particularly trying for researchers with middle-class, European backgrounds to adapt to societies where being alone is considered pitiful. It takes a dedicated individual to conduct research which is not in some way influenced by these personal discomforts. Nonetheless, fieldwork requires the researcher to spend as much time as possible in local life. A range of research methodologies can be utilised to extract information. (1) These can be classified as emic or etic. (2) While emic descriptions are considered more desirable nowadays, they are difficult to attain, even if the researcher does his utmost to reproduce the facts from the natives’ point of view. (3) More often than not, aspects of the researcher’s own culture, perspective and literary style seep into the narrative. Moreover, research generally involves translations from one language to another and from speech into writing. In doing this, the meaning of utterances is changed. (4) The only truly emic descriptions can be those given by the natives themselves in their own vernacular. The least invasive type of research methodology is observation. Here, the researcher studies the group and records findings without intruding too much on their privacy. This is not to say, however, that the presence of the researcher will have minimal impact on the findings. An example was Richard Borshay Lee, who, in studying local 9/16


groups in the Kalahari refused to provide the people with food so as not to taint his research, leading to an inevitable hostility towards the researcher which would not otherwise have been present. A variant on the observation technique, participant observation requires that the anthropologist not only observes the culture, but participates in it too. It allows for deeper immersion into the culture studied, hence a deeper understanding of it. By developing a deeper rapport with the people of the culture, it is hoped they will open up and divulge more about their culture and way of life than can simply be observed. Participant observation is still an imperfect methodology, however, since populations may adjust their behavior around the researcher, knowing that they are the subject of research. The participatory approach was conceived in an attempt to produce as emic a perspective as possible. The process involves not just the gathering of information from local people, but involves them in the interpretation of the findings. That is, rather than the researcher getting actively involved in the processes within the local community, the process is turned on its head. The local community is actively involved in the research process. 1. The main premise of the text is______. A. the steps to be followed when undertaking anthropological fieldwork B. a history of anthropological fieldwork methodology C. the effects that an anthropological fieldwork has on local communities D. the problems with conducting anthropological fieldwork 2. The main reason for anthropological researchers remaining in a community for an extended period of time is that ______. A. they can gather as much information as possible B. they can try out a range of different research methodologies C. they want local people to behave naturally around them. D. they need time to become accustomed to the conditions 3. What does the passage say about researchers who are considered a ‘clown’ by locals? A. They do culturally unacceptable things without realizing it. B. They do not gain respect among high-ranking members of the community. C. They cannot conduct any research of value. D. They do not study the language and culture of the region before their arrival. 4. What does ‘gamut’ mean? A. idea or impression B. prohibition or taboo C. range or extent D. secret or mystery 5. The writer believes that the most difficult aspect of fieldwork for educated westerners is ______. A. the lack of companionship B. poor sanitary conditions C. failure to meet expectations D. never being left alone 6. In paragraph 3, it is implied that ______. A. the fieldworker’s emotions and mood prejudice the research. B. the longer a researcher spends in the field, the more depressed he gets. C. middle-class Europeans find field research more difficult than researchers from other backgrounds.

10/16


D. anthropological texts tend to exaggerate the difficult conditions that researchers experience. 7. Where in paragraph 4 does this sentence belong? A native’s point of view of his own lifestyle is emic, while the analytical perspective of the outsider is etic. A. (1) B. (2) C. (3) D. (4) 8. Why is the example of Richard Borshay Lee given in paragraph 5? A. to demonstrate that observation is an ineffective method of gathering data. B. to highlight why it is important that researchers minimize their impact on a community. C. to show the dangers of researchers trying to lessen their impact on a community D. to show how a researcher’s choice of methodology can influence the validity of his findings. 9. How does participant observation differ vary from straightforward observation? A. It requires the researcher to become actively involved in the daily lives of those being studied. B. It allows the subjects of the research a greater degree of privacy. C. It eradicates the problem of research subjects altering their behaviour towards researchers. D. It takes longer to perform this type of research effectively. 10. Which of the following is NOT true of the participatory approach? A. It attempts to reduce etic accounts of a culture to a minimum. B. It does not require a researcher to be present. C. It aims to involve the subjects in both information gathering and analysis. D. It is the reverse of the participant observation technique. Your answers 1. 2.

3.

4.

5.

6.

7.

8.

9.

10.

Part 4: Read the passage and do the tasks. Write your answers in the corresponding numbered boxes provided. (15pts) (Adapted from Ielts reading actual tests 2018) Questions 1-5 The reading passage has five paragraphs, A-E. Choose the correct heading for paragraphs A-E from the list below. List of Headings i

The definition of three laws

ii

Quoting three laws against the homeopathy

iii

There are many methods of avoiding answering ambiguous questions.

iv

The purpose of illustrating the symptoms of homeopathy

v

The constant booming of homeopathy

vi

Some differences between homeopathy and placebo

vii

Placebo is probably better than homeopathy

viii

An example of further demonstrating the negative effect of homeopathy 11/16


ix

The purpose of staging a demonstration against homeopathy

Your answers 1. Paragraph A: _____

2. Paragraph B: _____

4. Paragraph D: _____

5. Paragraph E: _____

3. Paragraph C: _____

Homeopathy A. An international protest against homeopathy this week aims to demonstrate the truth about homeopathy - that there is literally nothing in it, says Martin Robbins at 10.23 am on 30 January, and more than 300 activists in the UK, Canada, Australia and the US will take part in a mass homeopathic "overdose". Skeptics will publicly swallow an entire bottle of homeopathic pills to demonstrate to the public that homeopathic remedies, the product of a scientifically unfounded 18th-century ritual, are simply sugar pills. Many of the skeptics will swallow 84 pills of arsenicum album, a homeopathic remedy based on arsenic which is used to treat a range of symptoms, including food poisoning and insomnia. The aim of the "10:23" campaign, led by the Merseyside Skeptics Society, based in Liverpool, UK, is to raise public awareness of just exactly what homeopathy is, and to put pressure on the UK's leading pharmacist, Boots, to remove the remedies from sale. The campaign is called 10:23 in honor of the Avogadro constant (approximately 6 x 1023, the number of atoms or molecules in one mole of a substance). B. That such a protest is even necessary in 2010 and is remarkable, but somehow the homeopathic industry has not only survived into the 21st century, but prospered. In the UK alone more than ÂŁ40 million is spent annually on homeopathic treatments, with ÂŁ4 million of this being sucked from the National Health Service budget. Yet the basis for homeopathy defies the laws of physics, and high-quality clinical trials have never been able to demonstrate that it works beyond the placebo effect. C. The discipline is based on three "laws"; the law of similars, the law of infinitesimals and the law of succession. The law of similars states that something which causes your symptoms will cure your symptoms, so that, for example, as caffeine keeps you awake, it can also be a cure for insomnia. Of course, that makes little sense, since drinking caffeine, well, keeps you awake. Next is the law of infinitesimals, which claims that diluting a substance makes it more potent. Homeopaths start by diluting one volume of their remedy - arsenic oxide, in the case of arsenicum album - in 99 volumes of distilled water or alcohol to create a "centesimal". They then dilute one volume of the centesimal in 99 volumes of water or alcohol, and so on, up to 30 times. Application of Avogadro's constant tells you that a dose of such a "30C" recipe is vanishingly unlikely to contain even a single molecule of the active ingredient. The third pillar of homeopathy is the law of succession. This states - and I'm not making this up - that by tapping the liquid in a special way during the dilution process, a memory of the active ingredient is somehow imprinted on it. This explains how water is able to carry a memory of arsenic oxide, but apparently not of the contents of your local sewer network. D. The final preparation is generally dropped onto a sugar pill which the patient swallows. Homeopaths claim that the application of these three laws results in a remedy that, even though it contains not a single molecule of the original ingredient, somehow 12/16


carries an "energy signature" of it that nobody can measure or detect. Unsurprisingly, when tested under rigorous scientific conditions, in randomized, controlled and doubleblind trials, homeopathic remedies have consistently been shown to be no better than a placebo. Of course, the placebo effect is quite powerful, but it's a bit like justifying building a car without any wheels on the basis that you can still enjoy the comfy leather seats and play with the gear shift. E. Even some retailers who sell the treatments have admitted there is no evidence that they work. In November, Paul Bennett, the superintendent pharmacist at Boots, appeared before the UK parliament's Commons Science and Technology Committee's "evidence check" on homeopathy. He was questioned by Member of Parliament Phil Willis, who asked: "Do they work beyond the placebo effect?" "I have no evidence before me to suggest that they are efficacious," Bennett replied. He defended Boots' decision to sell homeopathic remedies on the grounds of consumer choice. "A large number of our consumers actually do believe they are efficacious, but they are licensed medicinal products and, therefore, we believe it is right to make them available," he said. Questions 6-10 Do the following statements agree with the information given in the reading passage? In boxes 6-10 on your answer sheet, write YES

if the statement agrees with the information given in the passage

NO

if the statement contradicts the information given in the passage

NOT GIVEN

if there is no information about this in the passage

6. Skeptics are planning to hold a demonstration in "10.23" campaign to protest against UK's leading pharmacist, Boots. 7. National Health Service budget gained a small portion of homeopathic industry. 8. The example of Caffeine is to present that homeopathy resists the laws of similars. 9. Instilling the idea to people that homeopathy is equal to modern medicine poses danger. 10. Paul Bennett claimed effectiveness of taking the homeopathic medicine is proved. Your answers 6.

7.

8.

9.

10.

D. WRITING (50 pts) Part 1: Rewrite the following sentences with the given words in such a way that the second sentence has the same meaning as the first one. Do not change the form of the words in brackets. (10 pts) 1. He would do almost anything to win the girl's hand. (LENGTHS) He ……………………………………..………………...……… to win the girl's hand. 13/16


2. The crash victim was beyond help when emergency services reach her. (WHATSOEVER) There …………………….…………..…………………..… on reaching the crash victim. 3. His fake arrogance only hid his genuine insecurity. (LAY) Behind …….…………………………………………………………… insecurity. 4. They will try Abrams for murder at the High Court next week. (TRIAL) …………...……………………………………………………………….. 5. A great many people will congratulate her if she wins. (SHOWERED) ……………………………………………………………………………

Part 2: Finish the second sentence in such a way that it means the same as the sentence printed before. (10 pts) 1. Because I believed his lies, he got a lot of money from me. He conned............................................................................................................................ 2. It isn’t possible that Jane was absent from work today. Jane must ............................................................................................................................. 3. It doesn’t matter which chemical you put into the mixture first. The result will be the same. It makes ................................................................................................................................. 4. Just thinking about this face at the moment makes me laugh. The very.................................................................................................................................. 5. To pass the time, I looked through some magazines. I whiled ................................................................................................................................. Part 3: ESSAY WRITING (30 pts) Gaming has become a popular e-sport and has been included in some regional and international sports events. Some people advocate this, claiming that there is almost no risk of injury with e-sports; others are worried that the increasing popularity of competitive gaming tournaments may exacerbate video game addiction among young people. Write an essay of about 250 words to present your opinion on this issue. Give reasons and specific examples to support your position. 14/16


……………………………………………………………………………………………… ……………………………………………………………………………………………… ……………………………………………………………………………………………… ……………………………………………………………………………………………… ……………………………………………………………………………………………… ……………………………………………………………………………………………… ……………………………………………………………………………………………… ……………………………………………………………………………………………… ……………………………………………………………………………………………… ……………………………………………………………………………………………… ……………………………………………………………………………………………… ……………………………………………………………………………………………… ……………………………………………………………………………………………… ……………………………………………………………………………………………… ……………………………………………………………………………………………… ……………………………………………………………………………………………… ……………………………………………………………………………………………… ……………………………………………………………………………………………… ……………………………………………………………………………………………… ……………………………………………………………………………………………… ……………………………………………………………………………………………… ……………………………………………………………………………………………… ……………………………………………………………………………………………… ……………………………………………………………………………………………… ……………………………………………………………………………………………… ……………………………………………………………………………………………… ……………………………………………………………………………………………… ……………………………………………………………………………………………… ……………………………………………………………………………………………… ……………………………………………………………………………………………… ……………………………………………………………………………………………… ……………………………………………………………………………………………… 15/16


……………………………………………………………………………………………… ……………………………………………………………………………………………… ……………………………………………………………………………………………… ………………………………………………………………………………………… ……………………………………………………………………………………………… ……………………………………………………………………………………………… ……………………………………………………………………………………………… ……………………………………………………………………………………………… ……………………………………………………………………………………………… ……………………………………………………………………………………………… ……………………………………………………………………………………………… ……………………………………………………………………………………………… ……………………………………………………………………………………………… ……………………………………………………………………………………………… ……………………………………………………………………………………………… ……………………………………………………………………………………………… ……………………………………………………………………………………………… ……………………………………………………………………………………………… ……………………………………………………………………………………………… ……………………………………………………………………………………………… ……………………………………………………………………………………………… ……………………………………………………………………………………………… ……………………………………………………………………………………………… ……………………………………………………………………………………………… ……………………………………………………………………………………………… ……………………………………………………………………………………………… ……………………………………………………………………………………………… ……………………………………………………………………………………………… ……………………………………………………………………………………………… Nguyễn Thị Nguyệt (0915417566) Nguyễn Thị Yến (0906285895)

16/16


ĐỀ THI HỌC SINH GIỎI DHBB NĂM HỌC 2018 - 2019 Mônthi: TiếngAnh 10

SỞ GD&ĐT BẮC NINH TRƯỜNG THPT CHUYÊN BN ĐÁP ÁN ĐỀ XUẤT

SECTION A: LISTENING (50 points) Part 1: Listen to a woman calling an insurance company to report an accident. For questions 1-5, write NO MORE THAN THREE WORDS AND/OR A NUMBER for each answer. Write your answers in the corresponding numbered boxes provided. (10pts) (Adapted fromhttps://www.ielts-exam.net/practice_test) 1.Motor

2. 60 Forest

3. 12th

4. police

5.

Insurance

Road

September

report

minor injuries

Part 2: Listen to a discussion on expectations in today’s job market and choose the correct answer A, B, C, or D which fits best according to what your hear. Write your answers in the corresponding numbered boxes provided. (10pts)(Adopted from Cambridge Objective Proficiency) 1. B 2. B 3. D 4. C 5. A

Part 3: You will hear part of an interview with Norman Cowley, a well-known novelist and biographer.For questions 1-5, decide whether the statement is TRUE (T) or FALSE (F). Write your answers in the corresponding numbered boxes provided. (10 pts) (Adapted from Cambridge Certificate in Advanced English 6) 1. T 2. F 3. T 4. F 5. F

Part 4: You will hear an interview with a man called Daren Howarth, who works as a carbon coach. For questions 1-10, complete the sentences with NO MORE THAN THREE WORDS AND/OR A NUMBER for each answer. Write your answers in the corresponding numbered boxes provided. (20 pts) (Adopted from FCE listening test 6) 1. consultant

6. light bulbs

2. ecologist

7. its/their concerts

3. bills

8. Earthship

4. carbon

5. the

meter

government

9. the wind

10. handbook

power

1/4


SECTION B: LEXICO AND GRAMMAR (50 pts) Part 1:Choose the best answer to each of the following questions. Write your answers in the corresponding numbered boxes. (20 pts) (WORD PERFECT + OLYMPIC 30-4+ OXFORD WORD SKILLS) 1. D 2. D 3. C 4. D 5. C 6. D 7. A 8. B 9. C 10. A 11. A 12. B 13. D 14. C 15. A 16. D 17. B 18. B 19. B 20. C Part 2: The passage below contains 10 mistakes. Underline the mistakes and write the corrections in the corresponding numbered boxes. (10 pts) (United States History -Preparing for the advanced placement examination) Line 1 2 3 6 8 10 14 16 17 19 19

Mistake Correction 0. the a 1. where → somewhere 2. freely → free 3. pressure → are pressured 4. regard → regarding 5. take in → take up 6. experiment → experience 7. value → values 8. great → greater 9. so you would → so would you 10. fulfil in all → fulfil(l) all

Part 3: Fill each gap in the following sentences with one of the prepositions or particles. Write your answers in the corresponding numbered boxes (10 pts) 1. on 2. on 3. up 4. off 5. on 6. off 7. down 8. against 9. out 10. in Part 4: Write the correct form of each bracketed word in the following passage. Write your answers in the corresponding numbered boxes. (10pts) (CAE Reading and Use of English – Test 13). 1. misconceptions

2. venomous

3. comparison

4. fatalities

5. solitary

6. occurrences

7. unpleasant

8. 9. precautions 10. resistant underestimated

C. READING (50 pts) Part 1: For questions 1-10. Read the following passage and decide which answer A, B, C or D best fits each gap. Write your answers in corresponding numbered boxes. (10 pts)(FCE Test) 1. A

2. D

3. C

4. B

5. A 2/4


6. D

7. A

8. D

9. A

10. C

Part 2: Read the text below and think of the word which best fits each gap. Use only ONE word in each space. Write your answers in corresponding numbered boxes. (15pts) (Adapted from https://elearn.edu.vn/LessonExercis) 1. beings

2. by

3. pollutants

4. resulted

5.first

6. rare

7. efforts

8. funds

9. killing

10. interference

Part 3: Read the following passage and choose the best answer to each question. Write your answers in the corresponding numbered boxes provided below the passage (10 pts) (Adopted from TOEFL- Reading 6) 1. D

2. C

3. A

4. C

5. D

6. A

7. B

8. C

9. A

10. B

Part 4: Read the passage and do the tasks.Write your answers in the corresponding numbered boxes provided. (15pts) (Adapted from Ielts reading actual tests 2018) 1. ix

2. v

3. i

4. vii

5. iv

6. YES

7. NO

8. YES

9. NOT 10. GIVEN NO

D. WRITING (50 pts) Part 1: Rewrite the following sentences with the given words in such a way that the second sentence has the same meaning as the first one. Do not change the form of the words in brackets. (10 pts) 1. He would go to any/ great lengths to win the girl's hand. 2. There was nothing whatsoever emergency services could do on reaching the crash victim. 3. Behind his fake arrogance lay (a) hidden/ his genuine insecurity. 4. Abrams will stand trial for murder at the High Court next week. 5. She will be showered with congratulations if she wins. /She will have congratulations showered on her if she wins. Part 2: Finish the second sentence in such a way that it means the same as the sentence printed before. (10pts) 1. He conned me out of a lot of money. 2. Jane must have been at work today. 3. It makes no difference which chemical you put into the mixture first. 4. The very thought of his face at the moment makes me laugh. 5. I whiled away the time by looking through some magazines. 3/4


Part 3: Essay writing (30 pts) The mark given to part 3 is based on the following criteria: 1. Task achievement (10 points) • ALL requirements of the task are sufficiently addressed • Ideas are adequately supported and elaborated with relevant and reliable explanations, examples, evidence, personal experience, etc. 2. Organization (10 points) a. Ideas are well organized and presented with coherence, cohesion and unity b. The essay is well-structured: • Introduction: is presented with clear thesis statement • Body paragraphs are written with unity, coherence, and cohesion. Each body paragraph must have a topic sentence and supporting details and examples when necessary. • Conclusion summarizes the main points and offers personal opinions (prediction, recommendation, consideration, ...) on the issue. 3. Language use (5 points) a. Demonstration of a variety of topic-related vocabulary b. Excellent use and control of grammatical structures 4. Punctuation, spelling and handwriting (5 points) a. Correct punctuation and no spelling mistakes b. Legible handwriting GV rađề: NguyễnThịNguyệt (0915417566) NguyễnThịYến (0906285895)

4/4


ĐỀ THI ĐỀ XUẤT DHBB MÔN TIẾNG ANH KHỐI 10 TỈNH: PHÚ THỌ PARTI. LISTENING Section 1. Listen and complete the following table. Write NO MORE THAN THREE WORDS AND/OR A NUMBER for each answer. (10 points) Notes: Clark’s Bike Hire Example

Answer

Type: Touring bike Rental

£ 50 a week, or (1) £ ________a day

Late return fee

(2) £ _________

Deposit

£ 60

Accessories

£ 5 for (3)_______________: pannier or handlebar type free: pump (4)__________ strong lock

Insurance:

included, but must pay first £ 100 of claim

Pay:

by (5)_______________only Source: IELTS practice tests by Peter May

Section 2. Listen and choose the correct answer to each question. Write your answer (A, B, or C) in the space provided. (10 points) 1. These sessions with a counselor are _____. A. compulsory for all students B. available to any students C. for science students only 2. The counselor says that new students have to _____. A. spend more time on the college premises


B. get used to working independently C. work harder than they did at school 3. John complains that the resource center _____. A. has limited opening hours B. has too few resources C. gets too crowded 4. The counselor suggests to John that _____. A. most other students can cope B. he needs to study all the time C. he should be able to fit in some leisure activities 5. Before being able to help John, the counselor needs to _____. A. talk with some of his lectures B. consult his tutor C. get more information from him Your answers: 1.

2.

3.

4.

5.

Section 3. Listen to the talk about women in the workplace and answer the following questions (10 points) 1. How is the situation for women in the workplace changing? ………………………………………………………………………………………….. 2. In which management role that women constitute 17% of the staff? ………………………………………………………………………………………….. 3. How many percent of employees think that gender equality is a priority? ………………………………………………………………………………………….. 4. What is the action companies should take to understand the problem of gender inequality? …………………………………………………………………………………………..


5. What should companies do to make sure opportunities and advancement are equitable? ………………………………………………………………………………………….. Section 4. Complete the table below, using NO MORE THAN THREE WORDS AND/OR A NUMBER for each answer. (10 points) Apartments

Facilities

Other information

Cost

Rose Garden

Studio flat

Example:

£219

Apartments

Entertainment programme: Greek dancing

BlueBay

large salt-water

- just (1) ………… metres

Apartments

swimming pool

from beach

£275

- near shop (2) ………………

terrace

watersports

£490

Apartments The Grand

- Greek paintings - overlooking (4)

(5 ) £

- (3) …………..

………………

………….. - near a supermarket and a disco

Your answers:

1. ……………

2. ……………

3. ……………

4. ……………

5. ……………

PART II. LEXICO-GRAMMAR Section 1. Choose the best answer (A, B, C or D) to complete each sentence below. (20 points) 1. My mother _____ when she found out that I’d forgotten to do the washing-up again. A. stood her ground

B. hit the ceiling

C. felt off color

D. made my blood boil


2. Only during the early 20th century _____ in the United State. A. liquor was prohibited then

B. that liquor was prohibited

C. was liquor prohibited

D. when liquor was prohibited

3. _____ a scholarship, I entered the frightening and unknown territory of private education. A. to award

B. to be awarded

C. having awarded

D. having been awarded

4. Jane was full of _____ towards her stepmother. A. retribution

B. resentment

C. reprisal

D. vengeance

5. I caught the last bus by the skin of my _____. A. teeth

B. leg

C. neck

D. mouth

6. It is the _____ of stupidity to go walking in the mountains in this weather. A. height

B. depth

C. source

D. matter

7. In my view, the changes to the education system have been to good _____. A. influence

B. outcome

C. upshot

D. effect

8. Poor management brought the company to the _____ of collapse. A. brink

B. rim

C. fringe

D. brim

9. The plumber agreed that he had made a mistake and promised to put it _____ the next day. A. correct

B. proper

C. sound

D. right

10. I can’t understand why you have to make such a _____ about something so unimportant. A. mess

B. stir

C. fuss

D. bother

11. The judge looked at the _____ and reminded him that he had to tell the whole truth. A. bystander

B. witness

C. onlooker

D. viewer

12. I was disappointed that the restaurant had _____ flowers on the table. A. false

B. untrue

C. artificial

D. forged


13. As she is so heavily overlooked, there is a _____ possibility that she will have a nervous breakdown. A. distinctive

B. distinct

C. little

D. manifest

14. _____ to the invention of the steam engine, most forms of transport were horsedrawn. A. Akin

B. Prior

C. In addition

D. With reference

15. The truant was _____ from school for unbecoming behaviour. A. dispelled

B. repelled

C. expelled

D. compelled

16. Of course he can lift that! He is as ______ as a horse. A. tough

B. big

C. heavy

D. strong

17. Fearing for his life, he _____ the mugger for mercy. A. pleaded

B. petitioned

C. urged

D. begged

18. I am not convinced that financial advisers always act in their clients’ best _____. A. advantage

B. interest

C. intention

D. result

19. The case against the bank robbers was _____ for lack of evidence. A. discarded

B. dismissed

C. refused

D. eliminated

20. Five readers _____ the correct solution to our recent competition. A. communicated

B. qualified

C. submitted

D. subscribed

Section 2. There are 10 errors in this passage. Identify them by underlining the words that need to be corrected and correct them. (10 points) The New York Times is a daily newspaper published in the New York city. For a long time, it has been the newspaper on record in the US and one of the world’s great newspapers. Its strength is in its editorial excellence, it never has been the greatest newspaper in the term of circulation. The Times was established in 1851 as a penny paper whose editors wanted to report the news in a restrained and objective fashion. It enjoyed early success as its editors made a pattern for the future by appealing to a cultural, intellectual readership instead of a mass audience. Therefore, in the late 19th century, it came into competition


with a more popular, colorful, if not lurid, newspapers in New York. Their publications ran sensational stories, not because they were true, but because they sold newspapers. Despite prices increases, The Times was losing £ 1,000 a week when Adolph Simon Ochs bought it in 1896. Ochs built The Times into an international respected daily. He hired Carr Van Anda as editor. Van Anda placed greatest stress than ever on full reporting of the news of the day, and his emphasized existing good coverage of international news. The management of the paper decided to eliminate fiction of the paper, added a Sunday magazine section, and reduced the paper’s price back to a penny. In April 1912, the paper ran many risks to report every aspects of the sinking of the Titanic. This greatly enhanced its prestige, and in its coverage of the two world wars, The Times continued to enhance its reputation for excellence in world news. Section 3. Complete each of the following sentences with a suitable preposition or particle. (10 points) 1. The new regulations have thrown _______ a few problems for the company. 2. The workers were rather cynical after the meeting. Most of them were _______ no illusions that the management would take their complaints seriously. 3. Jack took early retirement as he was losing his grip _______ the job. 4. He scolded her so much that she was reduced _______ tears by the end of the meeting. 5. The sudden movement of the train threw me _______ balance and I fell head-first down the steps. 6. The price of shares in the company went up _______ over 50% when they announced the discovery of the new oilfield. 7. Ron usually primes himself _______ plenty of black coffee before starting the night shift. 8. We still haven’t sold the house because the buyers went back on their word and pulled _______ the deal.


9. Hi Brenda, Peter here. I’m just ringing _______ to say I’ve got two tickets for the concert on Saturday night. Fancy coming? 10.

The group rounded _______ the concert with their greatest hits. So everyone went

home happy. Section 4. Supply the correct form of the word given in brackets. (10 points) WOMEN ONLY Increasingly, women are taking their holidays without men. For _____ (1. SAFE) reasons, camaraderie or just plain fun, a growing number of female tourists are singing up for women-only trips. Twenty years ago only a _____ (2. HAND) of companies offered such holidays; now there are several hundred. Travel _____ (3. CONSULT) Andre Littlewood says that the combination of higher incomes with delayed marriage, divorce, retirement and widowhood has _____ (4. ABLE) more women to travel, often on their own. They are attracted by the sense of _____ (5. FREE) that a holiday without men affords them. “Women in a group tend to feel _____ (6. INHIBIT) and speak more openly than when men are around”, she adds. “Even on energy-sapping adventure holidays the atmosphere is relaxed and _____ (7. CO-OPERATE). It’s also a great deal more fun. Women laugh more _____ (8. READY) than men, probably because they don’t mind laughing at themselves.” Since her divorce Janice Cummings has been a regular traveler with Everywoman Tours, and Oxford-based Company whose very name is a _____ (9. DETER) to men. “And a good thing too,” she says. “Men simply cannot resist the _____ (10. TEMPT) to try and take control, no matter where they are. And that includes on holiday. Thankfully, there is none of that with Everywoman.” Write your answers here: 1.

2.

3.

4.

5.

____________

____________

____________

____________

____________

6.

7.

8.

9.

10.


____________

___________

____________

____________

___________

PART III: READING Section 1. Read the passage and choose the option that best completes each blank. (10points) The Video Loggers One rather unlikely word that has recently entered the language is “blog”, a shortened form of “web log”. A blog is a diary postedon the Internet by the person writing it – the “blogger” – who presumably expectsother people to read it. It is ironical that modern technology is being used to (1) ________ new life into such an old-fashioned form as the personal journal. And now, as the technology behind video cameras is making them easier to use, we have the video log or “vlog”. Vlogging does not require (2)________ sophisticated equipment: a digital video camera, a high speed Internet connectionand a host are all that is needed. Vloggers can put anything that (3) ________ their fancy onto their personal website. Some vloggers have no ambitions (4) ________ than to show films they have shot while on holiday in exotic places. However, vloggers can also (5) ________ more ambitious purposes. Forinstance, amateur film-makers who want to make a (6) ________ for themselves might publish their work onto the Internet, eager to receive advice or criticism. And, increasingly, vlogs are being used to (7) ________ political and social issues (8) ________ are not newsworthy enough to warrant coverage by mass media. It is still too early to predict (9) ________ vlogging will ever take off in a major way or if it is just a passing fad, but its (10) ________ is only now becoming apparent.

1. A. add

B. put

C. insert

D. breathe

2. A. absolutely

B. largely

C. mostly

D. highly

3. A. grasps

B. appeals

C. takes

D. gives

4. A. except

B. apart

C. rather

D. other


5. A. serve

B. employ

C. function

D. play

6. A. publicity

B. fame

C. name

D. promotion

7. A. emphasize

B. publicize

C. distribute

D. circulate

8. A. who

B. whose

C. which

D. why

9. A. that

B. whether

C. why

D. since

10. A. potential

B. possibility

C. ability

D. feasibility

Section 2. Cloze Test: Read the text below and fill each gap with ONE suitable word. (15 points) Speed limit A recent proposal to limit the speed cars can reach is proving controversial. The idea, put (1) ________ by the institute for Road Safety, involves (2) ___________ vehicles with a communications box containing a digital map of the road network. When a car is in motion, the communications box – which knows how fast the vehicle is(3)_________ permitted to travel on a particular section of road – automatically regulates the car’s speed. It (4)__________ becomes impossible for a vehicle fitted with such a (5)_______ to exceed the speed limit. There are plans to (6)_________ motorists to drive into the centre of major cities, and a communication box could be used to identify vehicles that enter this zone. At present, each time a vehicle does so, its number(7)___________ is photographed. Then the number is compared (8) __________ a list of authorized vehicles, but this system is costly and (9)___________ from foolproof The system could be put in place quickly if the government wished as part the technology has already been developed for another purpose. Electronic vehicle identification is (10) __________ with some people, who regard it as an infringement of their rights. They would certainly object to a plan to restrict the limit of their cars, which may make the government hesitate to go through with it.


Section 3. Read the following passages and choose the best answer to each question. Write your answers in the space provided. (10 points) There is a natural balance to all things: life and death, good and bad, happiness and sadness, pleasure and pain. The very idea that you should focus on positives and ignore negatives, likewise with strengths versus weakness, is not only delusional; it’s a recipe for disaster. Let me tell you a couple of stories to show you how dealing with reality as openly and genuinely as possible is the path to success and happiness, while focusing only on the positives and strengths can destroy your career and your company. It’s no secret that Steve jobs was forced out of Apple in 1984 because his management style had become toxic to the company. Much later, Jobs would come to realize that getting fired from the company he cofounded “was the best thing that could have happened” to him. He called it “awful-tasting medicine” that “the patient needed.” He also likened it to life hitting him in the head with a brick. That forced Jobs to look in the mirror and see the truth – that he wasn’t as capable or as strong a leader as he could be. And as he addressed the issues that stood in his way, the result was the founding of NeXT and Pixar, his eventual return to Apple, and the greatest turnaround in corporate history that built the most valuable company on earth. It’s easy to miss the obvious connection staring us right in the face, that it wasn’t just Apple that had it a wall, fallen on hard times, and found itself in need of a turnaround.The same was true of Jobs. And there was an undeniable connection between the two. It’s also easy to miss the insightfulness of Jobs’ realization that none of his later achievements would have occurred if he hadn’t faced reality. That sort of introspection only comes from someone who’s had some sort of intervention and gone through gutwrenching change as a result.


Not to compare myself with Jobs, but the truth is I’ve gotten a couple of those bricks to the head myself. I’ve been fired more than once and lost my wife early in our marriage. But in every case I looked in the mirror, faced what I saw, made some changes, and bounced back stronger than ever. If I’d just tried to stay positive, focused on my strengths, and searched for the silver lining in the clouds, I never would have figured out what was wrong and become a better person, a better husband, and a better leader. I never would have achieved so much in my career or won my wife back. While life is full of ups and downs, one thing is certain: if you attempt to filter your consciousness and disallow negative thoughts or make believe the weaknesses holding you back don’t exist, you’ll never get past those hurdles and get to the next stage in your personal and professional development. And neither will your business.

1. What could be the best title for the passage? A. Being Successful: Ignore the negatives B. To Be Successful? Quit Being So Positive C. Being Successful: Facing with Disasters D. To Be Successful? Stop Being So Negative 2. The writer uses the phrase “recipe for disaster”in paragraph 1 to stress that ________. A. focusing on positives can make the worse become the worst B. ignoring the negatives can be the key to dealing with any problem C. focusing on positives and ignoring negatives destroy the reality D. ignoring negatives goes against the natural balance to all things 3. The word “likened” in paragraph 3 is closest in meaning to ________. A. linked

B. associated

C. compared

D. related

4. The writer uses the phrase “the two” in paragraph 5 to refer to ________. A. strengths and weaknesses

B. life and death


C. hard times and turnabouts

D. happiness and sadness

5. The writer uses the “bricks” (as Steve Jobs did) in paragraph 6 to describe ________. A. weaknesses he had at the times of being fired B. hardships he faced with during his professional life C. changes he led in order to make turnabouts D. interventions he made in his own introspection 6. Which of the following statements is NOT true about Steve Jobs according to the passage? A. He got fired from Apple because of his poor management style. B. He founded NeXT and Pixar to be able to return to Apple. C. He made Apple the most valuable company in the world. D. He considered being fired from Apple the best lesson learnt. 7. What is true about the story teller when facing failures according to the passage? A. He tried to stay positive to bounce back stronger. B. He focused on his strengths as the motivation for turnabouts. C. He looked into the way Steve Jobs did to find his own solution. D. He examined the situation, making necessary changes to be better. 8. It can be inferred from the passage that ________. A. Only when you admit the negative as part of your life, you can proceed. B. Positives and negatives can never go along with each other in reality. C. It’s our weaknesses that save us in hardships and calamities. D. To earn happiness, one has to experience the feeling of sadness. 9. The phrase “filter your consciousness” in the last paragraph mostly means ________. A. release negative feelings from one’s heart B. clear one’s mind from worries C. push negative thoughts out of one’s mind


D. stay away from possible dangers 10. The tone of the passage is ________. A. informative

B. preventive

C. persuasive

D. argumentative

Write your answers here: 1.

2.

3.

4.

5.

____________

____________

____________

____________

____________

6.

7.

8.

9.

10.

____________

____________

____________

____________

___________

Section 4. Read the passage carefully and do the tasks.(15 points) The passage has six paragraphs, A-F. Choose the correct heading for paragraphs B and D-F from the list of headings below. There are two headings you do not need. List of Headings

Example: Paragraph A : vii

I

Effects of irrigation on sedimentation

ii

The danger of flooding the Cairo area

iii

Causing pollution in the Mediterranean

iv

Interrupting a natural process

v

The threat to food production

vi

Less valuable sediment than before

vii

Egypt's disappearing coastline

viii

Looking at the long-term impact


1. Part B:

…………….

2. Part C:

…………….

3. Part D:

…………….

4. Part E:

…………….

5. Part F:

……………. Alarming Environmental Problem of a Delta

A. The fertile land of the Nile delta is being eroded along Egypt's Mediterranean coast at an astounding rate. Formerly, land scoured away from the coastline by the currents of the Mediterranean Sea used to be replaced by sediment brought down to the delta by the River Nile, but this is no longer happening. B. People have blamed this on the two large dams at Aswan in the south of Egypt, which hold back virtually all of the sediment that used to flow down the river. It used to flow freely, carrying huge quantities of sediment north from Africa's interior to be deposited on the Nile delta in 7,000 years. Annual flooding brought in new, nutrientrich soil to the delta region, replacing what had been washed away by the sea, and dispensing with the need for fertilizers in Egypt's richest food-growing area. But when the Aswan dams were constructed in the 20th century to provide electricity and irrigation, and to protect the huge population centre of Cairo and its surrounding areas from annual flooding and drought, not passing down to the delta, most of the sediment with its natural fertilizer accumulated up above the dam in the southern, upstream half of Lake Nasser. C. Now, however, there turns out to be more to the story. It appears that the sedimentfree water emerging from the dams picks up silt and sand as it erodes the river bed and banks on the 800-kilometre trip to Cairo. Daniel Jean Stanley of the Smithsonian Institute noticed that water samples taken in Cairo before the river enters the delta indicated that the river sometimes carries more than 850 grams of sediment per cubic meter of water - almost half of what before the dams were built. 'The significance of this didn't strike me until after I had read 50 or 60 studies,' says Stanley. There is still a lot of sediment coming into the delta, but virtually no sediment comes out into the Mediterranean to replenish the Coastline.'


D. Most of the Nile water is diverted into more than 10,000 kilometers of irrigation canals and only a small proportion reaches the sea directly through the rivers in the delta. The water in the irrigation canals is still or very slow-moving, thus cannot carry sediment, he explains. The sediment sinks to the bottom and then is added to fields by farmers or pumped with the water into the four large freshwater lagoons that are located near the outer edges of the delta. So very little of it actually reaches the coastline to replace what is being washed away by the Mediterranean currents. E. The farms and fishing and aquaculture account for much of Egypt's food supply. But by the time the sediment has come to rest in the fields and lagoons it is laden with municipal, industrial and agricultural waste from the Cairo region. 'Pollutants are building up faster and faster' says Stanley. Based on his investigations of sediment from the delta lagoons, Frederic Siegel of GeorgeWashingtonUniversity concurs. ‘In Manzalah Lagoon, for example, the increase in mercury, lead, etc., coincided with the building of the High Dam at Aswan, the availability of cheap electricity, and major power-based industries development’ he says. Since then mercury concentration has increased significantly. Lead from engines using leaded fuels and from other industrial sources has also increased dramatically. These can easily enter the food chain, affecting the productivity of Fishing and Farming. Another problem is agricultural wastes including fertilizers in the lagoons can upset the ecology of the area with serious effects on the fishing industry. F. According to Siegel, international environmental organizations are beginning to pay loser attention to the region, partly because of the problems of the Nile delta, but principally because they fear the impact this situation could have on the whole Mediterranean coastal ecosystem. But there are no easy solutions. In the immediate future, Stanley believes that one solution would be to make artificial floods to flush out the delta waterways as natural floods did. He says, however, though this is an easier said than done matter, an alternative process such as desalination may have to be used to increase the amount of water available.


Decide whether each statement is true (T) or false (F) or not given (NG) 6. The River Nile no longer brings down sediment to land of the Nile delta 7. People living on the Nile delta could do without fertilizers for their land before the construction of Aswan dams. 8. The farmers pump water from the Nile into their fields to enrich the soil. 9. Aswan dams discharge toxic chemical like lead and mercury. 10. International environmental organizations are still looking for the best solution to the delta problems. PART IV. WRITING Section 1. Rewrite each sentence so that it begins with the words and the meaning stays the same. (10 points) 1. The direct aim of the statement is to make the public aware of the present situation. The statement boils…………………………………………………………………. 2. It’s not our concern what they do after lessons. It’s of………………………………………………………………………………... 3. He was so disgusted at the way she behaved that he refused to speak to her. Such………………………………………………………………………………… 4. All the passengers of the jumbo jet were killed in the crash → None………………………………………………………………………………… 9. The trip is so amazing that we will never forget it. It is too……………………………………………………………………………… Section 2. Rewrite the following sentences with the given words in such a way that the second sentencehas the same meaning as the first one. Do not change the form of the word in brackets. Write your answers in the space provided. (10 points). 1. The mischievous boy never expressed regret for what he had done to the dog. NO


At………………………………………………………………………………………… 2. Mary has been working so hard that it’s logical she feels completely exhausted. STANDS Mary has been working so hard that…………………………………………………. 3. The film script was very different from the novel it was based on.RESEMBLANCE The film script ……………………………………………………………………….. 4. It is difficult to know what my reaction would have been in that situation. (how) I am not………………………………………………………………………………. 5. I suddenly realized the meaning of a “freebie” (dawned) It …………………………………………………………………………………… Section 3. Paragraph (30 points) Some people say that after finishing secondary education, students should spend a gap year travelling, working or doing voluntary work instead of starting their tertiary education. Do you agree or disagree with this point of view? In 130 - 150 words, write a paragraph to express your opinion. Use specific reasons and examples to support your idea.

================ THE END ================= Ngườirađề: PhạmNgọcHân Mobile: 0982186313


ĐÁP ÁN ĐỀ THI ĐỀ XUẤT DHBB MÔN TIẾNG ANH KHỐI 10 TỈNH: PHÚ THỌ PARTI. LISTENING Section 1. 1. 14

2. 1.25

3. (lightweight) bags

4. repair kit

2. B

3. A

5. C

5. credit card

Section 2. 1. B

4. C

Section 3. 1. not fast enough 2. C-suite 3. 37% 4. track key metrics 5. create a level playing field Section 4. 1. 300

2. Sunshade

3. balcony (outside)

4. forest/ forests

5. 319

PART II. LEXICO-GRAMMAR Section 1. 1. B

2. C

3. D

4. B

5. A

6. A

7. D

8. B

9. D

10. C

11. B

12. C

13. B

14. B

15. C

16. D

17. D

18. B

19. B

20. C

Section 2. The New York Times is a daily newspaper published in the New York city. For a long time, it has been the newspaper on record in the US and one of the world’s great newspapers. Its strength is in its editorial excellence, it never has been the greatest newspaper in the term of circulation.


The Times was established in 1851 as a penny paper whose editors wanted to report the news in a restrained and objective fashion. It enjoyed early success as its editors made a pattern for the future by appealing to a cultural, intellectual readership instead of a mass audience. Therefore, in the late 19th century, it came into competition with a more popular, colorful, if not lurid, newspapers in New York. Their publications ran sensational stories, not because they were true, but because they sold newspapers. Despite prices increases, The Times was losing £ 1,000 a week when Adolph Simon Ochs bought it in 1896. Ochs built The Times into an international respected daily. He hired Carr Van Anda as editor. Van Anda placed greatest stress than ever on full reporting of the news of the day, and his emphasized existing good coverage of international news. The management of the paper decided to eliminate fiction of the paper, added a Sunday magazine section, and reduced the paper’s price back to a penny. In April 1912, the paper ran many risks to report every aspects of the sinking of the Titanic. This greatly enhanced its prestige, and in its coverage of the two world wars, The Times continued to enhance its reputation for excellence in world news. 1. on of 2. the term terms 3. made set 4. cultural cultured 5. Therefore However 6. publications publishers 7. prices price 8. greatest greater 9. of from 10. ran took Section 3. 1. up

2. under

3. on

4. to

5. off


6. by

7. with

8. out of

9. up

10. off

Section 4. 1. safety

2. handful

3. consultant

4. enabled

5. freedom

6. uninhibited

7. co-operative

8. readily

9. deterrent

10. temptation

PART III. READING Section 1. 1. D

2. D

3. C

4. D

5. A

6. C

7. B

8. C

9. B

10.A

Section 2. 1. forward

2. fitting

3. legally

4. therefore

5. device

6. charge

7. plate

8. against

9. far

10. unpopular

Section 3. 1. B

2. C

3. C

4. C

5. B

6. B

7. D

8. C

9. C

10.C

Section 4. 1. iv

2. vi

3. i

4. v

5. viii

6. T

7. T

8. NG

9. F

10. T

PART IV. WRITING Section 1. 1. The statement boils down to making the public aware of the present situation. 2. It’s of no/little concern to us what they do after lessons. 3. Such was his disgust at the way she behaved that he refused to speak to her.


4. None of the passengers of the jumbo jet survived the crash. 5. It is too amazing a trip for us to forget. Section 2. 1. At no time did the mischievous boy express (any) regret for what he had done to the dog. 2. Mary has been working so hard that it stands to reason that she feels completely exhausted. 3. The film script bore little/no resemblance to the novel it was based on./ The film script didn’t bear much/any resemblance to the novel it was based on. 4. I am not sure/ certain how I would have reacted in that situation. 5. It suddenly dawned on me what a "freebie" was/ what was the meaning of a "freebie". Section 3. Chú ý: Nếu bài viết lạc đề, không cho điểm. Mô tả tiêu chí đánh giá 1.

Bố cục

Điểm tối đa 5

- Câu chủ đề (Topic sentence) mạch lạc, nêu được ý chính của đoạn văn. - Bố cục hợp lí rõ ràng phù hợp yêu cầu của đề bài. - Có thể có câu kết luận (Concluding sentence) tóm tắt ý chính của đoạn văn, hoặc đưa ra một gợi ý, một quan điểm hay một dự đoán. 2.

Phát triển ý

10

- Phát triển ý có trình tự logic, đủ thuyết phục người đọc. - Có các ý triển khai (suppoting ideas) trong đoạn văn. - Có dẫn chứng, ví dụ, ... đủ để bảo vệ ý kiến của mình. 3.

Sử dụng ngôn từ

10

- Sử dụng ngôn từ đa dạng phù hợp, liên quan đến chủ đề đoạn văn. - Sử dụng ngôn từ đúng văn phong/thể loại. - Sử dụng từ nối các ý cho bài viết uyển chuyển. 4.

Ngữ pháp, dấu câu, và chính tả

5


- Sử dụng đúng dấu câu. - Chính tả: viết đúng chính tả. + Lỗi chính tả gây hiểu nhầm/sai lệch ý sẽ bị tính một lỗi + Cùng một lỗi chính tả lặp lại chỉ tính là một lỗi. - Sử dụng đúng thời, thể, cấu trúc câu đúng ngữ pháp. - Độ dài: 130-150 từ. Tổng

30


Tapescript: Section 1:


Section 2: COUNSELLOR: Well to be quite honest, John, I think they would be useful for everybody but well, everybody has their own way of going about things. I prefer people just to drop in when they can. JOHN: Yes. COUNSELLOR: I find that talking to students about the requirements of a course helps to clarify what needs to be done. I mean the biggest difference between college and school is that new college students really have to do a lot of work on their own, and it's sometimes useful to get advice on how to take control of your time and work effectively. JOHN: Yes. I mean, it seems like a very light workload until assignment time comes and then I seem to be working all night sometimes. I'm not the only one. It's ridiculous. The resource centre is very good but it closes so early. It's in the library and so you'd think you could use it more. It's a real problem for me.


COUNSELLOR: Well, you're certainly not the only person in that position, as I'm sure you've found. It really comes down to using every available hour in a systematic way. If you do this with a plan, then you'll find that you still have time for yourself and your hobbies as well. JOHN: Yeah. I've heard from Thomas that you made him a sort of plan like this, and he's going away for the weekend with all his work handed in, whereas I haven't even started. COUNSELLOR: I need to find out a few more things about you first. I'll give you this form to fill in about your lectures and things before you leave. COUNSELLOR: Now, what are your main problems? JOHN: Well, what most concerns me is I'm still not doing very well in my assignments. COUNSELLOR: Well, I know that you plan your writing carefully, but this can come to nothing if the assignment doesn't answer the question. That really is the key. You must read the question carefully and give it a great deal of thought before you even start planning or writing your first draft. It's also vital to check your work for errors. Everybody makes them, and they can influence the person marking the work. So, always take time at the end to check what you have written. JOHN: As far as listening is concerned, I find it hard to keep up sometimes in lectures, especially two-hour ones. I sometimes just seem to go off into a dream. COUNSELLOR: It's a good idea to find out from your lecturers if they mind you recording the lectures. You only need one of those small cassette recorders. The quality is pretty good and a second listening can really clarify things. Something else you can do is check your notes with a friend after the lecture. JOHN: Yes. That's a good idea. Thanks. It's hard to do all that all the time though, especially when there's so much reading to do. COUNSELLOR: Yes. It's important, though, not to confine yourself to reading on your subject. You should also read things of general interest that appeal to you. You know, novels, newspapers, that kind of thing. Do you have a good dictionary?


JOHN: Not really. I've never bothered with one COUNSELLOR: Mmmm. It would probably be a good idea to get one. Dictionaries are not expensive and they can help a lot. Also you can underline or highlight new words and ... Section 3 Women are graduating from college at higher rates than ever before. Earning more than half of bachelor’s degrees last year. Companies are increasingly focused on the benefits of gender equality in the workplace. However, the situation for women in the workplace is not changing fast enough. Women make up 45% at entry level position across the workforce; which decreases to 32% at management roles. At the top, women only make up seventeen percent of the C-suite. At the current rate of progress, it would be one hundred years before we see real equality in the C-suite. Contrary to popular belief, women are leaving organisations at the same rate as men. However, women are less eager to become a top executive citing stress and pressure as the key deterrents. Most women see barriers to advancement and feel they have fewer opportunities to advance due to their gender. While 74% of companies claim gender equality is a priority for their CEO, less than half of the employees believe it. And only one-third feel it is a top priority for their direct manager. More than 90% of women and men believe taking extended family leave will hurt their career. So while programs widely exist, participation is low. This inequality even extends to the home. 41 % of working women report doing more childcare than men, 30% report doing more chores but this does not need to be our reality while these facts are a wake-up call we can affect real change. Here are five key actions companies can take. 1)Track key metrics to understand the problem: you cannot change why you cannot see.


2)Demonstrate that gender equality is a top priority: employees must believe that their leaders want change. 3)Identify and interrupt gender bias : we all have unconscious biases to overcome. 4)Rethink work: redefine what successful careers look like. 5)Create a level playing field: make sure opportunities and advancement are equitable. It is up to all of us to take action, to speed the process of gender equality so everyone benefits, individuals, organisation and society. To learn more go to the womenintheworkplace.com. Section 4: Man: Greek Island Holidays, can I help you? Woman: Yes, I hope so. I have a friend who’s just come back from Corfu and she’s recommended some apartments in Arilas. She thought they might be on your list. Man: Arilas, Arilas, let me see. Can you give me the names? Woman: Yes, the first’s Rose Garden Apartments. I’d like to go with another friend in the last week of October. Man: Well, we’ve got a lovely studio flat available at that time. I’m sure you’d enjoy the entertainment programme there too, with Greek dancing in the restaurant. Woman: And the cost for each of us? Man: £219. Woman: That sounds very reasonable! I’m just jotting down some notes. Now the second one she mentioned was called BlueBay. Man: BlueBay? Yes, in fact that’s very popular and it has some special features. Really? Man: The main attraction is the large swimming pool with salt water. Woman: Much healthier, I understand. Man: That’s right. And it isn’t far from the beach, either – only 300 metres, and only around half a kilometre to some shops, so you don’t have to be too energetic. Woman: Is it much more expensive than the first one?


Man: Let me just check. I think at the time you want to go it’s around £260 – no £275 to be exact. Woman: Right, I’ve got that. Now there are just two more apartments to ask you about. Um, I can’t read my own writing! Something to do with sun… Sunshine, is it? Man: I think you meant the Sunshade Apartments. They’re on a mountainside. Woman: Any special features? Man: Yes, each room has its own sun terrace and there are shared barbecue facilities. Woman: Sounds lovely! Man: Yes, it is rather well-equipped. It also provides water sports – it has its own beach. There are facilities for water-skiing. Woman: Any kite-surfing? My friend’s quite keen. Man: Not at the hotel but I’m sure you’ll find some in Arilas. There’s also satellite TV in the apartments. Woman: And how much is that one? Man: £490 with two sharing. Woman: You mean £245 each? Man: I’m afraid not! Each person has to pay that amount and there must be at least two in an apartment. Woman: I don’t think that would be within our budget, unfortunately. And the last one sounds a bit expensive too – the Grand! Man: Actually it’s quite reasonable. It’s an older style house with Greek paintings in every room, and a balcony outside. Woman: Sounds nice. What are the views like? Man: Well, there are forests all round and they hide a supermarket just down the road, so that’s very useful for all your shopping needs. There’s a disco in the area too. Woman: And the price? Man: £319 at that time, but if you leave it till November it goes down by 40%. Woman: Too late, I’m afraid.


Man: Well, why don’t I send you a brochure with full details, Ms …? Woman: Nash. But don’t worry about that. I’m coming to Upminster soon and I’ll call and get one. I just wanted to get an idea first. Man: Well, that’s fine. We’ve got plenty there when you come.


KỲ THI HỌC SINH GIỎI CÁC TRƯỜNG THPT CHUYÊN KHU VỰC DUYÊN HẢI VÀ ĐỒNG BẰNG BẮC BỘ LẦN THỨ X, NĂM 2019 ĐỀ THI MÔN: TIẾNG ANH 10 Thời gian: 180 phút (Không kể thời gian giao đề) Ngày thi:

ĐỀ CHÍNH THỨC

(Thí sinh viết câu trả lời vào bảng cho sẵn trong đề)

(Đề thi gồm 16 trang)

Điểm Bằng số

Giám khảo 1

Bằng chữ

Giám khảo 2

Số phách

A. LISTENING (50 points): HƯỚNG DẪN PHẦN THI NGHE HIỂU •

Bài nghe gồm 4 phần, mỗi phần được nghe 2 lần, thí sinh có 30-40 giây giữa mỗi phần nghe để đọc bài.

Mọi hướng dẫn cho thí sinh ( bằng tiếng Anh) đã có trong bài nghe.

Part 1. Listen and complete the notes below. Write NO MORE THAN TWO WORDS OR A NUMBER for each answer. Write your answers in the corresponding numbered boxes. (10 points) TRANSPORT TO BAYSWATER •

Express train leaves at (1)_________________

Nearest station is (2) _________________

Number 706 bus goes to (3) _________________

Number (4) _________________bus goes to station

Earlier bus leaves at (5) _________________

Your answers: 1.

2.

3.

4.

5.

1


Part 2. You will hear an interview with a man who makes models for film and television. Listen carefully and choose the correct answer A, B or C for each question. Write your answers in the corresponding numbered boxes. (10 points) 1. Matt got a job doing holiday relief work because he wanted ___________.

A. to do part-time work B. a career in photography C. to work in television 2. Why were Matt’s models used on the news? A. They were better than pictures. B. Some equipment had been destroyed. C. The studio was trying new ideas. 3. Matt thinks he was successful at getting work in television because___________ A. he had good experience. B. he knew some of the staff. C. he was available at the right time. 4. Matt was invited on children’s television to ___________. A. tell stories about his design work B. explain the purpose of space research C. help children make models themselves 5. Matt remebers Heart of Darkness because ___________

A. it was his favorite comedy. B. his work was recognised. C. a film was made of it. Your answers: 1.

2.

3.

4.

5.

Part 3. You’ll hear part of a talk about dolls. Decide whether the statements are true (T) or false (F). Write your answers in the corresponding numbered boxes. (10 points) 1. The first know dolls were found in graves in ancient Egypt. 2. Early European dolls were dressed like their mechants. 3. 17th century dolls may cost as much as twenty thousand dollars. 4. If you can take off the doll’s hair, you may see the marker’s names underneath. 5. From the 1930s, dolls were made of steel and plastic. Your answers: 1.

2.

3.

4.

5.

2


Part 4: You will hear wolf expert Shaun Ellis talking about his experience of living with wolves. For questions 1-10, complete the sentences with NO MORE THAN THREE WORDS for each gap. Write your answers in the corresponding numbered boxes. (20 points) As a young child, Shaun’s knowledge of wolves came from (1)_______________. In his twenties, Shaun became a (2)________________ in the USA. Shaun’s colleagues considered his method of studying wolves extremely (3)_________________. After his initial encounter with wolves, Shaun developed (4)_____________ for them. Shaun and the wolves lived mainly on a diet of (5)_________________. Shaun felt no desire for (6)________________during his time with the wolves. One day an incident involving food made Shaun aware that he was in a very (7)_______________ position. On one occasion, a wolf protected Shaun and the pack from a (8)_______________. After leaving the wolves, Shaun experienced a massive (9)_______________________. Shaun wanted to show everyone that wolves are not (10)_________________________.

Your answers: 1.

5.

9.

2.

6.

10.

3.

7.

4.

8.

B. LEXICO - GRAMMAR (50 points) Part 1. Choose one of the words marked A, B, C, or D which best completes each of the following sentences. Write your answers in the corresponding numbered boxes. (20 points)

1. John did nine hours’ ____ studying a day for his exam. A. strong

B. solid

C. heavy

D. big

2. I suppose I____ lent the book to Simon, but I’m almost sure I didn’t. A. might

B. could

C. might have

D. must have

3. People were injured by ____ stones the size of tennis balls. A. sleet

B. gale

C. hail

D. snow

4. The police ____ off the street when the bomb had gone off. A. cordoned

B. battened

C. fastened

D. shuttered

5. “Were you told to get dinner ready?” “No, I did it of my own____.”

3


A. desire

B. accord

C. idea

D. will

6. Many Asian countries still rely on rice as the ____ food. A. staple

B. capital

C. superior

D. winning

7. He was arrested for trying to pass ____ notes at the bank. A. camouflaged

B. fake

C. counterfeit

D. fraudulent

8. He offered me $500 to break my contract. That’s____. A. bribery

B. blackmail

C. compensation

D. reward

C. place

D. pull

9. I recognise that voice, but I can’t quite ____ it. A. know

B. realize

10. It’s so bad to have a boss who ____ all the time. A. breathes down your B. follows your nose

C. keeps your temper

D. draws your eyes

neck 11. The estate company is now ____ their new apartment building. A. hiring

B. chartering

C. leasing

D. mortgaging

12. He still suffers from a rare tropical disease which he ____ while in Africa. A. infected

B. complained

C. gained

D. contracted

13. The athlete is now making his last attempt ____ the world 100-meter breastroke record. A. in

B. at

C. for

D. on

14. Can you stuck your shirt ________ your trousers? A. on

B. around

C. into

D. at

15. Jack________ at the map for a while, unable to belive his eyes. A. stared

B. glimpsed

C. glanced

D. gazed

16. The minister has, in a ________ of speaking, resigned. A. style

B. format

C. form

D. manner

17. Oliver was ________ the worse for his experience in the Bolivian jungle. A. no

B. nothing

C. none

D. hardly

18. Each of the five peace plans will be judged on ________. A. its own pros

B. its own merits

C. its own strength

D. its own benefit

19. As there is little hope of being rescued, I have ________ myself to the worst. A. resigned

B. convinced

C. regarded

D. reminded

20. Let me please ________ my memory before I get down to answering the questions. A. resume

B. ease

C. awake

D. refresh

Your answers:

4


1.

2.

3.

4.

5.

6.

7.

8.

9.

10.

11.

12.

13.

14.

15.

16.

17.

18.

19.

20.

Part 2. Read the text below. Some of the lines contain an extra, incorrect word. If there is an extra, incorrect word in a line type the word in the box on the right. If the line is correct, write 'OK'. (10 points) Your answers 0 Can you barely see your desk at the work for the mountain (0)Line 0:the-> a 00 of paperwork on it? If this is the case then you probably (00) OK 1 feel like going to home before you’ve even started work. Line 1:__________ 2 As some people argue that they never file away important Line 2:__________ 3 documents because they are afraid of not being able to Line 3:__________ 4 find them out. However, it has been calculated that the average Line 4:__________ 5 office worker spends around 22 minutes per day looking Line 5:__________ 6 for lost paperwork because of it hadn’t been filed correctly. Line 6:__________ 7 Some people’s desks are so much cluttered with paper and Line 7:__________ 8 equipment that they hardly have had any space left for working Line 8:__________ 9 on. Think how much more productive and creative they Line 9:_________ 10 could be with a clear desk! Don’t fall on into the trap of Line 10:_________ 11 thinking about that clutter can only be of the paper variety. Line 11:_________ 12 The principles of desk-clearing can also be apply to the storing Line 12:_________ 13 of electronic documents on computer. It could be time to go Line 13:_________ 14 through your hard disk and delete any of files that you do not Line 14:_________ 15 need anymore or at least to copy them onto an ordered system Line 15:_________ of floppy disks

Part 3. Complete each of the following sentences with a suitable preposition or particle. Write your answer in the boxes provided. (10 points) 1. My son drove me crazy about buying a new bike, so finally I gave _______. 2. Eric is very sick at the moment but I am sure he’ll pull _______. 3. You shouldn't talk _______to him like that. You are not his boss. 4. John told me that he didn't steal but I didn't fall _______ his lies.

5


5. Don’t worry about the broken windown. I’ll get _______ to it one of these days. 6. Can you point _______ the man carrying a big backpack? 7. The police chief couldn't solve the case, so he wanted to call _______ the FBI. 8. When I saw Linda, I ran_______ to her and gave her a hug. 9. The clock is completely broken. I’ll try to take it _______ and fix it. 10. Have you picked _______ a dress to wear at the party? Your answers: 1.

2.

3.

4.

5.

6.

7.

8.

9.

10.

Part 4. Give the correct form of the words in brackets. Write your answers in the corresponding numbered boxes. (10 points) The cosmic game of darts ‘The End of the World is Nigh!’ screamed newspaper (1. HEAD)__________ recently after astronomers warned that a massive asteroid was heading (2. AVOID) __________for Earth. It was predicted that the mile-wide asteroid ZF11 would hit the Earth in 2028 with (3. CATASTROPHE) __________ consequesces. It sounded like the stuff of science fiction, but for a while it seemed (4. ALARM) __________threatening.Then suddenly, the danged disappeared. New calculations showed that the asteroid would miss by 600,000 miles-only fairly close in (5. ASTRONOMY) __________terms, but in practical terms a fairly (6. SIGNIFY) __________risk. ZF11 may not pose a danger, but the threat of other strikes still looms, and there is (7. ABOUND) __________evidence of past asteroid strikes. One of the most (8.ORDINARY) __________of these is the crater in the Yucatan Peninsula which measures some 200km in diameter. It is thought that this is the impact site of an asteroid that struck sixty-five million years ago, wiping out the dinosaurs and leading to the destructon of 70% of the Earth’s species. Scientist belive that it is only a matter of time before another celestial hulk hits home in this giant game of cosmic darts. But what really worries astonomers is the (9. REAL) __________that they have identified very few of the asteroids near the Earth. There are (10. FORTUNATE) __________ thousands more that they do not know about. Your answers: 1.

5.

9.

2.

6.

10.

3.

7.

4.

8.

6


C. READING (60 points) Part 1. Read the following passage and decide which answer (A, B, C, or D) best fits each gap. Write your answers in corresponding numbered boxes. (15 pts) TALKING RUBBISH Reduce! Re-use! Recycle! The message hits Canadian consumers through all the media. As newcomers from Sri Lanka, we compare the situation here with the one back home. We may not be the most environmentally (1)________ citizens in the world but, compared with this, we do not have a rubbish problem - yet. Like many shoppers in Colombo, my partner Shahid and I used to have a cane basket we took along with us to the Sunday market or pola every week. No environmentalist could have complained about it. You need a good strong basket at the pola. There are no supermarket carts to push around. Most items - rice, flour, vegetables, fruit, biscuits, eggs - are bought (2)________ or wrapped in newspaper. At (3)________ we would carry one plastic bag separately. For eggs we took a reusable plastic tray with us. When income (4)________ are low, people need to buy in small quantities. It is quite normal to ask for a (108)________ envelope, two eggs or 100 grams of sugar. The (5)________ is that, for the most part, urban consumers in Sri Lanka cannot afford the luxury of waste. Most people do not buy more from the grocers than they know they will actually consume. They re-use whatever they can and are loath to discard bags, jars, tins or boxes that can be (6)________ to other uses. But in recent years Western-style supermarkets have begun to spring up in Colombo. They hold out the (7) ________ of a clean, efficient, streamlined service to customers. A imported goods, dressed up in their layers of attractive, colourful

(8) ________ of

(9) ________ beckons from the

shelves. These are the (10) ________ products that demand your attention on the TV advertisements. Along with them, Sri Lanka, like so many other developing countries, may have imported a problem that once never existed.

1. A. qualified

B. concerned

C. worried

D. experienced

2. A. free

B. in pieces

C. bit by bit

D. loose

3. A. maximum

B. most

C. highest

D. best

4. A. rates

B. amounts

C. sizes

D. levels

5. A. point

B. case

C. example

D. question

6. A. made

B. set

C. given

D. put

7. A. promise

B. advantage

C. evidence

D. sight

7


8. A. set

B. range

C. store

D. band

9. A. packets

B.packs

C. packaging

D. pecks

10. A. very

B. just

C. similar

D. likely

Your answers 1.

2.

3.

4.

5.

6.

7.

8.

9.

10.

Part 2. Read the text below and think of the word which best fits each space. Use only ONE word in each space. Write your answers in the corresponding numbered boxes. (15 points) VIETNAM TOURISM GUIDE Vietnam is one of Southeast Asia’s most beautiful countries, attracting travellers to its lush mountains, bustling cities and golden sand beaches. (1)_____ rapid modernisation in Vietnam’s urban centres of Hanoi and Ho Chi Minh, time-honoured traditions (2)_____ intact amongst the locals. There are numerous ancient landmarks and colonial structures that have survived throughout the (3)_____ and scrapes of history. Buddhist shrines can be found next to towering skyscrapers, (4)_____ you can experience vibrant festivals and ceremonies dating back hundreds of years. Long, sandy beaches are (5)_____ to luxurious resorts and fine-dining venues, making them ideal romantic getaways, while expansive rice (6)_____ on the hills make for excellent hiking excursions. (7)_____ you’re planning a historical tour of Hanoi and Ho Chi Minh, a relaxing beach break or simply want to escape the bustling city life, our Vietnam travel guide caters (8)_____ just about any holiday preference. If this is your first time to Vietnam, getting to know the country’s many cultural issues, festivals, and customs allows you to (9)_____

experience the local lifestyle as you get to enjoy vibrant

celebrations and religious (10)______ that you won’t find in other places. We’ve also compiled other local information you may need during your travels to help you further understand this unique country. (Source: http://www.vietnam-guide.com) Your answers 1.

2.

3.

4.

5.

6.

7.

8.

9.

10.

Part 3. Read the passage and choose the best option A, B, C, or D to answer the questions. Write your answers in the corresponding numbered boxes. (15 points)

8


When people learn to play video games, they are learning a new literacy. Of course, this is not the way the word "literacy" is normally used. Traditionally, people think of literacy as the ability to read and write. Why, then, should we think of literacy more broadly, in regard to video games or anything else, for that matter? There are two reasons. First, in the modern world, language is not the only important communicational system. Today images, symbols, graphs, diagrams, artifacts, and many other visual symbols are particularly significant. Thus, the idea of different types of "visual literacy" would seem to be an important one. For example, being able to "read" the images in advertising is one type of visual literacy. And, of course, there are different ways to read such images, ways that are more or less aligned with the intentions and interests of the advertisers. Knowing how to read interior designs in homes, modernist art in museums, and videos on MTV are other forms of visual literacy. Furthermore, very often today words and images of various sorts are juxtaposed and integrated in a variety of ways. In newspaper and magazines as well as in textbooks, images take up more and more of the space alongside words. In fact, in many modern high school and college textbooks in the sciences images not only take up more space, they now carry meanings that are independent of the words in the text. If you can't read these images, you will not be able to recover their meanings from the words in the text as was more usual in the past. In such multimodal texts (texts that mix words and images), the images often communicate different things from the words. And the combination of the two modes communicates things that neither of the modes does separately. Thus, the idea of different sorts of multimodal literacy seems an important one. Both modes and multimodality go far beyond images and words to include sounds, music, movement, bodily sensations, and smells. None of this news today, of course. We very obviously live in a world awash with images. It is our first answer to the question why we should think of literacy more broadly. The second answer is this: Even though reading and writing seem so central to what literacy means traditionally, reading and writing are not such general and obvious matters as they might at first seem. After all, we never just read or write; rather, we always read or write something in some way. So there are different ways to read different types of texts. Literacy is multiple, then, in the sense that the legal literacy needed for reading law books is not the same as the literacy needed for reading physics texts or superhero comic books. And we should not be too quick to dismiss the latter form of literacy. Many a superhero comic is replete with post-Freudian irony of a sort that would make a modern literary critic's heart beat fast and confuse any otherwise normal adult. Literacy, then, even as traditionally conceived to involve only print, is not a unitary thing but a multiple matter. There are, even in regard to printed texts and even leaving aside images and multimodal texts, different "literacies."

9


Once we see this multiplicity of literacy (literacies), we realize that when we think about reading and writing, we have to think beyond print. Reading and writing in any domain, whether it is law, rap songs, academic essays, superhero comics, or whatever, are not just ways of decoding print, they are also caught up with and in social practices... Video games are a new form of art. They will not replace books; they will sit beside them, interact with them, and change them and their role in society in various ways, as, indeed, they are already doing strongly with movies. (Today many movies are based on video games and many more are influenced by them.) We have no idea yet how people "read" video games, what meanings they make from them. Still less do we know how they will "read" them in the future. 1. According to the first paragraph, the broadest definition of "literacy" is______. A. The ability to analyze literature B. The ability comprehend basic cultural cues C. The ability to read and write D. The ability to compose poetry 2. All are mentioned as being types of "visual literacy" EXCEPT______. A. Musical tones

B. Interior Design

C. Diagrams

D. Modern Art

3. An example from a science textbook of the phenomenon the author describes in the third paragraph could be______. A. A genetic tree that coincides with the discussion of specific mammal classes in the text B. A diagram of a specific chemical reaction that is used to explain a broad definition in the text C. An illustration of a plant cycle that accompanies a chapter on photosynthesis D. A cartoon that references the same methods discussed in the text about laboratory safety 4. What is an example of a "multimodal" text? A. A dictionary B. A movie script C. A photo album D. An art book that describes the art as well as reproduces images of the original prints 5. The idiom in the sixth paragraph, "read against the grain of the text" is closest in meaning to______. A. Reading to understand the underlying meanings and themes of the author's words-not just a literal interpretation B. Reading text that defines different types of wheat and grains C. To read the text from right to left rather than left to right D. To read books that use recycled paper and other green alternatives

10


6. In the seventh paragraph, the author suggests that literacy is multiple, meaning that______. A. To be "literate" can mean participating in any form of expression B. One's literacy increases exponentially as greater mastery of reading and writing is achieved C. Different genres and modes of expression require different background knowledge and perspectives to understand them D. Literacy can only be gained by exploring every type of media and expression 7. Why does the author give the example of superhero comics to explain multiple literacies? A. To explain that comic books are written for children and purely for entertainment. They require only a basic knowledge of the action that occurs in the story B. To once again refer to his earlier points about "multimodal" texts C. To insist that even when an author may intend multiple meanings and interpretations, they are rarely successful in conveying those to readers D. Things that may seem on the surface to be only meant for a particular group of people can actually have very profound meanings to those who possess other types of literacy 8. The author suggests that all of the following require different types of literacy and the ability to decode meaning EXCEPT______. A. Rap music

B. Comic books

C. Academic papers

D. Symphonies

9. The author says that video games______. A. Are not yet entirely understood in terms of literacy, but are already impacting other forms of expression such as filmmaking B. Are unrealistic and should not fall into the same categories as the other texts he describes C. Are too violent to risk experimenting with for the purposes of understanding literacy D. Are irrelevant in academic discussion because no one has yet determined how to explain the ways that people understand them 10. What would be the most logical information for the next paragraph to contain if the article continued? A. A technological definition of video games, how they are made, and how they are played B. A historical explanation of the very first video game and its evolution C. Examples of the way that some people currently interpret video games and what they mean to them D. A price comparison of video game consoles and whether or not quality has a direct impact on literacy

11


Your answers 1.

2.

3.

4.

5.

6.

7.

8.

9.

10.

Part 4. Read the text and do the following tasks. (15 points) For questions 1-6, choose the correct heading for paragraphs A-F from the list of headings below. Write your answers in the corresponding numbered boxes. i. A question of time ii. Lack of teacher training iii. Student success iv. The argument for flexibility v. Importance of teaching experience vi. Extra-curricular pressures vii. The benefits of a varied curriculum viii.

Imbalanced focus

ix. Over-reliance on examinations x. Quality of quantity?

1. Paragraph A ____________ 2. Paragraph B ____________ 3. Paragraph C ____________ 4. Paragraph D ____________ 5. Paragraph E ____________ 6. Paragraph F ____________ Weakness of the school system A. By attempting to fit in as much as possible, the school day is continually being added to. In many ways, this would appear to be a good idea, as our knowledge and understanding of the world is always growing and it would seem logical to incorporate this into schools. The reality, however, has some decided drawbacks. There is a growing feeling amongst many that the modern school curriculum, in an effort to teach as many varied subjects as possible, is actually teaching students less. It seems that by constantly adding to what should be taught in the classroom, the classes are less focused, not offering the deeper learning that institutions perhaps should.

12


B. With classes sometimes only 30 minutes long, the overwhelming amount of information teachers are required to present often only gives students time to learn facts, not to think in any great detail about what they are being presented with. The problem is that students are not getting the opportunity to absorb what they are being taught as the curriculum expands in order to keep what has already been taught and supplement it with everything new that comes along. The weaknesses of such a system are clear – well informed though such students may be, there is the risk of an increasing number of graduates who have no real creative or intellectual ability. By denying students the opportunity to sit and think their way through problems, or even consider their own opinion, some schools are not always providing a truly educational atmosphere. There are, of course, certain aspects of education which need to be taught by simply inputting the information. Basic mathematics, for example. But there are many other subjects which could be best learned by having an opportunity to think and discuss what is being taught. Literature, writing and the social sciences are good examples of subjects which cannot be considered as ‘covered’ by a mass of information without the opportunity to discuss, debate or consider meaning or implications. There are also important social skills to be learned during such periods of open discussion, skills which are not addressed by an endless flow of teacher-centred information. C. Teachers themselves have also voiced concerns about the amount of information they are required to impress upon their students. There is a feeling in many educational establishments that students are no longer being educated, but taught how to pass tests. In a world where academic success is too often measured by examination results, this is a serious concern. If there is too much information to simply be memorised and not enough time to truly assimilate it, what happens to students who fail to meet the grade? By current standards, they are failures, yet they may have great potential in areas not covered by the test and there are many students who, despite clear intellectual ability, simply do not perform well in tests. Again, the problem is one of focus, as education authorities are looking at the outcome of schooling rather than the content presented in the class. D.

It is here that many teachers feel the situation could be addressed at a local level. By giving

more discretion to teachers, school courses could be tailored to suit the students rather than tailoring students to meet ever-expanding course requirements. In addition, by running a curriculum that gives options rather than defines an entire course, considerably more freedom would be possible. As it is, progression through most primary and secondary schools is regimented, and there is little room for students to identify and develop their own skills and strengths. If material could be chosen on the basis of its merits rather than simply because it has

13


been put in the curriculum, then what is selected may be taught to a depth that would serve some purpose. There is, of course, a counter-argument, which claims that such open guidelines could lead to vast differences in standards between schools. What one teacher may see as essential for a student’s education, another may see as irrelevant, and this will result in students with widely different educational strengths. E. With such a high-pressure learning environment, there are also a number of social aspects to schooling which need to be considered. The increased student workload cannot be covered in the classroom alone for the simple reason that there is not enough time in the average school week, and much of this extra workload has been pushed into the realm of homework. At its best, homework should be the opportunity to look in greater detail at what has been studied. In other words, to actually think about it and its relevance. The reality, however, is often very different. Concerned parents and overextended students are finding that homework is taking an increasingly large part of a student’s evening, cutting into time many feel should be spent as part of a child’s social education. Other social pressures have compounded the situation, as many of the areas of educating a young child which should be the responsibility of the parents have ill-advisedly become the school’s responsibility. Drug awareness and health issues, for example, are occupying an increasingly large part of the school day. F. Many people believe that we should be teaching less, but teaching it better, and it is here that they think a solution can be found. Yet the process of rewriting a curriculum to incorporate only that which is essential but can be well learned would take far longer than most educational authorities have, and would be considered by many to be a ‘regressive’ step. Changes in the curriculum have largely been motivated by changes in the nature of employment, as job mobility demands that people know something about considerably more areas than were traditionally necessary. A little about a lot allows for the job mobility which has become so common. No matter what the final verdict may be, one thing is for sure – change will be slow, and not always for the best. Questions 7-10: Complete the summary using the list of words, A-G, below. A. more discretion

B. in detail

C. differences in standards

D. the extra workload

E. job mobility

F. shorter classes

G. facts

H. a regimented progression

I. a weaker system

J. general knowledge

14


Too much emphasis is placed on learning (7) response to increased (8)

. The modern school curriculum is largely a

for which graduates are expected to have a much broader

(9)_________. One potential solution to this could be to give individual schools (10)

regarding

what is taught. Your answers 1.

2.

3.

4.

5.

6.

7.

8.

9.

10.

D. WRITING (40 points) Part 1. Finish each of the following sentences in such a way that it means the same as the one printed before it. Write your answers in the space provided. (10 points) 1. You can use it as long as you like, and it won’t wear out. No…………………………………………………………………………… 2. It was not until five years had pased that the whole truth came out. Not for ………………………………………………………………………… 3. The only thing that makes this job worthwhile is the experience. Were it ……………………………………………………………. 4. The inhabitants were far worse-off twenty years ago than they are now. The inhabitants are nowhere ………………………………………………………… 5. Mr. David asked me to write this letter to you. It is at Mr. David’s ………………………………………………………………

Part II. Rewrite the sentences below in such a way that their meanings stay the same. You must use the words in capital without changing their forms. Write your answers in the space provided (10 points) 1. Since she left me, I’ve been leading a boring life.

VEGETABLE

Since she left me, ……………………………………………………………………… 2. My mother insisted me on wearing the dress

PRESSED

My mother ………………………………………………………………………………….. 3. The project failed because of a lack of financial backing.

WANT

The project failed ………………………………………………………………………….. 4. I would give up my job in no time if I could find a better one.

HAT

I would give up my job …………………………………………………………………..

15


5. She decided to have five bowls of rice because she was hungry. MUNCHIES ……………………………………………….., she decided to have five bowls of rice.

Part 3. Paragraph writing. (20 points) Some people think that the teenage years are the happiest times of most people’s lives. Do you agree with this statement? Write a paragraph of 150-200 words to state your viewpoint. ……………………………………………………………………………………………………… ……………………………………………………………………………………………………… ……………………………………………………………………………………………………… ……………………………………………………………………………………………………… ……………………………………………………………………………………………………… ……………………………………………………………………………………………………… ……………………………………………………………………………………………………… ……………………………………………………………………………………………………… ……………………………………………………………………………………………………… ……………………………………………………………………………………………………… ……………………………………………………………………………………………………… ……………………………………………………………………………………………………… ……………………………………………………………………………………………………… ……………………………………………………………………………………………………… ……………………………………………………………………………………………………… ……………………………………………………………………………………………………… ……………………………………………………………………………………………………… ……………………………………………………………………………………………………… ……………………………………………………………………………………………………… ……………………………………………………………………………………………………… ……………………………………………………………………………………………………… ……………………………………………………………………………………………………… ……………………………………………………………………………………………………… ……………………………………………………………………………………………………… ……………………………………………………………………………………………………… ……………………………………………………………………………………………………… ………………………………………………………………………………………………………

16


……………………………………………………………………………………………………… ……………………………………………………………………………………………………… ……………………………………………………………………………………………………… ……………………………………………………………………………………………………… ……………………………………………………………………………………………………… ……………………………………………………………………………………………………… ……………………………………………………………………………………………………… ……………………………………………………………………………………………………… ……………………………………………………………………………………………………… ……………………………………………………………………………………………………… ……………………………………………………………………………………………………… ……………………………………………………………………………………………………… ……………………………………………………………………………………………………… ………………………………………………………………………………………………………

-------------- THE END --------------

17


KỲ THI HỌC SINH GIỎI CÁC TRƯỜNG THPT CHUYÊN KHU VỰC DUYÊN HẢI VÀ ĐỒNG BẰNG BẮC BỘ LẦN THỨ XII, NĂM 2019 ĐÁP ÁN MÔN: TIẾNG ANH LỚP: 10

ĐÁP ÁN (Đáp án gồm 4 trang) A. LISTENING (50 points): Part 1. 10 points- Each correct answer is given 2 pts 1. 9.30 (am)

2. Helendale 3. Central Street/St

4. (Number/No/#) 792

5. 8.55 (am)

Part 2. 10 points- Each correct answer is given 2 pts 1. C

2. B

3. C

4. C

5. B

4. T

5. F

Part 3: 10 points- Each correct answer is given 2 pts 1. T

2. F

3. F

Part 4: 20 points- Each correct answer is given 2 pts 1. books and films

5. deer and elk

9. culture shock

2. basic field biologist

6. human contact

10. savage and ruthless

3. dangerous

7. vulnerable

4. respect

8. bear

B. LEXICO - GRAMMAR (50 points) Part 1. 20 points- Each correct answer is given 1 pt 1. B

2. C

3. C

4. A

5. B

6. A

7. C

8. A

9. C

10. A

11. C

12. D

13. D

14. C

15. A

16. D

17. C

18. B

19. A

20. D

Part 2. 10 points - Each correct answer is given 1 pt

1


Line

Mistakes

1.

1

to

2.

2

As

3.

4

out

4.

6

of

5.

7

much

6.

8

had

7.

10

on

8.

11

about

9.

12

be

10.

14

of

Part 3. 10 points - Each correct answer is given 1 pt. 1. in

2. through

3. down

4. for

5. around

6. to

7. in

8. over

9. apart

10. out

Part 4. 10 points - Each correct answer is given 1 pt. 1. headlines

5. astronomical

9. realization

2. unavoidably

6. significant

10. unfortunately

3. catastrophic

7. abundant

4. alarmingly

8. extraordinary

C. READING (60 points) Part 1. 15 points- Each correct answer is given 1.5 pts, 1. B 6. D

2. D 7. A

3. B 8. B

4. D 9. C

5. A 10. A

Part 2. 15 points- Each correct answer is given 1.5 pts. 1. Despite 6. terraces

2. remain 7. Whether/ If

3. bumps 8. to

4. where 9. fully

5. home 10. rituals

Part 3. 15 points- Each correct answer is given 1.5 pts.

2


1. C 6. C

2. A 7. D

3. B 8. D

4. D 9. A

5. A 10. C

Part 4. 15 points- Each correct answer is given 1.5 pts. 1. x 6. i

2. viii 7. G (facts)

3. ix 8. E (job mobility)

4. iv 9. J (general knowledge)

5. vi 10. A (more discretion)

D. WRITING (40 points) Part I. 10 points- Each correct answer is given 2 pts 1. No matter how long you use it, it won’t wear out. 2. Not for another five years did the whole truth come out. 3. Were it not for the experience, this job would not be worthwhile. 4. The inhabitants are nowhere near as badly-off as they were twenty years ago. 5. It is at Mr. David’s request that I’m writing this letter to you. Part II. 10 points- Each correct answer is given 2 pts. 1. Since she left me, I’ve become a vegetable. 2. My mother pressed the dress on me/ My mother pressed me into wearing dress. 3. The project failed for (the) want of financial backing. 4. I would give up my job at the drop of a hat if I could find a better one. 5. Having the munchies, she decided to have five bowls of rice. Part 3. Paragraph writing. (20 points) Some people think that the teenage years are the happiest times of most people’s lives. Do you agree with this statement? Write a paragraph of 150-200 words to state your viewpoint. Criteria: 1. Task response (5 points) The writer clearly states their point of view (agree/disagree) and provides a well-supported paragraph related to the topic. 2. Lexical resource (5points) -

The writer uses synonyms and paraphrases flexibly.

-

The writer uses topic-related vocabulary.

3. Coherence and cohesion (5 points) -

The paragraph has a topic sentence with controlling idea.

-

The topic is well-developed with relevant supporting evidence, examples and facts.

-

Ideas are well connected with suitable cohesive devices.

3


-

The paragraph shows a certain organization pattern (for example: by order of importance, etc.)

-

The writer uses pronouns consistently and coherently, with third-person pronouns (They, this, these, one/ones) gaining higher scores.

4. Grammatical range and accuracy (5 points) -

The writer uses a wide range of sentence structures (simple, compound and complex)

-

The writer uses verb tense and forms accurately.

-

The writer shows good control of spelling and punctuation. THE END

4


TAPE SCRIPT Part 1

Woman: Good morning, Travel Link. How can I help you? Man: Good morning. I live in Bayswater and I’d like to get to Harbour City tomorrow before 11am. Woman: Well, to get to Bayswater … Man: No, no. I live in Bayswater – my destination is Harbour City. Woman: Sorry. Right; so that’s Bayswater to Harbour City. Are you planning to travel by bus or train? Man: I don’t mind really, whichever option is faster, I suppose. Woman: Well, if you catch a railway express, that’ll get you there in under an hour … Let’s see – yes, if you can make the 9.30am express, I’d recommend you do that. Man: Great. Which station does that leave from? Woman: Helendale is the nearest train station to you. Man: Did you say Helensvale? Woman: No, Helendale – that’s H-E-L-E-N-D-A-L-E Man: What’s the best way to get to the Helendale station then? Woman: Well, hang on a minute while I look into that … Now, it seems to me that you have two options. Option one would be to take the 706 bus from the Bayswater Shopping Centre to Central Street. When you get there, you transfer to another bus which will take you to the station. Or, the second option, if you don’t mind walking a couple of kilometres, is to go directly to Central Street and get straight on the bus going to the train station. Man: Okay. Which bus is that? Woman: The 792 will take you to the station. Man: I guess the walk will be good for me so that might be the better option. What time do I catch the 792? Woman: There are two buses that should get you to the station on time: one just before nine o’clock and one just after. But look, at that time of the morning it might be better to take the earlier one just in case there’s a traffic jam or something. The 8.55 is probably safer than the 9.05. Man: Yeah, I don’t want to the miss the train, so I’ll be sure to get on the five- to- nine bus. Part 2.

5


Part 3

6


Part 4: Gap-filling Shaun Ellis Like most young children, I grew up with an innate fear of wolves. It wasn’t until I was a bit older and saw a wolf in a zoo that I realized how far away this animal was from the mythological creature I’d learned about in books and films. I grew up in a small village in Norfolk and was always interested in the natural world and wild animals. I knew I wanted to work with them in some way when I was older. In my 20s, I read about an American naturalist, Levi Holt, who ran a wolf research centre in Idaho and I thought, ‘That’s where I want to go.’ I sold

7


everything I had and raised enough money for my plane fare. When I met up with biologists working on the reservation, they took me on as a basic field biologist, teaching me how to track wolves and collect data for them. But I couldn’t help wondering, ‘Could a human become part of their family?’ If I could, I thought – imagine what information I could share. I found myself wanting to get closer to the wolves to really understand their behaviour, even though the other biologists and scientists thought it would be dangerous. After a year or two of working for the centre and getting to know the area – a rugged, mountainous landscape covered in forest – I moved to the wild. The first time I got up close to a wolf, within about thirty metres, any fear I had quickly turned to respect. I stayed in a den area, a remote spot where wolves look after their young, and very soon one pack began to trust me. I lived with them day and night, and from the start they accepted me into their group. I ate what they ate, mostly raw deer and elk, which they would often bring back for me, or fruit and berries. I never fell ill and my body adapted quickly to its new diet. It’s easy to look back and think, ‘What horrible food’, but when you haven’t eaten for a week, it looks appetizing. I couldn’t hunt, but I soon became useful looking after the younger ones. I would spend days sitting outside the den, observing their behaviour and trying to make sure they kept out of danger. I stayed with the same pack for over a year, watching pups grow to adulthood. I never missed human contact during that time. I felt a tremendous sense of belonging with the wolves. Whenever I began to think about my old life, I would quickly switch my thoughts back; in terms of survival, I had constantly to focus on my new habitat. Although I didn’t see anyone, there were people back at the reservation and I had a rendezvous point where I could leave messages if I felt I was in danger. I was only ever truly scared on two occasions: once, when all the wolves were feeding, I ate the wrong piece of meat – there’s a strict hierarchy of who eats what part of an animal – and one of the wolves leapt on me in seconds because of my mistake. He took my entire face in his mouth and started to squeeze hard. I could feel the bones in my jaw begin to bend, and in that splitsecond I realized how vulnerable I was and how restrained they were most of the time. The other time, I wanted to get a drink from the stream and one of the wolves stopped me dead in my tracks, growling, snarling and nipping me. I thought, ‘This is the end, he’s going to finish me off.’ An hour or so later, he started to lick my face and we both went to the stream for a drink. There I saw evidence of recent bear tracks and droppings, and I realized this was why he guarded me. I would almost certainly have been killed but, more importantly, my tracks would have led back to their young, so it was for their protection. Eventually I had to leave; I had lost so much weight and looked gaunt and worn after a year. Life expectancy in that sort of environment was short and I felt it was time to come back to society. Returning to the world was a tremendous culture shock, but I knew I could do a lot with the knowledge I’d acquired. I now run a centre in Devon that helps wild and captive wolves, and offers educational courses. I want to show people that wolves aren’t savage and ruthless – they’re balanced and trustworthy creatures that place their family above all else.

8


SƠN LA TRƯỜNG THPT CHUYÊN S

ĐỀ ĐỀ XUẤT ĐỀ THI HSG KHU VỰC NĂ 2019 LẦN THỨ XII, NĂM MÔN TIẾNG ANH - LỚP 10 Thời gian làm bài: 180 phút.

________________________________________________________________________ PART I: LISTENING. SECTION 1 Questions 1-5: Complete the form with missing information.

VIDEO LIBRARY APPLICATION FORM EXAMPLE ANSWER Surname: Jones First names: Address:

Louise Cynthia (1) ...................................... Street Highbridge

Apartment 1, 72

Post code:

(2) ......................................

Telephone: 9835 6712 (Home) Driver's licence number: DOB: 25th Month:

-

(3) ...................................... (Office)

(4) ......................................

(5) ......................................

Year: 1977

Your answers: 1.

2.

4.

5.

3.

SECTION 2 Questions 6-10: You will hear an interview with Pamela Green, a young fashion designer. For questions 6 - 10, choose the best answer (A, B or C). 6. What helped Pamela to decide to become a fashion designer? A. working as an assistant in a fashion shop B. doing research into the fashion industry


C. attending a course on fashion design 7. What does Pamela say about having a degree in fashion? A. It's essential for promotion. B. It's evidence of your ability. C. It guarantees you a better income. 8. Pamela says that when starting your own fashion label, it's most important to… A. enjoy the creative process. B. contact shops that might sell it. C. have a business plan. 9. According to Pamela, successful designers need to be able to… A. predict future fashions. B. recognise all past styles. C. get their designs published. 10. What advice does Pamela have for people who want a career in fashion? A. Be aware of the options available. B. Don't be afraid of sudden fame. C. Learn from your own errors. Your answers: 6.

7.

9.

10.

8.

SECTION 3 Questions 11 - 15: You will hear a man called Steve and a woman called Caroline talking about summer jobs. Decide if each sentence is correct or incorrect. If it is correct, put a tick ( ) in Yes column. If it is not correct, put a tick ( ) in No column. YES

11. Steve hasn’t arranged any work for the summer yet. 12. Caroline’s work will allow her to have free time during the day. 13. Caroline’s work will be located in a city.

NO


14. Caroline found out about the job from the internet. 15. Caroline says that work at music festivals is badly paid. SECTION 4 Questions 16 - 25: You are going to hear somebody giving their opinion about media and its influence on society. For questions 16-25, complete the sentences with NO MORE THAN THREE WORDS. The three form of media are TV, radio and (16) ______________. Politicians use the media during political (17) ____________________ . Entertainers use it to stay in the (18) ____________________. The percentage of people who did not have a particular point of view about the privacy of celebrities was (19)__________________. The most influential form of media is (20)______________________ . We can be deceived by TV when we are shown carefully selected (21) ____________________. Some TV stations turn true events into (22)____________________. A newspaper had to pay (23) ____________________ for printing lies about a pop singer. Sometimes a paper can avoid being sued if it makes a public (24) ____________________. There are two kinds of newspaper; the “quality” papers and the (25) _________________________ . Your answers: 16. ………………

17. ………………

18. ………………

19. ………………

20. ………………

21. ………………

22. ………………

23. ………………

24. ………………

25. ………………

PART II: LEXICO - GRAMMAR. SECTION 1 Choose the word or phrase which best completes each sentence. 1. It’s nearly impossible to ____________ up a meeting because people are all free at different times. A. arrange

B. make

C. set

2. It’s quite out of ____________ for Paul to behave so terribly.

D. take


A. temperament

B. personality

C. nature

D. character

3. She doesn’t like to keep anything hidden: she always ____________ her mind. A. talks

B. speaks

C. tells

D. says

4. Despite all the evidence, he wouldn’t admit that he was in the ____________. A. fault

B. error

C. wrong

D. slip

5. It’s impossible to get ____________ of him! He’s never there! A. feel

B. contact

C. hold

D. touch

6. He ____________ this city down so much that I don’t know why he doesn’t leave. A. holds

B. pulls

C. talks

D. runs

7. I’m afraid Mr. Logan isn’t in the office today, but I can ____________ you through to his secretary. A. put

B. get

C. switch

D. run

8. He is held in high ____________ by everyone who works with him. A. reputation B. respect

C. regard

D. renown

9. Sheila had butterflies in her ____________ before the interview. A. face

B. heart

C. palms

D. stomach

10. ____________ he caught his plane; he hasn’t phoned to say anything went wrong. A. Presumably

B. Supposedly

C. Assuming D. Granted

11. He was ____________ more and more impatient as the time passed. A. showing

B. developing

C. seeming

D. growing

12. I thought I had made it ____________ that I didn’t wish to discuss this matter. A. distinct

B. plain

C. frank

D. straight

13. It really gets me ____________ having to work so much overtime. A. down

B. up

C. on

D. by

14. It was a bad mistake but it had no ____________ on the outcome of the match. A. bearing

B. relevance C. significance

D. repercussion

15. We are looking for self-motivated people who can ____________ the initiative. A. do

B. take

C. get

D. make

16. We took such a great ____________ to the place that we decided to go and live there. A. affection

B. fondness

C. liking

D. attraction


17. A Personnel Manager has to be experienced ____________ dealing with people. A. with

B. for

C. on

D. in

18. She works very hard and finds it difficult to ____________ when she gets home. A. give in

B. switch off C. let out

D. wind up

19. I was worried but their reassurances put my mind at ____________. A. rest

B. comfort

C. calm

D. relief

20. She is highly-skilled ____________ making high-quality furniture. A. to

B. for

C. in

D. on

Your answers: 1.

6.

11.

16.

2.

7.

12.

17.

3.

8.

13.

18.

4.

9.

14.

19.

5.

10.

15.

20.

SECTION 2 The passage below contains 10 errors. Underline the errors and write their correct forms in the space provided in the column on the right. (0) has been done as an example. A feminine0 is a person, usually a woman, who believes that women 0. feminist should be regarded as equally to men. She, or he, deplores

1. ……………….

discrimination against women in the home, place of work or anywhere, and her principle enemy is the male chauvinist, who believes that men

2. ……………….

are naturally super. Tired of being referred to as “the weaker sex”, 3. ………………. women are becoming more and more militancy and are winning the

4. ……………….

age-old battle of the sexes. They are sick to death of sexy jokes which poke fun at women. They are no longer content to be regarded as

5. ……………….

second - class citizens in term of economic, political and social status. 6. ………………. They criticize beauty contests and the use of glamour female models in

7. ……………….


advertisements which they describe as the exploit of female beauty, 8. ………………. since women in these situations were represented as mere sex objects.

9. ……………….

We no longer live in the male – dominate societies of the past. Let us hope, moreover, that the revolution stops before we have a boring world

10……………….

in which sex doesn’t make much difference. We already have unisex hairdressers and fashions. What next? Your answers: 1.

2.

3.

4.

5.

6.

7.

8.

9.

10.

SECTION 3 Complete the following sentences with suitable prepositions or particles. 1. There was tremendous excitement in the streets and the shouting didn’t die _______________ till after midnight. 2. My father’s going to go _____ the wall when he finds out that I’ve lost the car keys. 3. My teacher and I got _____on the wrong foot, but now we are good friends. 4. The book centres _____ a woman who becomes an astronaut. 5. I love watching TV, but it does bite _____ my free time. 6. She left the potatoes cooking for so long that the water boiled _____ and the potatoes were burnt. 7. He thinks I broke his window and threatens to pay me _____ for it. 8. The company has decided to bring _____ a new magazine aimed at the teenage market. 9. I’m passing the chemist on my way home so I’ll pop _____ and see if the photos are ready. 10. The politician was drowned _____ by people shouting and yelling.


Your answers: 1.

6.

2.

7.

3.

8.

4.

9.

5.

10.

SECTION 4 Read the text below. Use the word given in CAPITAL at the end of each line to form a word that fits in the space in the same line. (0) has been done as an example. TV ADVERTISEMENTS In most parts of the world, the (0) economic basis of the television

0. ECONOMY

industry is the (1)____________ of advertising time. This is so

1. SELL

important and (2) ___________ that it funds all the technical and

2. PROFIT

staff costs involved in the (3)___________ of programmes for a

3. PRODUCE

variety of audiences. There is no (4)____________ from that

4. DIFFER

employed by newspapers and magazines when they sell advertising space on their pages, except that (5)__________ have little choice

5. VIEW

but to watch the advertisements, with the added annoyance of (6)____________ during their favourite programmes.

6. INTERRUPT

One (7) ______________ is the BBC which is state funded and so

7. EXCEPT

does not show (8) ____________ . This avoids the situation where

8. COMMERCE

children watching TV(9)______________ influenced by persuasive

9. EASY

advertising for junk food items which their parents may consider 10. HEALTH

(10) ______________ . Your answers: 1.

6.

2.

7.

3.

8.


4.

9.

5.

10.

PART III: READING. SECTION 1 Read the following passage and choose the best answer to fill in each blank. DINOSAUR DISCOVERIES In the late 1930s, a group of primary American scientists seeking dinosaur fossils made some (1)__________ finds, Although one of their expeditions discovered no bones at all, it nonetheless proved to be important in terms of the information about dinosaurs it provided. During that historic expedition, which took place along the banks of the Paluxy river in Texas, something extraordinary was revealed: a dinosaur track, clearly (2)________in the rock. These dinosaur footprints (3)________ their preservation to the salts and mud that covered them and then hardened into rock, before (4) ________ to light 100 million years later. Tracks like these are (5)__________ to experts. There have been great gaps in scientists' understanding of dinosaur (6) _________, and so such footprints are useful since they provide direct evidence of how dinosaurs actually moved. Scientists have been able to use these footprints, and others like them, to determine how quickly different species walked, and to conclude that many kinds of dinosaur must have travelled in (7) _________. (8) _________, the tracks of four-legged dinosaurs seem to (9)________ that in spite of being reptiles, these creatures must have moved in a very similar way to living mammals, such as elephants a pattern of movement distinct from that of most contemporary reptiles, such as crocodiles. This leads to an intriguing question. Might existing mammals have more to teach us about the (10) _________ reptiles that once walked the earth? 1. A. noteworthy

B. noticeable

C. notifiable

D. notional

2. A. blatant

B. substantial

C. distinguishable D. ostensible


3. A. owe

B. derive

C. result

D. thank

4. A. coming

B. bringing

C. appearing

D. surfacing

5. A. unique

B. invaluable

C. costly

D. rare

6. A. action

B. manners

C. behavior

D. customs

7. A. sets

B. herds

C. masses

D. bunches

8. A. Accordingly B. Characteristically

C. interestingly

D. Alternatively

9. A. point

B. specify

C. express

D. indicate

10. A. abolished

B. departed

C. extinct

D. extinguished

Your answers:

1.1.

2. 2.

3. 3.

4. 4.

5. 5.

6. 6.

7. 7.

8. 8.

9. 9.

10.

SECTION 2 Supply each blank with ONE suitable word. ENJOY BENEFITS OF STRESS! Are you looking forward to another busy week? You should be according to some experts. They argue that the stress encountered in our daily lives is not only good for us, but essential to survival. They say that the response to (1) _________, which creates a chemical called adrenal in, helps the mind and body to act quickly (2) ___________ emergencies. Animals and human beings use it to meet the hostile conditions which exist on the planet. Whilst nobody denies the pressures of everyday life, what is surprising is that we are yet to develop successful ways of dealing with them. (3) ________ the experts consider the current strategies to be inadequate and often dangerous. They believe that (4) ________ of trying to manage our response to stress with drugs or relaxation techniques, we must exploit it. Apparently, research shows that people (5) ________ create conditions of stress for (6) _______ by doing exciting and risky sports or looking for challenges, cope much better with life's problems. Activities of this type have been shown to create a lot of emotion; people may actually cry or feel extremely uncomfortable. But there is a point (7) _______ which they realise they have succeeded and know that it was a positive experience. This is because


we learn through challenge and difficulty. That's (8) ________we get our wisdom. Few of us, unfortunately, understand this fact. For example, many people believe they (9) _______ from stress at work, and take time off as a result. Yet it has been found in some companies that by far (10) ________ healthiest people are those with the most responsibility. So next time you're in a stressful situation, just remember that it will be a positive learning experience and could also benefit your health! Your answers: 1.

6.

2.

7.

3.

8.

4.

9.

5.

10.

SECTION 3. Read the passage and choose the best answer for each question. THEORIES OF LIFE ON MARS In the mid-1700s, astronomers observed polar ice caps on Mars. William Herschel saw that the ice caps changed in size with the seasons. The presence of water and seasons, features present on Earth, prompted speculation that life might exist on Mars. Improved telescopes in the nineteenth century further fueled these speculations. They allowed Mars’s surface features to be identified. In 1877, an Italian astronomer, Giovanni Schiapareli, used a 22-centimeter telescope to draw the first map of Mars. His map depicted long lines that he called canali, the Italian word for channels. But his term was mistakenly translated into English as canals. Because channels are usually natural and canals are manmade, the mistake gave birth to imaginative theories of life on Mars over the next 100 years. The leading proponent of the view that the canals were built by intelligent beings was an American astronomer, Percival Lowell. At the Lowell Observatory in the high altitude and clear air of Flagstaff, Arizona, Lowell studied Mars extensively and made detailed


drawings of its surface features. He published books about his Mars studies, including Mars and Its Canals in 1906 and Mars As the Abode of Life in 1908. Lowell proposed that the canals had been constructed by a long-extinct civilization as an elaborate irrigation system. The canals conveyed water from the polar regions to the dry population centers of the planet. Lowell’s ideas were embraced by popular culture. The British novelist H.G. Wells wrote the most famous novel describing life on Mars, War of the Worlds. He imagined that Martians invaded Earth in order to flee the death of their own planet. Bigger and better telescope lenses in the twentieth century failed to confirm that the features Lowell observed really were canals. In fact, they ultimately were shown to be optical illusions. Life on Mars was dealt a further blow beginning in 1965. The United States launched the Mariner spacecrafts to fly close to the surface to take photographs and test the atmosphere. Those probes showed that Mars has a thin atmosphere made mostly of carbon dioxide and that the polar ice caps are frozen dioxide, not water. The photos revealed that Mars had no rivers, oceans, or any other visible signs of life. Scientists concluded that Mars’ thin atmosphere and lack of a magnetic field made it vulnerable to harmful cosmic radiation. Biological and soil experiments were conducted by the Viking project in 1976. To the scientists’ surprise, the test showed that Mars’s surface has no organic matter at all. Thus the present view is that Mars is a dead planet, though there may have been life early in its history that later became extinct. Beginning in 1996, the United States launched a highly successful series of landings on Mars. While they have not found life, they have expanded our knowledge of Martian geology and chemistry. A robotic exploration vehicle in 2001 sent back to Earth spectacular images of Mars’s terrain. Other missions have detected hydrogen and methane. In 2004, Martian vehicles provided conclusive evidence that water existed in the distant past. Additional American missions have been planned through 2009. The European Space Agency wants to land humans on Mars by 2035. And in 2004, the American President declared a national goal of sending astronauts to land on and explore Mars. 1. The word they in the paragraph 2 refers to ….. A. ice caps

B. water and reasons

C. improved telescopes

D. speculations


2. According to paragraph 2, the first map of Mars showed which of the following surface feature? A. mountains

B. craters

C. deserts

D. canals

3. What does the author imply about water on Mars? A. It never existed because Mars has no oxygen. B. It disappeared because of Mars’ thin atmosphere and cosmic radiation. C. It was carried by manmade canals. D. It was used to irrigate the deserts. 4. The word conveyed in paragraph 3 is closest in meaning to ……… A. evaporated

B. collected

C. transported

D. condensed

5. Why does the author mention H.G. Wells’s novel, War of the Worlds? A. To show that another astronomer agreed with Lowell B. To give an example of how Lowell’s view was adopted by popular culture C. To identify a famous novel of the early 1900s D. to prove that there were canals on Mars 6. According to the passage, which of the following is NOT true about Mars? A. It has methane.

B. It had water at one time.

C. It has a thick atmosphere.

D. Its surface has no organic matter.

7. Which of the following can be inferred from paragraph 5 about Earth? A. Its atmosphere is made mostly of carbon dioxide and water. B. Its atmosphere is thinner than Mars’s. C. Its oceans were once frozen carbon dioxide. D. Its magnetic field protects life forms from cosmic radiation.

8 – 9 – 10. Directions: An introductory sentence for a brief summary of the passage is provided below. Complete the summary by selecting the THREE answer choices that express the most important ideas in the passage. Write your answers in the spaces provided. This passage discusses the history of theories of life on Mars. Answer Choices:


A. H.G. Wells wrote War of the Worlds. B. The Italian word for channels is canali. C. Observations of polar ice caps and canals suggested that life existed. D. Recent space probes have proved that life is lacking. E. Improved telescopes showed that there are no canals. F. The Viking project conducted soil surveys on Mars.

Your answers: 1.

2.

3.

4.

5.

6.

7.

8.

9.

10.

SECTION 4 1. Reading the text and match the headings (A – G) with the paragraphs (1-5). Write your answers in the space provided below. There is one heading that you don’t need to use. (0) has been done as an example. A laying the foundation B An inflexible approach C Controversial decisions D A man before his time E A great incentive to achieve F Evidence of a more liberal attitude G Moving with the times 0.____D____ In the latter years of the 19th century, Joseph Pulitzer stood out as the very embodiment of American journalism. Hungarian-born, an intense indomitable figure, Pulitzer was the most skillful of newspaper publishers, a passionate crusader against dishonest government, a fierce, hawk-like competitor who did not shrink from sensationalism in circulation struggles, and a visionary who richly endowed his profession. His innovative New

York

World and St.

Louis

Post-Dispatch reshaped

newspaper

journalism. Pulitzer was the first to call for the training of journalists at the university level


in a school of journalism. And certainly, the lasting influence of the Pulitzer Prizes on journalism, literature, music, and drama is to be attributed to his visionary acumen. 1._______ In writing his 1904 will, Pulitzer left $2 million to Columbia University for the establishment of a School of Journalism. He also made provision for the establishment of the Pulitzer Prizes, specifying the number and types of awards, but giving the body which awarded the prizes, later renamed the Pulitzer Prize Board, the power to increase the number of the prizes and also to withhold any ward where entries fell below its standards of excellence. And indeed, over the years, the board has increased the number of awards to 21, introducing prizes for poetry, music, and photography. 2._______ In 1997, the 150th anniversary of Pulitzer’s birth, two fundamental adjustments were introduced. In recognition of the growing importance of online journalism, the submission by newspapers of online presentations was allowed. The other major change was in music, a category added in 1943. Until then, the prize had always gone to composers of classical music. The Board decided to broaden the definition and entry requirements to bring mainstream music into the process. Thus, in 1997 the prize went to Wynton Marsalis’s Blood on the Fields, which has strong jazz elements. The Board also acknowledged previously overlooked George Gershwin and Duke Ellington. 3._______ Similarly in literature, the board has grown less conservative over the years in matters of taste. In 1963 Edward Albee’s Who’s Afraid of Virginia Woolf? was believed to have too many allusions to sex and too rough dialogue to receive the prize. Yet, in 1993 the prize went to Tony Kushner’s Angels in America: Millennium Approaches, a play that dealt with problems of homosexuality and AIDS and was full of obscenities. 4. _______ Over the years the Pulitzer board has at times been targeted by critics as the prize was hardly ever in tune with popular inclinations. Many, if not almost, of the honored books have not been on bestseller lists, and many of the winning plays have been staged offBroadway or in regional theaters. In journalism, the major newspapers such as The New York Times, The Wall Street Journal, and The Washington Post, have harvested many awards, but small, little known papers have also been recognized for exposing corruption. 5._______ Announced each April by the president of Columbia University, which Pulitzer established as the seat of administration for board, the prizes are regarded as the highest


national honor that can be bestowed in newspaper journalism, literary achievement and musical composition. As such they are perceived as a major encouragement for high quality journalism and art, focusing worldwide attention on American achievements in letters and music. Your answers 0

1

2

3

4

5

D 2. Read the text carefully, and then choose from the sentences A-G the one which fits each gap (1-5). Write your answers in the space provided below. There is one extra sentence which you do not need to use. (0) has been done as an example. A. It avoids wasting time learning the same information twice, and is also a good way of keeping your mind focused on the task. B. This is because they have invented a personal meaning for each one. C. These will be easier to memories than individual digits because there are fewer of them. D. Nevertheless, an improved memory is certainly very valuable to people involved in academic study. E. To recall the names later, you simply follow the same route in your mind. F. The same thing happens with spelling: most people repeat the same mistakes. G. For less obvious names, you'll have to get more creative. Memory isn't like a muscle, something I specific you can exercise. It's a way of organizing information in your brain. So to improve your memory, you need to change and reorganize the way you think and this will help to support how your memory works. Short-term memory is limited, and most people can only remember about seven items (numbers, names, objects, etc.) at a time. So when given a string of numbers to remember such as 123957001969, break it into chunks: 12 39 57 00 19 69 or even 1239 5700 1969. (0)___C___ . The chunks will be even more memorable if you can attach a meaning to some or all of them. For example, you might be able to link them to the age of someone


you know, an address or a famous date (e.g. 1969, the first moon landing). These linked meanings can then form a story to help remember a really long sequence. Another technique for memorizing information relies on images. A classic way of remembering a person's name is to try and imagine it (or something associated to it) on the person's face. This is easy if you meet John Bridge: just imagine a bridge on his face. (1) _______ Psychologists have found that the more unusual and vivid the image is, the better it works. In order to improve your ability to memorize random information, look for meaning in everything - especially if you can refer it back to yourself. The human mind has a particular fondness for meaning. A sequence of playing cards is difficult to remember because the cards are essentially meaningless. And yet, some people have trained themselves to memorize the precise order of every card in a pack, and it only takes them a few minutes to do it. (2) _______ Make a mistake whilst learning something for the first time and you're more likely to make the same mistake again. But get it right from the start and it'll stick. This is called error-free learning. For instance, if you cycle to somebody's house for the first time and you take a wrong turning, it is quite common that you will repeat that mistake next time you go. This is because, when you 40 repeat the journey, you recognise the route and the landmarks, and your brain will carry on misguiding you until you realise that you are just remembering the things you did wrong last time! It's not that your memory is poor but that it's misleading you. (3) _______. So if you're learning something new, cut out all distractions; this will help you learn it correctly first time. In general, it is often better to test yourself on something you've learned than to keep re-learning it. This is because while testing yourself, you can reflect on your progress, check how well you have learnt things, and fill in the gaps rather than re-learn everything again. (4) __________. This applies, for example, when you are learning vocabulary in a foreign language. And finally, there is an excellent technique for remembering facts, called the 'Roman Room' method. It's a particularly good way of remembering a sequence of related information such as a list of names, and it works like this. Choose a place you know well - such as your home - and take a mental walk through the rooms. Then, put the


names from your list one by one into the rooms. Suppose you want to remember the names of your friend's brothers and sisters in order of age. Visualize Harry, the eldest, in your front room, then Sally the second in the back room. Molly, the third oldest, is in the kitchen ... and so on. (5) __________ It seems a strange way of remembering, but with practice it is very successful. It works because you are attaching new information to something which is already very familiar and easy to recall. Your answers: 0

1

2

3

4

5

C SECTION IV: WRITING. SECTION 1 Rewrite the following sentences, keeping their meaning unchanged, beginning with the words given. 1. Immediately after his arrival, things went wrong.

No sooner ……………………………………………………………………………. 2. It was only when I left home that I realized how much my father meant to me.

Not until ………………………………………………………………………………. 3. I dislike it when people criticize me unfairly.

I object ………………………………………………………………………………… 4. My shoes need to be cleaned before the interview.

I must …………………………………………………………………………………. 5. Our present financial problems will soon be a thing in the past.

We ……………………………………………………………………………………. SECTION 2 Complete the following sentences so that the second sentence has a similar


meaning to the first sentence. Do not change the word given. You must use between two and five words, including the words given. 1. On business trips, I prefer driving home to staying in a hotel overnight.

rather

On business trips, I’d _______________________________ in a hotel overnight. 2. Jack found it difficult to control his skis on the steep slope.

under

Jack found it difficult to________________________________ on the steep slope. 3. My views on this subject are exactly the same as yours.

difference

There _________________________________ my views on this subject and yours. 4. In my opinion, these two kinds of music are completely different from each other. comparison

In my opinion, there is _____________________________these two kinds of music. 5. I arrived late because I missed the 10.30 train

turned

If I’d caught the 10.30 train, I _______________________________________time. SECTION 3 ESSAY WRITING: Some people say that uniforms produces unity at schools but others claim that wearing uniform at schools limits freedom of expression. Should uniforms be compulsory at schools? Write an essay of about 200 - 250 words to express your viewpoint. THE END -------------------------------------------------------------------------------------------

Người ra đề: Hà Huy Khánh Số ĐT: 0912453903


SƠN LA TRƯỜNG THPT CHUYÊN S

HƯỚNG DẪN CHẤM ĐỀ ĐỀ XUẤT ĐỀ THI HSG KHU VỰC NĂ 2019 LẦN THỨ XII - NĂM MÔN TIẾNG ANH - LỚP 10 Thời gian làm bài: 180 phút.

________________________________________________________________________ PART I: LISTENING. (50 points) SECTION 1 (10 points) Questions 1-5: Complete the form with missing information. 1. Black

2. 2085

3. 9456 1309

4. 2020BD

5. July

SECTION 2 (10 points) Questions 6-10: You will hear an interview with Pamela Green, a young fashion designer. For questions 6 - 10, choose the best answer (A, B or C). 6. B

7. B

8. C

9. B

10. A

SECTION 3 (10 points) Questions 11 - 15: You will hear a man called Steve and a woman called Caroline

talking about summer jobs. Decide if each sentence is correct or incorrect. If it is correct, put a tick ( ) in Yes column. If it is not correct, put a tick ( ) in No column. YES 1. Steve hasn’t arranged any work for the summer yet.

2. Caroline’s work will allow her to have free time during the day.

NO

3. Caroline’s work will be located in a city.

4. Caroline found out about the job from the internet.

5. Caroline says that work at music festivals is badly paid.

SECTION 4 (20 points) Questions 16 - 25: You are going to hear somebody giving their opinion about

media and its influence on society. For questions 16-25, complete the sentences with NO MORE THAN THREE WORDS.


16. newspapers

17. campaigns

18. public eye

19. 4% / four percent

20. TV

21. images

22. fiction

23. one million pounds

24. apology

25. tabloids

PART II: LEXICO - GRAMMAR. (50 points) SECTION 1 (20 points) Choose the word or phrase which best completes each sentence. 1. C

2. D

3. B

4. C

5. C

6. D

7. A

8. C

9. D

10. A

11. D 12. B 13. A 14. A 15. B 16. C 17. D 18. B 19. A 20. C SECTION 2 (10 points) The passage below contains 10 errors. Underline the errors and write their correct forms in the space provided in the column on the right. (0) has been done as an example. 1. equally equal

2. principle principal

3. super superior

4. militancy militant

5. sexy sexist

6. glamour glamorous

7. exploit exploitation 8. were are

9. male - dominate male - dominated

10. moreover however SECTION 3 (10 points) Complete the following sentences with suitable prepositions or particles. 1. down

6. away

2. up

7. back

3. off

8. out

4. around

9. in

5. into

10. out


SECTION 4 (10 points) Read the text below. Use the word given in CAPITAL at the end of each line to form a word that fits in the space in the same line. (0) has been done as an example.

1. sale

2. profitable

6. interruption

3. production

7. exception 8. commercials

4. different 9. easily

5. viewers 10. unhealthily

PART III: READING. (50 points) SECTION 1 (10 points) Read the following passage and choose the best answer to fill in each blank. DINOSAUR DISCOVERIES 1. A

2.C

3.A

4.A

5.B

6.C

7.B

8.C

9.D

10.C

SECTION 2 (15 points) Supply each blank with ONE suitable word. ENJOY BENEFITS OF STRESS! 1. stress

2. in / during

3. Even

4. instead

5. who / that

6. themselves

7. at

8. how

9. suffer

10. the

SECTION 3. (10 points) Read the passage and choose the best answer for each question. THEORIES OF LIFE ON MARS 1. C

2. D

3. B

4. C

5. B

6. C

7. D

8. C

9. D

10.E

SECTION 4 (15 points) 1. Reading the text and match the headings (A – G) with the paragraphs (1-5). Write your answers in the space provided below. There is one heading that you don’t need to use. (0) has been done as an example. 1.A

2.G

3.F

4.C

5.E


2. Read the text carefully, and then choose from the sentences A-G the one which fits each gap (1-5). Write your answers in the space provided below. There is one extra sentence which you do not need to use. (0) has been done as an example. 1. G

2.B

3. F

4. A

5. E

SECTION IV: WRITING. SECTION 1 (10 points) Rewrite the following sentences, keeping their meaning unchanged, beginning with the words given. 1. No sooner had he arrived than things went wrong. 2. Not until I left home did I realize how much my father meant to me. 3. I object to people criticizing on me unfairly. 4. I must have (get) my shoes cleaned before the interview. 5. We shall soon find a solution to our present financial problems. SECTION 2 (10 points) Complete the following sentences so that the second sentence has a similar meaning to the first sentence. Do not change the word given. You must use between two and five words, including the words given. 1. rather drive home than stay 2. keep/ get/ have his skis under control 3. is no difference between/ is not any difference between 4. no (possible) comparison (at all) between 5. would have turned up on/ in time SECTION 3 (30 points) ESSAY WRITING:

1. Completion: 4 points 2. Content: 8 points Provide relevant and convincing ideas about the topic, supported by specific example and/or reasonable justification. 3. Organization: 8 points


- Ideas are well organized and presented with unity, cohesion and coherence. 4. Language: 7 points - Demonstrate of a wide range of vocabulary and structures. - Good use of grammatical structures. - Present the ideas with clarity. 5. Punctuations and spelling: 3 points


TRANSCRIPTS SECTION 1. LOUISE: Oh hello, I'd like to join the video library. MR MAX: OK. Would you like to fill in the application form now? LOUISE: Yes, I can do it now. MR MAX: Hold on and I'll get a form. Now, I'll just ask you a few questions and then I'll get you to sign at the bottom. LOUISE: Right. MR MAX: What's your full name? LOUISE: Louise Cynthia Jones. (Example) MR MAX: Jones? LOUISE: Yes, that's right. (Repeat) MR MAX: OK, and what's your address? LOUISE: Apartment 1, 72 Black Street, Highbridge. MR MAX: Black Street, that's just around the corner, isn't it? LOUISE: Yes. MR MAX: OK, so the post code is 2085, right? LOUISE: Yes, 2085. MR MAX: Mm. And your telephone number? I need both home and work. LOUISE: Home is 9835 6712 and work is 9456 1309. Do you need any ID or anything like that? MR MAX: Yes, we need your driver's licence number, that is if you have one. LOUISE: Yes, 1 know if off by heart, it's an easy one, 2020BD. Do you need to see it? MR MAX: Yes, I'm afraid I do. LOUISE: Mm . . . here. MR MAX: Right, thanks. And could you tell me your date of birth, please? LOUISE: 25 July 1977. MR MAX: That's the most important part out of the way, but could I just ask you a few questions for a survey we're conducting? LOUISE: OK. SECTION 2: Interviewer: Hello, Pamela, welcome to the programme. So many young people want to be fashion designers these days, but don't know how to get started.


Pamela Green: Hi. I felt exactly like that myself! You must first discover if this is really what you want to do. I wasn't sure to begin with, so I started off by looking for a store in my neighbourhood that sold its own clothes. The owner invited me into her studio and told me what a typical day was like. She allowed me to ask as many questions as I wanted. Having made up my mind, I then contacted a few colleges to see what courses in fashion were on offer, and I was lucky to find one that seemed ideal. Interviewer: So a degree in fashion is a must? Pamela Green: Well, you often meet designers who go to college later in life, after years of working in the industry. The truth is the best students aren't always the best designers, but there's no denying that a degree will show that you've got certain basic skills and get you your first job. Don't be surprised to find colleagues with fewer qualifications on higher pay than yourself though. Making progress from that point will depend entirely on your personal talent. Interviewer: What basic skills do you need? Pamela Green: When you ask a fashion student what they want to do, they often reply 'have my own line'. Not an easy task, I must say. You need work experience first, ideally in a successful fashion shop, to understand that this industry is led by commerce. Starting your own line requires capital and a clear overview of how it's going to develop. Without it, clothes design can only be a hobby. Of course, if you've got an eye for colour, style and shape and an ability to draw, you shouldn't let go of the dream. Interviewer: Where do you get the inspiration for your designs? Pamela Green: To be a good designer, you have to be aware of the world you live in, you need to go out and look at people's lives and attitudes, you really have to learn how to observe what's happening around you. And I don't mean going abroad necessarily - my social circle is invaluable for me, for example - a constant source of ideas. You have to remember the clothes are not for you, you have to adapt to what other people want. And don't be tempted to imitate the famous designers, however beautiful their collections might look. Interviewer: Now you're a successful designer, are things easier? Pamela Green: It took me a while to learn to cope with criticism, though. You think your design drawings look brilliant, but you mustn't get upset if the garment doesn't look as you'd imagined it. What I've never managed to get used to is the sheer amount of work involved in finishing your collection well in advance of the season. Some designers stop attending


fashion shows, for example, which involve lots of time-consuming travelling, but I'd be unhappy to give that up. Interviewer: Do you have to do a lot of reading to keep up with trends? Pamela Green: You have to read fashion magazines and other media that reflect current trends and tastes. It doesn't matter whether you want to use them in your own designs. Nobody knows what styles will be fashionable in, say, two years' time, but the point is you have to know just about everything that's been done before, so that you can spot it when it becomes popular again. Interviewer: This is a very competitive industry. Realistically, what are the chances for somebody starting? Pamela Green: Don't make the mistake of aiming just for designing outfits, which is just one part of a vast industry. You may be perfectly happy as an obscure but competent designer of small pieces for collections - jewellery, hats, shoes - all of which need to be created. And then somebody has to market them, sell them, write about them. Fame and glory is just for the top twenty world designers, and life isn't always wonderful even for them. Interviewer: Pamela, many thanks. SECTION 3. Caroline:So, what are you going to do this summer? Steve:

Well, I’m probably going to go on holiday with my parents and my sister for a week, but I suppose I really should get a summer job. I haven’t planned anything yet, but I really need to get some money. What about you?

Caroline:I’m going to Scotland for the summer. Steve:

For the whole summer?

Caroline:Yes, well, I’m going for seven weeks. I’m going to work in a youth hostel in the mountains. Steve:

Really? That sounds great!

Caroline:Yes, I think so. I mean, the work won’t be very interesting. It’ll be mostly cooking, cleaning and working on reception, selling drinks and things. But my hours are seven till ten in the morning and five till nine in the evening, so I’ll have plenty of time during the day to go walking.


Steve:

That sounds fun! It’ll be like a holiday, but you’ll be earning money too.

Caroline:Yes, it’s a chance to get out of the city and maybe meet some new people. Steve:

I wish I could get a job like that. How did you find out about it?

Caroline:I read about it in a book called Summer Work Round Britain. It tells you about all kinds of different jobs in different places. You can work at farms picking fruit, or at hotels near the sea. You can even get work at music festivals. Steve:

Oh I like the sound of that!

Caroline:Yes, of course the problem with that sort of work is that it only lasts a few days. And once you’ve paid for your accommodation and food, you won’t have much money left. Steve:

Oh yes, I didn’t think about that. And knowing me, I’d spend all my extra money on music. So do you have to pay for your accommodation and food in Scotland then?

Caroline:Well, it’s quite good. I’ll be staying in a shared bedroom, and they’ll take £15 a week out of my wages to pay for that. My breakfast will be free, because I’ll probably be cooking breakfast for the guests! I can get dinner in the evening at a reduced price, so I only have to worry about lunch. Steve:

Well, I hope you have a great time.

Caroline:Thanks. I’ll lend you the book if you like. Maybe you’ll find something too. Steve:

Isn’t it too late to apply?

Caroline:I don’t think so. Anyway, it’s worth a try.


SECTION 4.


KÌ THI HỌC SINH GIỎI NĂM HỌC 2018-2019

HỘI CÁC TRƯỜNG THPT CHUYÊN VÙNG DUYÊN HẢI VÀ ĐỒNG BẰNG BẮC BỘ

MÔN THI: TIẾNG ANH LỚP 10 TRƯỜNG THPT CHUYÊN THÁI NGUYÊN ĐỀ THI ĐỀ XUẤT

Ngàythi: …….. Thờigianlàmbài: 180 phút (khôngkểthờigiangiaođề) (Đềthigồm 16trang)

A. SECTION I: LISTENING (50 points) HƯỚNG DẪN PHẦN THI NGHE HIỂU • Bàinghegồm 4 phần, mỗiphầnđượcnghe 2 lần. • Mởđầuvàkếtthúcphầnnghecótínhiệunhạc. • Mọihướngdẫnchothísinh (bằngtiếngAnh) đãcótrongbàinghe. Part 1.( 10 points) Complete the notes below. Write ONE WORD for each answer. TRANSPORT SURVEY Travelled to town today: by bus Name: Luisa (1) ………………………………….. Address: (2) ………………………. White Stone Road Area: Bradfield Postcode: (3) ………………………………………….. Occupation: (4) ……………………………………….. Reason for visit to town: to go to the (5) …………….. Your answers: 1. 2. 3. 4. Part 2.( 10 points) Listen and choose the correct letter A, B, or C. Theatre Studies Course 1. What helped Rob to prepare to play the character of a doctor? A. the stories his grandfather told him B. the times when he watched his grandfather working C. the way he imagined his grandfather at work

5.


2. In the play’s first scene, the boredom of village life was suggested by A. repetition of words and phrases B. scenery painted in dull colours C. long pauses within conversations 3. What has Rob learned about himself through working in a group? A. He likes to have clear guidelines. B. He copes well with stress. C. He thinks he is a good leader. 4. To support the production, research material was used which described A. political developments B. changing social attitudes C. economic transformations 5. What problems did the students overcome in the final rehearsal? A. one person forgetting their words B. an equipment failure C. the injury of one character Your answers: 1. 2. 3. 4.

5.

Part 3.( 10 points) You will listen to a short talk about the entertainment program and decide whether the following statements are true or false. 1. Sally cannot reveal the name of the band for the first party. 2. There will be a concert in the Cotswold Theatre on Monday. 3. There is an important event on Thursday. 4. There is no formal dress code for the Freshers’ Ball. 5. Students should check the notice board for changes to the announced program. Your answers: 1. 2. 3. 4. 5.


Part 4.( 20 points) You will hear a radio programme about the history of roller skating. For questions 1- 10, complete the sentences using NO MORE THAN THREE WORDS. History of roller skating The country where the first roller skates were probably made was (1) __________ In 1760, John Merlin went to a ball in London playing a (2) _________________ whilst on roller skates. Unfortunately, John Merlin injured himself when he broke a (3) ______________ at the ball. In Germany, roller skating was used in a ballet called (4) __________________James Plimpton’s invention helped roller skaters to control the (5) ___________ of their skates. The first team sport to be played on roller skates was (6) _________________ In Detroit in 1937, the first (7) __________________________ in the sport took place. The use of plastics meant that both the (8) ______________ and _____________ of roller skates improved. The musical “Starlight Express” was seen by as many as (9) _______________ in London. The speaker says that modern roller skates are now (10) ______ and ______ than ever before. Your answers: 1.

6.

2.

7.

3.

8.

4.

9.

5.

10.

B. SECTION TWO: LEXICO-GRAMMAR (50 points) Part 1.Choose the word or phrase (A, B, C or D) which best completes each sentence.(20 pts) 1. My brother found it impossible to ______his anger and started shouting. A. restrict B. inhibit C. reserve D. contain 2. We don’t have a secretary ___, but we do have a student who comes in to do a bit of filing. A. as such B. the least bit C. whatsoever D. little more 3. The problem _______because neither side was prepared to compromise. A. amassed B. escalated C. proliferated D. enhanced 4. A meeting was held so that staff could _______their view freely. A. divulge B. vent C. utter D. air 5. This evidence should prove ___ that he was telling the truth. A. once and for all B. now and then C. over and above D. from time to time


6. Not even losing all the time could _______his enthusiasm for tennis. A. recede B. dampen C. erode

D. belittle

7. It was a _________lie and nobody at all believed it. A. blatant B. salient C. conspicuous D. pronounced 8. You’ll just have to ______yourself to the fact that you can’t always have what you want. A. acknowledge B. reconcile C. concede D. allow 9. We were under no _____ about how difficult it would be to achieve our aims. A. fantasies B. daydreams C. illusions D. deceptions 10. I’m afraid that the facts don’t ______your theory. A. pull off B. check out C. show off D. bear out 11 You'd better pack those glasses extremely carefully if you want them to arrive ______. A. entire B. intact C. whole D. complete 12. We are not in a ______ hurry so let’s have another coffee. A. dashing B. racing C. rushing D. tearing 13. It was a bad mistake but it had no ______on the outcome of the match. A. bearing B. relevance C. significance D. repercussion 14. Time was running out, so the committee had to make a ____________ decision. A. brief B. snap C. sharp D. curt 15. The island was quite content with its _______ prosperity. A. heinous B. burgeoning C. insatiable D. abortive 16. He ________at the last minute and withdrew from the competition. A. put his foot down B. got up his nose C. couldn’t keep a straight face D. got cold feet 17. George wasn’t in a particularly good mood, but then he is always a bit ______in the morning. A. irate B. crotchety C. indignant D. infuriated 18. We were alerted to the fire when we saw thick, heavy smoke _____out of the window. A. clouding B. billowing C. choking D. pushing 19. I must take this watch to be repaired; it ____________ over twenty minutes a day. A. increases B. progresses C. accelerates D. gains 20. Why don’t you have a night out? It would take your ______ off your worries. A. thoughts B. heart C. mind D. head Part 2.The following passage contains 10 errors. Identify and correct them.


1

An environmental history of mankind would have to be a history of the exploitation of abundant natural resources, the spiral demand for these

5

resources, and their inevitable depletion. As humanity spread over the globe, leaving colonies in their wake, essential resources such as coal, oil, and even fresh water were extracted through industrial mining and manufacturing operations that had a massive impact on the earth itself. That was once a plethora of riches has inexorably declined, and entrepreneurs are now looking into the limits of land, sea, and sky; the new target is the stars and, more especially, asteroids. Spinning around the Sun are tens of thousands of

10

15

20

asteroids, and scientists have convinced that these mountain-like formations contain a treasure trove of minerals and metals. The asteroid 16 Psyche has enough iron-nickel ore to sustain the Earth for several million years. Even a comparatively small asteroid could contain more than 2,000 million metric tons of serviceable mineral-metal reserves. In addition, some asteroids have a high ice content which means that they could be an economically viable source of fresh water. Large –scale mining of asteroids is possibly the key to solve many of our escalating environmental problems. Any mining venture contains an element of risk, however. Asteroids have traditionally been considered dead perils lurking out in the dark depths of space, bided their time before smashing headlong into our insignificant planet. In order to transplant mining operations from the Earth to the stars, we need to find solutions to extensive safety and logistical problems.

1. _______________________ 2. _______________________ 3. _______________________ 4. _______________________ 5. _______________________

6. _______________________ 7. _______________________ 8. _______________________ 9. _______________________ 10. ______________________

Part 3. Fill in each blank with a suitable preposition or particle. (10 points) 1. Josie banked ________the fire to last till morning. 2. The education expenses of the children are eating _______ their savings, but they do not mind at all.


3. The school has certainly lived _______ its reputation as one which produces top scholars year after year. 4. After some careful consideration, James decided to turn _______ the post which carried heavy responsibilities. 5.Looking_________, we must expect radical changes to be made in our system of government. 6. My wife backed me ______ over my decision to quit my job. 7. Traditionally, Amish weddings are held on Tuesdays and Thursdays, so there is time in _______ to get ready for and clean up after each. 8. According to a government study, Korean elderly women are much more vulnerable ________ social isolation and illnesses compared to their male counterparts. 9. The practice of acupuncture is rooted ________ the idea of promoting harmony between humans and the world around them and a balance between yin and yang. 10. I never doubt that this candidate can pull ______ a victory. III. Complete each sentence with one suitable particle or preposition. Write your answer in the box provided. (10 points) 1. My wife backed me ______ over my decision to quit my job. 2. My mum and dad always find a way to work well together and are not critical ________ each other. 3. At first, residents opposed plans for a new city skate park, but the skaters eventually won them _______. 4. Traditionally, Amish weddings are held on Tuesdays and Thursdays, so there is time in ________ to get ready for and clean up after each. 5. According to a government study, Korean elderly women are much more vulnerable ________ social isolation and illnesses compared to their male counterparts. 6. What is there so remarkable of him that people will hang ________ his every word? 7. The Minister was shouted ______ by hecklers and was unable to finish her speech. 8. Do you have to go already? Can’t you stick ________ for a few minutes? 9. The practice of acupuncture is rooted ________ the idea of promoting harmony between humans and the world around them and a balance between yin and yang. 10. I never doubt that this candidate can pull ______ a victory. Answers:


1.

2.

3.

4.

5.

6.

7.

8.

9.

10.

Part 4:Write the correct form of the words given in the brackets. Write your answers in the spaces provided below. (10 points) The Media Commentators A live broadcast of any public event, such as a space flight or sporting occasion, is almost (1) ________ accompanied by the thoughts of a 1. VARY commentator. This may be on television, along with the relevant pictures, or (2) ________ on radio. The technique involved differs between the two 2. ALTER media, with radio broadcasters needing to be more explicit and (3) ________. Because of the (4) _______ of visual information. TV 3. DESCRIBE commentators do not need to paint a picture for their audience; instead their 4. ABSENT various observations should add to the images that are already there. There will sometimes be silences and pauses in TV (5) ________, although these are becoming increasingly rare. Both types of commentators should try to 5.COMMENT be more informative, but should avoid sounding (6) _______. In sports ones, fairness and (7)_______ to both sides is vital, but spontaneity and 6. OPINION (8)________ are valued by those watching or listening. Sports 7. PART commentators usually broadcast live in an essentially unscripted way, 8. ENTHUSE although they may refer to previously prepared materials such as sports statistics. Because of the (9) ________ nature of live events, thorough preparation in advance is vital. The internet has helped enormously with 9. PREDICT this aspect of the job. Anyone interested in becoming a commentator should have excellent (10)________ skills, the willingness to work irregular hours and a strong voice. 10.ORGANIZE Answers: 1.

2.

3.

4.

5.

6.

7.

8.

9.

10.

C. SECTION III: READING (50 points)


Part 1. For questions 1- 10, read the passage and choose the best answer A, B, C or D (10 points) Read the text below and decide which answer best fits each space. Greenhouse gases are being released into the atmosphere 30 times faster than the time when the Earth experienced a (1)______ episode of global warming. A study comparing the rate at which carbon dioxide and methane are being (2)______ now, compared to 55 million years ago when global warming also occurred, has found dramatic differences in the speed of release. James Zachos, professor of earth sciences at the University of California, Santa Cruz, said the speed of the present build-up of greenhouse gases is far greater than during the global warming after the (3)______ of the dinosaurs. "The emissions that caused this past episode of global warming probably lasted 10,000 years," Professor Zachos told the American Association for the Advancement of Science at a meeting in St. Louis. "By burning fossil fuels, we are likely to emit the same amount over the next three centuries." He warned that studies of global warming events in the geological past (4)__________the Earth's climate passes a (5)__________beyond which climate change accelerates with the help of positive feedbacks - vicious circles of warming. professor Zachos is a leading (6)__________on the episode of global warming known as the palaeocene-eocene thermal maximum, when average global temperatures increased by up to 5oC due to a massive release of carbon dioxide and methane. His research into the deep ocean (7)__________suggests at this time that about 4.5 billion tons of carbon entered the atmosphere over 10,000 years. "This will be the same amount of carbon released into the atmosphere from cars and industrial emissions over the next 300 years if present (8)__________continue", he said. Although carbon can be released suddenly and naturally into the atmosphere from volcanic activity, it takes many thousands of years for it to be removed permanently by natural processes. The ocean is capable of removing carbon, and quickly, but this natural (9)__________can be easily (10)__________which is probably what happened 55 million years ago. "It will take tens of thousands of years before atmospheric carbon dioxide comes down to preindustrial levels," the professor said. "Even after humans stop burning fossil fuels, the effects will be long-lasting." 1. A. prearranged B. premier C. previous D. fundamental 2. A. emitted B. exhaled C. incorporated D. digested 3. A. dementia B. demolition C. detachment D. demise 4. A. comment B. mark C. compliment D. indicate 5. A. barricade B. verge C. threshold D. perimeter


6. A. autocrat 7. A. dusts

B. authority B. sediments

C. administrator C. dirt

D. proprietor D. powder

8. A. trends

B. gadgets

C. fads

D. crazes

9. A. capacity 10. A. overcharged

B. competence B. overstated

C. intelligence C. overshadowed

D. bulk D. overwhelmed

Answers: 1.

2.

3.

4.

5.

6.

7.

8.

9.

10.

Part 2: For questions 1 - 10, fill each of the following numbered blanks with ONE suitable word. Write your answers in the corresponding numbered boxes provided. (15 points) MALNUTRITION GOALS IN AFRICA “VERY FAR AWAY� A new report shows that no country in Africa will meet goals (1)______ to end childhood malnutrition by the year 2030. That target was set by the United Nations in 2015 (2)______ a Sustainable Development Goal. The UN adopted a set of goals, "to end poverty, protect the planet and ensure prosperity for all as part of a new sustainable development agenda". The new report is published in the journal Nature. It identifies poor child nutrition and (3)_______ levels of education across 51 African countries. These were (4)______ factors in countries battling to (5)_______ children with sufficient food. Researcher Simon Hay said the goal of ending childhood malnutrition was always an "aspirational" target. He said: "This aspiration is very, very far away." There was some good news in the report. It highlighted the fact that many African nations, (6)_______ Ghana and Nigeria, have shown (7)______ of improvement in childhood development since the year 2000. However, it is a different story for countries like Chad, Central African Republic and Eritrea. The report indicates that malnutrition remained "persistently high" in 14 countries between Senegal in the west and Somalia in the east. Many of these countries have (8)_______ war, famine and mass migration, all of (9)____ have put massive strains (10)_______ health and agriculture. One researcher said considerable investment was needed in health and infrastructure in order to address "serious inequalities". Part 3: Read the passage and choose the best answer to each of the questions ( 10 points) HISTORY OF THE CHICKENPOX VACCINE


Chickenpox is a highly contagious infectious disease caused by the Varicellazoster virus; sufferers develop a fleeting itchy rash that can spread throughout the body. The disease can last for up to 14 days and can occur in both children and adults, though the young are particularly vulnerable. Individuals infected with chickenpox can expect to experience a high but tolerable level of discomfort and a fever as the disease works its way through the system. The ailment was once considered to be a “rite of passage� by parents in the U.S. and thought to provide children with greater and improved immunity to other forms of sickness later in life. This view, however, was altered after additional research by scientists demonstrated unexpected dangers associated with the virus. Over time, the fruits of this research have transformed attitudes toward the disease and the utility of seeking preemptive measures against it. A vaccine against chickenpox was originally invented by Michiaki Takahashi, a Japanese doctor and research scientist, in the mid-1960s. Dr. Takahashi began his work to isolate and grow the virus in 1965 and in 1972 began clinical trials with a live but weakened form of the virus that caused the human body to create antibodies. Japan and several other countries began widespread chickenpox vaccination programs in 1974. However, it took over 20 years for the chickenpox vaccine to be approved by the U.S. Food & Drug Administration (FDA), finally earning the U.S. government’s seal of approval for widespread use in 1995. Yet even though the chickenpox vaccine was available and recommended by the FDA, parents did not immediately choose to vaccinate their children against this disease. Mothers and fathers typically cited the notion that chickenpox did not constitute a serious enough disease against which a person needed to be vaccinated. Strong belief in that view eroded when scientists discovered the link between Varicella zoster, the virus that causes chickenpox, and shingles, a far more serious, harmful, and longerlasting disease in older adults that impacts the nervous system. They reached the conclusion that Varicella zoster remains dormant inside the body, making it significantly more likely for someone to develop shingles. As a result, the medical community in the U.S. encouraged the development, adoption, and use of a vaccine against chickenpox to the public. Although the appearance of chickenpox and shingles within one person can be many years apart - generally many decades - the increased risk in developing shingles as a younger adult (30-40 years old rather than 60-70 years old) proved to be enough to convince the medical community that immunization should be preferred to the traditional alternative. Another reason that the chickenpox vaccine was not immediately accepted and used by parents in the U.S. centered on observations made by scientists that the vaccine simply did not last


long enough and did not confer a lifetime of immunity. In other words, scientists considered the benefits of the vaccine to be temporary when given to young children. They also feared that it increased the odds that a person could become infected with chickenpox later as a young adult, when the rash is more painful and prevalent and can last up to three or four weeks. Hence, allowing young children to develop chickenpox rather than take a vaccine against it was believed to be the “lesser of two evils.” This idea changed over time as booster shots of the vaccine elongated immunity and countered the perceived limits on the strength of the vaccine itself. Today, use of the chickenpox vaccine is common throughout the world. Pediatricians suggest an initial vaccination shot after a child turns one year old, with booster shots recommended after the child turns eight. The vaccine is estimated to be up to 90% effective and has reduced worldwide cases of chickenpox infection to 400,000 cases per year from over 4,000,000 cases before vaccination became widespread. ■ (A) In light of such statistics, most doctors insist that the potential risks of developing shingles outweigh the benefits of avoiding rare complications associated with inoculations. ■ (B) Of course, many parents continue to think of the disease as an innocuous ailment, refusing to take preemptive steps against it. ■ (C) As increasing numbers of students are vaccinated and the virus becomes increasingly rarer, however, even this trend among parents has failed to halt the decline of chickenpox among the most vulnerable populations. ■ (D). 1. The word “tolerable” in the passage 1 is closest in meaning to A. sudden B. bearable C. infrequent D. unexpected 2. According to paragraph 1, which of the following is true of the chickenpox virus? A. It leads to a potentially deadly disease in adults. B. It is associated with a possibly permanent rash. C. It is easily transmittable by an infected individual. D. It has been virtually eradicated in the modern world. 3. Which of the following best expresses the essential information in the highlighted sentence? Incorrect answer choices change the meaning in important ways or leave out essential information. A. U.S. parents believed that having chickenpox benefited their children. B. U.S. parents believed that chickenpox led to immunity against most sickness. C. U.S. parents wanted to make sure that their children developed chickenpox. D. U.S. parents did not think that other vaccinations were needed after chickenpox. 4. Which of the following can be inferred from paragraph 2 about the clinical trials for the chickenpox vaccine?


A. They took longer than expected. C. They took a long time to finish.

B. They cost a lot of money to complete. D. They were ultimately successful.

5. The word “notion” in the passage 2 is closest in meaning to A. history B. findings C. fact D. belief 6. According to paragraph 3, which of the following is true of Varicella Zoster? A. It typically attacks adults who are over 60 years old. B. It is linked to a serious disease that occurs more commonly in adults. C. It likely is not a serious enough threat to human health to require a vaccine. D. It is completely eradicated from the body after chickenpox occurs. 7. According to paragraph 3, all of the following is true about the chickenpox virus EXCEPT: A. It causes two distinct yet related ailments. B. People did not view it as a serious public health threat. C. It tended to quickly become dormant and remain inoperative over time. D. Vaccination against it would help prevent the onset of shingles. 8. The author uses boostershots as an example of A. a way to increase the effectiveness of the chickenpox vaccine B. a preferred method of chickenpox rash and fever treatment C. a scientifically approved medicine to eliminate chickenpox D. a strategy for parents to avoid vaccinating their child altogether 9. According to paragraph 4, many parents did not choose the chickenpox vaccine because A. they believed that the virus was weak and not especially harmful B. they thought that scientists did not have enough data to reach a conclusion C. they were unsure about the utility of the vaccine given its expected duration D. they were convinced it was potentially very toxic, particularly for older children 10. Look at the four squares [▪] that indicate where the following sentence could be added to the passage. Meanwhile, some continue to remain unconvinced, citing a supposed potential of the vaccine to do harm. Where would the sentence fit best? A. A B. B C. C D. D Answers: 1.

2.

3.

4.

5.


6.

7.

8.

9.

10.

Part 4: Read the following passage then do the tasks that follow. (15 points) HOW DOES THE BIOLOGICAL CLOCK TICK? A Our life span is restricted. Everyone accepts this as 'biologically' obvious. ‘Nothing lives for ever!’ However, in this statement we think of artificially produced, technical objects, products which are subjected to natural wear and tear during use. This leads to the result that at some time or other the object stops working and is unusable ('death' in the biological sense). But are the wear and tear and loss of function of technical objects and the death of living organisms really similar or comparable? B Our ‘dead’ products are ‘static’, closed systems. It is always the basic material which constitutes the object and which, in the natural course of things, is worn down and becomes 'older’. Ageing in this case must occur according to the laws of physical chemistry and of thermodynamics. Although the same law holds for a living organism, the result of this law is not inexorable in the same way. At least as long as a biological system has the ability to renew itself it could actually become older without ageing; an organism is an open, dynamic system through which new material continuously flows. Destruction of old material and formation of new material are thus in permanent dynamic equilibrium. The material of which the organism is formed changes continuously. Thus our bodies continuously exchange old substance for new, just like a spring which more or less maintains its form and movement, but in which the water molecules are always different. C Thus ageing and death should not be seen as inevitable, particularly as the organism possesses many mechanisms for repair. It is not, in principle, necessary for a biological system to age and die. Nevertheless, a restricted life span, ageing, and then death are basic characteristics of life. The reason for this is easy to recognise: in nature, the existent organisms either adapt or are regularly replaced by new types. Because of changes in the genetic material (mutations) these have new characteristics and in the course of their individual lives they are tested for optimal or better adaptation to the environmental conditions. Immortality would disturb this system - it needs room for new and better life. This is the basic problem of evolution D Every organism has a life span which is highly characteristic. There are striking differences in life span between different species, but within one species the parameter is relatively constant. For example, the average duration of human life has hardly changed in thousands of years. Although more and more people attain an advanced age as a result of developments in medical care and better nutrition, the characteristic upper limit for most remains 80 years. A further argument


against the simple wear and tear theory is the observation that the time within which organisms age lies between a few days (even a few hours for unicellular organisms) and several thousand years, as with mammoth trees. E If a lifespan is a genetically determined biological characteristic, it is logically necessary to propose the existence of an internal clock, which in some way measures and controls the aging process and which finally determines death as the last step in a fixed programme. Like the fife span, the metabolic rate has for different organisms a fixed mathematical relationship to the body mass. In comparison to the life span this relationship is ‘inverted’: the larger the organism the lower its metabolic rate. Again this relationship is valid not only for birds, but also, similarly on average within the systematic unit, for all other organisms (plants, animals, unicellular organisms). F Animals which behave ‘frugally’ with energy become particularly old for example, crocodiles and tortoises. Parrots and birds of prey are often held chained up. Thus they are not able to ‘experience life’ and so they attain a high life span in captivity. Animals which save energy by hibernation or lethargy (e.g. bats or hedgehogs) live much longer than those which are always active, The metabolic rate of mice can be reduced by a very low consumption of food (hunger diet) They then may live twice as long as their well fed comrades. Women become distinctly (about 10 per cent) older than men. If you examine the metabolic rates of the two sexes you establish that the higher male metabolic rate roughly accounts for the lower male life span. That means that they live life ‘energetically’ - more intensively, but not for as long. G It follows from the above that sparing use of energy reserves should tend to extend life. Extreme high performance sports may lead to optimal cardiovascular performance, but they quite certainly do not prolong life. Relaxation lowers metabolic rate, as does adequate sleep and in general an equable and balanced personality. Each of us can develop his or her own ‘energy saving programme’ with a little self observation, critical self-control and, above all, logical consistency. Experience will show that to live in this way not only increases the life span but is also very healthy. This final aspect should not be forgotten. The Reading Passage has seven paragraphs, A-G, For question 1-6, choose the correct heading for paragraphs A-G from the list of headings below. Write the correct number, i-x, in the corresponding numbered boxes. LIST OF HEADINGS


i The biological clock ii Why dying is beneficial iii The ageing process of men and women iv Prolonging your life v Limitations of life span vi Modes of development of different species vii A stable life span despite improvements viii Energy consumption ix Fundamental differences in ageing of objects and organisms x Repair of genetic material Example answer: Paragraph A: v Your answers 1. Paragraph B ….............

2. Paragraph C ….............

3. Paragraph D ….............

4. Paragraph E ….............

5. Paragraph F ….............

6. Paragraph G ….............

Questions 7-10, complete the notes below Choose NO MORE THAN TWO WORDS from the passage for each answer. Write your answers in boxes 7-10 • Objects age in accordance with principles of(7) __________ and of (8) _________ • Through mutations, organisms can (9) __________ better to the environment • (10) __________ would pose a serious problem for the theory of evolution Your answers 7.

8.

9.

10.

D. SECTION IV: WRITING (50 points) Part 1. Finish each of the following sentences in such a way that it has the same meaning as the printed above. (10 points) 1. He is very good at cooking spaghetti.


He is a dab ………………………………………………………………………… 2. I’ll have to wait before I know whether he’ll keep his promise or not. It remains …………………………………………………………………………. 3. I expected the book to be far better because it had been written by such a good novelist. The book fell ....................................................................................................... 4. We only came to this restaurant because you insisted that we did so. It was at ………………………………………………………………………… 5. Arguing with her won’t get you anywhere. It won’t do …………………………………………………………………….. Part 2: Rewrite the following sentences without changing their meaning, using the words given. These words must not be changed in any way. (10 points) 1.Sally became known throughout the country as a result of her popular TV series. HOUSEHOLD Sally became ………………………………………………..of her popular TV series. 2. I treated him in the same way as he had treated me. PAID I……………………………………………………………………………… 3. Jenny isn’t a bad accountant, but I don’t think it is a very suitable occupation for her. CUT I don’t think Jenny ………………………………………………………….. 4. You can't just suddenly decide to go on a safari. You need to plan things very carefully.SPUR → Going on safari isn't a decision you can make ...........................................moment. You need to plan things very carefully. 5. If they ever discover your role in the incident, you will go to prison. LIGHT → If your role in the incident ……………………….., you'll go to prison. Part 3.Write about the following topic. You should write at least 200 words.( 30 points) School children are becoming far too dependent on computers. This is having an alarming effect on reading and writing skills. Teachers need to avoid using computers in the classroom at all costs andgobacktoteachingbasicstudyskills.Do you agree or disagree?


HỘI CÁC TRƯỜNG THPT CHUYÊN VÙNG DUYÊN HẢI VÀ ĐỒNG BẰNG BẮC

KÌ THI HỌC SINH GIỎI NĂM HỌC 2018-2019

BỘ

MÔN THI: TIẾNG ANH LỚP 10 Ngàythi: ……. (Hướngdẫnchấmgồm 02 trang)

TRƯỜNG THPT CHUYÊN THÁI NGUYÊN HƯỚNG DẪN CHẤM SECTION I: LISTENING (50 points)

Part 1 (10 points) 2. 19 1. Hardie

3. GT8 2LC

4. hairdresser 5. dentist/dentist’s

Part 2 (10 points) 1. C 2. A

3.A

4.B

5. B

Part 3 (10 points) 1. T 2. F

3. T

4. T

5. F

Part 4: ( 20 points)

1 Holland 2 violin

6(roller) hockey

3 (large) mirror

4 Winter Pleasures

5 direction

7 championships 8 design/performance 9 eight 10 lighter/safer (in either order) million/8.000,000 (in either people order)

SECTION TWO: LEXICO-GRAMMAR (50 points) Part 1. 1.D

2.A

3.B

4.D

5.A

6.B

7.A

8.B

9.C

10.D

11.B

12.D

13.A

14.B

15.B


16.D

17.B

18,B

19.D

20.C

Part 2. 1. Line 2: spiral (demand) spiraling 2. Line 3: over (the globe) across 3. Line 4: their (wake) its 4. Line 7: That (was once) What 5. Line 8: into (the limits) beyond 6. Line 9: (more) especially specifically 7. Line 10: have (convinced) are 8. Line 17: (to solve) solving 9. Line 20: dead (perils) deadly 10. Line 20: bided (their time) biding Part 3. 1. up

2. into

3. up to

4. down

5. ahead

6. up

7. between

8. to

9. in

10. off

Part 4: 1. invariably

2. alternatively

3. descriptive

4. absence

5. commentary/ commentaries

6. opinionated

7. impartiality

8. enthusiasm

9.unpredictable

10.organizational

SECTION III: READING (50 points) Part 1. Answers: 1. C

2. A

3. D

4. D

5. C

6. B

7. B

8. A

9.A

10.D

Part 2: 1. set 6. including

2. as 7. signs

3. low 8. experienced

4. key 9. which

5. provide 10. on


Part 3. 1. B

2. C

3. A

4. D

5. D

6. B

7. C

8. A

9.C

10.B

Part 4. Read the following passage then do the tasks that follow. (15 points – 1.5 points/ correct answer) 1. ix 2. ii

7. physical chemistry 8. thermodynamics

3. vii 4. i 5. viii 6. iv

9. adapt 10. immortality (7 and 8 can be in either order)

1. xii

6.

2. x

reflex 7. use

3. iv

4. ii

5. viii

8. the disabled

9. factors

10.challenge/ challenging

mechanisms/ reflexes SECTION IV: WRITING (50 points) Part 1. 1. He is a dab hand at cooking.

2. It remains to be seen whether he’ll keep his promise or not. 3. The book fell short of my expectations even though it had been written by such a good novelist. 4. It was at your insistence that we came to this restaurant. 5. It won’t do you any good to argue with her. Part 2. 1.Sallybecame a household name as a result of her popular TV series. 2. I paid him back in his own coin. 3. I don’t think Jenny is cut out to be an accountant/ is cut out for being an accountant.


4. Going on safari isn't a decision you can make on the spur of the moment. You need to plan things very carefully. 5. If your role in the incident ever comes to light, you'll go to prison. Part 3: Paragraph writing (30 points) Mộtsốtiêuchuẩnchobàiviếtcầnlưu ý: 1. Vềhìnhthứcvànội dung: Bàiviếtdướidạngmộtđoạnvăn, phảicóđủ 3 phần, cụthể: • Mởbài: giớithiệuvấnđề ( the writer (dis)agrees with the statement that teachers need to avoid using computers in the classroom at all costs and go back to teaching basic study skills) • Thânbài: Nêuđượccácluậnđiểmchính (about 2 – 3 supporting ideas), mỗiluậnđiểmchínhcầncócácluậnđiểmphụđểgiảithíchrõvà/hoặcnêuvídụ minh họathíchhợp. • Kếtbài: Khẳngđịnhlạinội dung chínhvừatrìnhbàytrongthânbài. 2. Vềvănphong: • Ngônngữphảichuẩnmực, chínhxác, khôngsửdụngthànhngữ (idiom), khôngsửdụngdạngviếttắt (short form)… • Khôngđượcphépnhắcđếnbấtcứthông tin cánhângìtrongbàiviết (họtên, địachỉ, họtênbốmẹ, nghềnghiệp…) • Phảisửdụngcácdấuhiệuchuyển ý, chuyểnđoạnthíchhợp (transition signals) • Bàiviếtkhôngquángắn/ quádài: 200 -250 từ .

Tapescript PART 1: Interviewer: Excuse me. Luisa: Yes? Interviewer: I wonder if you could spare a few minutes to do a survey on transport. It won’t take long. Luisa: No, that’s fine. Interviewer: Lovely. The survey is on behalf of our local council. They’d like to know about what transport you use and any suggestions for improvement. Can I start by asking you how you travelled to town today? Luisa: Sure. I came on the bus. Interviewer: Great. Now can I get a few details about yourself? Luisa: OK Interviewer: What’s your name? Luisa: It’s Luisa. Interviewer: Yes


Luisa: Interviewer: Luisa: Interviewer: Luisa: Interviewer: Luisa: Interviewer: Luisa: Interviewer: Luisa: Interviewer:

Hardie. Can you spell that, please? H-A-R-D-I-E Great. Thanks. And can I have your address? It’s 19 White Stone Road. Oh, right. I know that area. It’s Bradfield, isn’t it? That’s right. Is the postcode GT7? It’s actually G-T-8, 2-L-C. Great. And could I ask what your job is? Are you a student? I’ve actually just finished my training. I’m a hairdresser. Oh, right. And one more question in this section. What is the reason for you coming into town today? Luisa: Actually, it’s not for shopping today, which would be my normal reason, but to see the dentist. Interviewer: Right. Thanks.

PART 2:


PART 3: We've also got a great entertainment programme lined up for you, starting tonight with our welcorning party. We have a top band lined up for your entertainment, but I'm not allowed to say who they are. All I can say is that I am sure you will not be disappointed. So come along to Blackmoor Hall at nine o’clock this evening to get your university experience off to a flying start! Just one point — I'm afraid this event is limited to freshers only. Because of space restrictions you can’t bring a friend tonight. Sorry about that!


There’s more fun and games On Monday in the Cotswold Theatre here on campus. We have booked two of the cleverest comedians in the country, Paul Frasier and Jenny Brown, for a three-hour show. Paul has assured us that he and Jenny have packed the show with new material and as they always get rave reviews for their shows, I think we can look forward to an evening of great entertainment. That' in the Cotswold Theatre on Monday evening at 7:30. Moving along a bit, on Thursday there is an important date for your diaries. This is the official freshers' opening ceremony, when the Dean welcomes you to Upton University. So remember, Thursday the front 2.30 to 3.30 in Blackmoor Hall. You certainly should go to this one and by the way, light refreshments will be available. At the end of the week, on Saturday, you have the chance to dress up in your smartest evening wear for the official freshers' ball. Actually, although it' called a ball, it is quite a relaxed affair we are more than happy if you turn up wearing jeans and a T-shirt, the important thing is to relax and enjoy yourselves. Time and place are the same as for this evening party — Blackmoor Hall from nine in the evening to three o’clock in the morning Right, I think I've covered the most important and exciting events we have lined up for you, but there will be plenty of other things going on throughout the Week. So remember to check the notice board in the entrance lobby regularly. Enjoy the rest of the day% and I look forward to meeting as many of you as possible this evening at the welcoming party. Part 4. (FCE) In today's programme, I'm going to be talking about roller skating: how the sport started and how it has developed over the years. So who was the first person to come up with the idea of attaching wheels to the feet in order to get about more quickly and easily? Well, roller skates are not a new invention. In fact, roller skating developed out of the much older activity of ice-skating, which has existed in Scandinavia and other northern countries for centuries. The actual inventor of the first roller skates is not known, but it's generally thought that they originated in Holland in the early 1700s. Roller skates first arrived in Britain in 1760 when the Belgian clockmaker John Merlin wore some to a formal ball in London. Merlin was known as something of a mad inventor, but he surprised everybody at the ball when he whizzed past them on wheels, playing the violin at the same time. Unfortunately, Merlin did not manage to persuade people that roller skating was a good idea. His skates had no brakes and he ended up crashing into a large mirror. Merlin was quite seriously injured in the accident and, as a result, roller skating did not immediately become popular in Britain.


In Germany, however, roller skates made a better impression. They were used in a ballet with the name Winter Pleasures, which included a scene where the dancers skated on ice. Because they couldn't produce the ice on stage, the organisers decided to use roller skates instead. After this, the sport gradually became more popular, but it was only thanks to technical advances that it became safer. In 1863, an American named James Plimpton solved the problem of controlling direction when skating by fitting them with rubber springs. His design is widely regarded as the origin of the modern roller skate, although rubber toe brakes, another important safety feature didn't come in until the 1870s. The late nineteenth century saw the beginnings of events such as speed contests, artistic displays and roller dancing as well as the first team sport on roller skates, roller hockey. During the first decades of the twentieth century, hundreds of indoor and outdoor roller skating rinks opened, especially in the USA, and the sport became really established as a popular pastime. The first roller skating championships were held in Detroit in 1937. The real development of the modern roller skate only began in the second half of the twentieth century. From the 1950s onwards, the use of plastics led to improvements in the design and performance of roller skates, and roller disco movies of the 1970s and 1980s increased the popularity of the sport, with roller discos opening in many parts of the world. Meanwhile, the stage musical Starlight Express, which features roller skating, ran for seventeen years and was seen by eight million people. The sport of roller skating has also been gaining a more serious following, especially in southern Europe and South America. The biggest modern change to roller skates came in 1983 with the introduction of in-line skates, also known as rollerblades. Then during the 1990s, new materials, brakes and boot fastenings all combined to make skates both lighter and safer than they had ever been in the past. So why is roller skating so popular? I went to talk to some fans at a rink in Huddersfield ...

Ngườisoạnđề: NguyễnAnhTuấn Mobile phone: 0912165314


Trường THPT Chuyên Vĩnh Phúc Đề đề xuất KỲ THI HỌC SINH GIỎI CÁC TRƯỜNG THPT CHUYÊN KHU VỰC DUYÊN HẢI VÀ ĐỒNG BẰNG BẮC BỘ LẦN THỨ XII, NĂM 2019 ĐỀ THI MÔN: TIẾNG ANH LỚP 10 Thời gian làm bài: 180 phút SECTION I: LISTENING (50/200) Part 1: Write NO MORE THAN TWO WORDS for each answer. (10 points) PRIME RECRUITMENT Employee record Example

Answer

Surname:

Riley

Email: Nationality:

(1)…….@worldnet.com (2)…….

Reference (professional): Name: John Keen Job: manager of (3)…….. Reference ( personal):

Name: Elleen Dorsini Job: (4)…..

Special qualifications:

current (5)…… certificate Certificate of competence in sailing.

Part 2: Choose the correct letter A,B,C.( 10 points) 1. What is Chloe concerned about?


A. her knowledge of maths B. her ability to write essays C. her lack of business experience 2. Which of the following does Ivan feel he has improved? A. his computer skills B. his presentation skills C. his time management 3. What does Chloe especially like about the course? A. She won’t have to do a final examination B. She can spend time working in a business C. She can study a foreign language 4. Ivan is pleased that the university is going to have A. more lecture rooms B. a larger library C. more courses 5. What does Ivan advise Chloe to do? A. contact his tutor B. read about some other university C. visit the university Part 3: Listen to part of a radio programme about a psychological condition known as prosopagnosia. Decide whether the statements are true (T) or false (F). (10 points) 1. The speaker compares face-blindness to the inability to hear.


2. Scientists do not understand how normal people remember faces. 3. The face-blind subjects could not distinguish between the faces or the objects. 4. Some people with this condition are so severely affected that they cannot recognise members of their own family. 5. It could help scientists to understand human evolution if they knew more about face-blindness.

Part 4: Write NO MORE THAN THREE WORDS.(20 points) Volunteer at the Children’s University The children’s University was started by a (1)…….. five years ago. The focus of this year’s Children’s University was the topic of (2)…….. . Laura’s partner was Mark, who works as a (3)……. when he is not volunteering. Laura’s group of volunteers gave some workshops about how (4)…… is made. Laura says the children had a booklet called a (5)…… which was stamped to show their progress. Laura and the children went to the graduation ceremony in the (6)……. hall of the local University. Some children received a (7)…….. for attending a lot of workshops. Laura said the scheme allowed her to develop skills such as (8)…….. Laura will most probably become a( 9)…….. in the future. Laura says she can give people in her audience something called an (10)…. for volunteers.

SECTION II: LEXICO – GRAMMAR (50 points) Part 1. Choose the best answer A, B, C or D to complete each of the following sentences.(20 points)


1. There has been a lot of ____ surrounding the government’s proposed scheme. A. controversy

B. consent

C. conformity

D. consequence

2. Our town has a real problem with youth crime, ____ do many other British towns. A. so

B. nor

C. as

D. like

3. Warning: anyone caught stealing from these premises will be _____. A. advocated

B. undermined

C. prosecuted

D. enforced

4. The local authorities need to _____ down on illegal parking, in my opinion. A. hit

B. force

C. move

D. crack

5. If the service isn’t up to standard, I think you have _____ right to complain. A. all

B. each

C. much

D. every

6. Jim’s tough character and certainly won’t let anyone push him ______. A. up

B. off

C. around

D. through

7. The ______ I don’t understand is why Emily lets her boyfriend get away with it. A. reason

B. object

C. item

D. thing

8. My uncle pulled a few _____ and got me a job in the company where he works. A. ropes

B. strings

C. threads

D. chords

9. Although she would have preferred to carry on working, my mum _____ her career in order to have children. A. devoted

B. repealed

C. sacrificed

D. abolished

10. I find the offer quite ____, but I think I’d rather study at Oxford. A. tempting

B. desirous

C. inclined

D. envious

11. I don’t normally like noisy clubs, but I had a sudden _____ to see what the Blue Parrot was like. A. force

B. motive

C. pressure

D. impulse

12. Jerry loves snowboarding so much that it’s almost like a drug ________.


A. passion

B. obsession

C. addition

D. requirement

13. I don’t want to do the course in applied statistics, but it’s ______. A. compulsory

B. inevitable

C. bound

D. indecisive

14. Don’t worry about me – I’m quite ______ to sit here and wait for you to come back. A. ecstatic

B. delighted

C. joyful

D. content

15. When I was pregnant, I often got a sudden _____ for tinned sardines. A. preference

B. craving

C. envy

D. greed

16. Thank you for thinking of us, but I’m afraid we’re going to have to _____ your kind invitation. A. decline

B. deny

C. condemn

D. reject

17. I’ve never seen anyone so _____ to their job as Philip is. A. eager

B. keen

C. dedicated

D. interested

18. Why do you have such a _____ with model railways? A. desire

B. fascination

C. love

D. preference

19. I wish you would stop wasting so much time on computer games and do something a little more _____. A. welcome

B. enviable

C. feasible

D. worthwhile

20. Olivia has always ______ to return to the country she was born in. A. favoured

B. yearned

C. urged

D. inclined

Part 2: The passage below contains 10 mistakes. Find out and correct them.(10 points) Even before the turn of the century, movies began to develop in two major directions: the realistic and the formalistic. Realism and formalism are merely general, rather than absolute, terms. When using to suggest a tendency toward either polarity, such labels can be helpful, but at the end they are still just labels. Few films are exclusive formalist in style, and fewer yet are completely realist.


There is also an important difference between realism and reality, although this distinct is often forgotten. Realism is a particular style, where physical reality is the source of all the raw materials of film, both realistic and formalistic. Virtually all movie directors go to the photographable world for their subject matter, but what they do with this material - what they shape and manipulate it - determines their stylistic emphasis. Generally speaking, realistic films attempt to reproduce the surface of concrete reality with a minimum of distortion. In photographing objects and events, the filmmaker tries to suggest the copiousness of life himself. Both realist and formalist film directors must select (and hence emphasize) certain details from the chaotic sprawl of reality. But the element of selectivity in realistic films is less obvious. Realists, in short, try to preserve the illusion that their film world is unmanipulated, an objective mirror of the actual world. Formalists, on the other hand, make no such pretense. They deliberately stylize and distort their crude materials so that only the very naive should mistake a manipulated image of an object or event to the real thing. Part 3. Fill in each blank with a suitable preposition. Write your answers in the corresponding numbered boxes. (10 points) 1. Her husband was courteous ….her 2. You should comply....the school rules 3. Be calm and don’t make a fuss…..such trifles. 4. John is very good…..his hands 5. She knows how to get…….her father to buy her a new car. 6. the police are looking …the past record of the suspect 7. When she hear the news of the crash she broke …….and cried 8. She always sides……..her son against her husband. 9. in the warm sun, the grass dries…….quickly 10.You shouldn’t look down on the people who aren’t as well……as you are. Part 4. Supply the correct form of the words in bracket. Write your answers in the corresponding numbered boxes. (10 points) In the late 30s, (1. ANALYSE) knew that the centre of the historic town of Coventry in the West Midlands needed to be redeveloped. Plans had to be (2 . SHELF) when the Second World War started in 1939. However, the architect’s opportunity (3. MATERIAL) when the city center was practically destroyed during


the war. Many buildings were (4. REPAIR) damaged and demolition work began. Aiming to create a much more (5. SPACE) area for (6. RESIDE) to work and shop in, town planners came up with a radical idea. They would make the city more (7. INHABIT) by pedestrianising the center, preventing cars entering. There were (8. OBJECT) from local shopkeepers, who thought that it would have an impact on trade, but the planners went ahead. What was once a (9. DENSE) populated area became a pleasant, attractive place to visit. It was a real (10. ARCHITECT) achievement, one that many British towns have emulated since. III: READING (50 points) Part 1: Read the text below and decide which answer best fits each space. Write your answers in the corresponding numbered boxes. (10 points) The Alexandra Palace in north London was built with private funds as a “People’s Palace”. Serviced by its own station, it was opened in 1873 and was extremely well (1)_______ until, two weeks after its opening, it burnt down. It was replaced by a slightly larger building which opened in 1875 and featured, (2)________other things, a splendid organ an Great Hall, which was the size of a football pitch. Despite the extraordinarily wide range of events (3)_______ there – from dog shows to great concerts and banquets, from elephant displays to bicycle matches – it always operated at a loss and by 1877 much of the park around it had been sold to speculative builders, leaving only about half of the original land. In 1900, a committee was appointed, whose principal duty was to run the palace and park “for the free use of the people forever.” There were, however, (4) ________ to charge for entry so that the substantial costs could be (5) _______ . The Palace continued, with (6) ________ degrees of success, as an entertainment centre. In the 1930s, it was probably most (7) ________ for being the home of the world’s first high definition television broadcasts. In 1980 the building was once more devastated by fire and (8) _______ to a ruin. It was then decided to (9) _________ it and to create a major exhibition centre with community (10)_______, such as a restaurant and a health club. 1. A. inhabited

B. attended

C. crowded

D. visited


2. A. among

B. between

C. from

D. around

3. A. performed

B. set

C. staged

D. laid

4. A. powers

B. terms

C. allowances

D. authorities

5. A. fulfilled

B. covered

C. matched

D. made

6. A. unsteady

B. varying

C. altering

D. unsettled

7. A. distinct

B. marked

C. considerable

D. notable

8. A. turned

B. converted

C. reduced

D. wrecked

9. A. recover

B. revise

C. restore

D. reform

10. A. facilities

B. conveniences

C. supplies

D. appliances

Part 2: Fill in each blank space with an appropriate word. Write your answers in the corresponding numbered boxes. (15 points) Where do you start if you want to write a successful story? Clearly, what you need first of (1) is an idea which you can develop into a strong plot. But (2) do ideas like this come from? The (3) is “anywhere and everywhere”. They may come from something that has (4) to you or to (5) else, from a newspaper, an interesting picture, or even a song. It is a good idea to keep a notebook nearby (6) that you can write down the details or any odd incidents (7) catch your imagination. Make a note of ideas (8) titles too, and any special phrases or descriptions that you think of. A small tape recorder can (9) useful for this purpose. Some writers even keep (10) by their bed in case they wake up with the “idea of the century”. Part 3: Read the passage and choose the best answer for each of the following questions. Write your answers in the corresponding numbered boxes. (10 points) Have you ever thought about the names of the months? Why are “January” and “February” not called “Primo” or “Secondo”? Is it because the original names were created in ancient times? Or is it because the originators preferred odd words? Take February, for example. Say it aloud a few minutes and you start to wonder. Most people don’t know who developed these names. However, a little research reveals that the names of the months came mostly from a combination of


the names of Roman gods and goddesses, important festivals, and the original numbers of the months. Julius Caesar and Pope Gregory XIII change the calendar to make it more exact. Caesar developed a new calendar of 364 and a quarter days, the time it takes the earth to orbit the sun from one spring season to the next. The Pope’s astronomers refined the calendar regarding leap years; they determined that there should be no leap year in years ending in 00- unless they were divisible by 400; the years 1700, 1800, 1900 and 2100 would not be considered leap years, while the years 1600 and 2000 would be. This new Gregorian calendar was so accurate that today, scientists need only add leap seconds every few years to the clock in order to keep the calendar matching the Earth’s cycles. 1. What is the topic of the passage? A. how the modern calendar was named and developed B. how the months were named C. how the leap year system was developed D. how accurate the modern day is calendar 2. It can be inferred from paragraph 1 that the author think the names of the months are __________. A. odd B. difficult to pronounce C. inappropriate D. none of the answers 3. The word “they” in paragraph 3 refers to ______________. A. calendars B. days C. astronomers D. years 4. The word “accurate” in paragraph 3 is closest in meaning to ____________. A. interesting B. informative C. correct D. simple 5. Which of the following will be a leap year? A. 2300 B. 2400 C. 2200 D. 2500 6. Which of the following is true of the Gregorian calendar? A. It needs major improvements. B. It was so well designed, it needs little adjusting today. C. It copied the Roman calendar’s formula of leap years. D. none of the answers 7. Why is Caesar important in calendar making? A. He changed the length of the year . B. He extended summer. C. He has a month named for him. D. He altered the number of days in the year. 8. In what order is the information in the passage presented? A. Caesar’s calendar, the Gregorian calendar, the modern calendar B. Roman Gods, important festivals, original numbers of months


C. names of months, Caesar’s calendar, the Gregorian calendar D. none of the answers 9. The word “refined” in paragraph 3 is closest in meaning to ____________. A. studied B. invented C. observed D. improved 10. Why is the number of 364 and a quarter important? A. It is the length of time from the beginning of spring to the end of winter. B. It is the length of a planetary year. C. It is the most accurate number for calendars. D. It was a number randomly chosen by Caesar for his calendar. Part 4: Read the text below and do the following tasks.(15 points) Gifted children and learning A. Internationally, ‘giftedness’ is most frequently determined by a score on a general intelligence test, known as an IQ test, which is above a chosen cut-off point, usually at around the top 2-5%. Children’s educational environment contributes to the IQ score and the way intelligence is used. For example, a very close positive relationship was found when children’s IQ scores were compared with their home educational provision (Freeman, 2010). The higher the children’s IQ scores, especially over IQ 130, the better the quality of their educational backup, measured in terms of reported verbal interactions with parents, number of books and activities in their home etc. Because IQ tests are decidedly influenced by what the child has learned, they are to some extent measures of current achievement based on age-norms; that is, how well the children have learned to manipulate their knowledge and know-how within the terms of the tests. The vocabulary aspect, for example, is dependent on having heard those words. But IQ tests can neither identify the processes of learning and thinking nor predict creativity. B. Excellence does not emerge without appropriate help. To reach an exceptionally high standard in any area very able children need the means to learn, which includes material to work with and focused challenging tuitionand the encouragement to follow their dream. There appears to be a qualitative difference in the way the intellectually highly able think, compared with more average- ability or older pupils, for whom external regulation by the teacher often compensates for lack of external regulation. To be at their most effective in their self-regulation, all children can be helped to identify their own ways of


learning _ metacognition _ which will include strategies of planning, monitoring, evaluation, and choice of what to learn. Emotional awareness is also part of metacognition, so children should be helped to be aware of their feelings around the area to be learned, feelings of curiosity or confidence, for example. C. High achievers have been found to use self-regulatory learning strategies more often and more effectively than lower achievers, and are better able to transfer these strategies to deal with unfamiliar tasks. This happens to such a high degree in some children that they appear to be demonstrating talent in particular areas. Overviewing research on the thinking process of highly able children, (Shore and Kanevsky, 1993) put the instructor’s problem succinctly: ‘If they (the gifted) merely think more quickly, then we need only teach more quickly. If they merely make fewer errors, then they can shorten the practice’. But of course, this is not entirely the case; adjustments have to be made in methods of learning and teaching, to take account of the many ways individuals think. D. Yet in order to learn by themselves, the gifted do need some support from their teachers. Conversely, teachers who have a tendency to ‘overdirect’ can diminish their gifted pupils’ learning autonomy. Although ‘spoon-feeding’ can produce extremely high examination results, these are not always followed by equally impressive life successes. Too much dependence on the teacher risks loss of autonomy and motivation to discover. However, when teachers help pupils to reflect on their own learning and thinking activities, they increase their pupils’ self-regulation. For a young child, it may be just a simple question ‘What have you learn today?’ which helps them to recognize what they are doing. Given that a fundamental goal of education is to transfer the control of learning from teachers to pupils, improving pupils’ learning to learn techniques should be a major outcome of the school experience, especially for the highly competent. There are quite a number of new methods which can help, such as child-initiated learning, ability-peer tutoring, etc. Such practices have been found to be particularly useful for bright children from deprived areas. E. But scientific progress is not all theoretical, knowledge is also vital to outstanding performance: individuals who know a great deal about a specific domain will achieve at a higher level than those who do not (Elshout, 1995). Research with creative scientists by Simonton (1988) brought him to the conclusion that above a certain high level, characteristics such as independence seemed to contribute more to reaching the highest levels of expertise than


intellectual skills, due to the great demands of effort and time needed for learning and practice. Creativity in all forms can be seen as expertise mixed with a high level of motivation (Weisberg, 1993). F. To sum up, learning is affected by emotions of both the individual and significant others. Positive emotions facilitate the creative aspects of learning and negative emotions inhibit it. Fear, for example, can limit the development of curiosity, which is a strong force in scientific advance, because it motivates problem-solving behavior. In Boekaerts’ (1991) review of emotion in the learning of very high IQ and highly achieving children, she found emotional forces in harness. They were not only curious, but often had a strong desire to control their environment, improve their learning efficiency, and increase their own learning resources. Question 1-5 Look at the following statements (Questions 1-5) and the list of people bellow. Match each statement with the correct person or people, A-E. Write the correct letter A-E on your answer sheet. 1. Less time can be spent on exercises with gifted pupils who produce accurate work. 2. Self-reliance is a valuable tool that helps gifted students reach their goals. 3. Gifted children know how to channel their feelings to assist their learning. 4. The very gifted child benefits from appropriate support from close relatives. 5. Really successful students have learnt a considerable amount about their subject. List of people A. Freeman B. Shore and Kanevsky C. Elshout D. Simonton E. Boekaerts Questions 6-9


Which paragraph contains the following information? Write the correct letter, A-F on your answer sheet. NB You may use any letter more than once. 6. a reference to the influence of the domestic background on the gifted child. 7. reference to what can be lost if learners are given too much guidance. 8. a reference to the damaging effects of anxiety. 9. examples of classroom techniques which favour socially-disadvantaged children Questions 10 Complete the sentence below. Choose no more than two words from the passage for the answer. - One study found a strong connection between children’s IQ and the availability of (10) and activities at home. V. WRITING (50 points) Part 1. Rewrite each of the following sentences in such a way that it has the same meaning as the printed above sentence. (10 points) 1. He never thought of telling her. It never_________________________________________________ 2. The students’ riotous behaviour should have been severely punished. The students deserved______________________________________ 3. He suddenly thought that he might have misunderstood her. It crossed_________________________________________________ 4. If we wait long enough, we’ll get what we want.


It’s just___________________________________________________ 5. I was not surprised to hear that Harry had failed his driving test. It came___________________________________________________ Part 2. Rewrite the following sentences with the given words in such a way that the second sentence has the same meaning as the first one. Do not change the form of the word in brackets. (10 points) 1.Having to get up so early is so irritating.

(NECK)

____________________________________________________ 2. It’s likely they were delayed in a traffic jam.

(UP)

__________________________________________________ 3. My dad is not happy with me at all because he heard me swearing. (BOOKS) ____________________________________________________ 4. We can’t possibly imagine how we are going to afford a new car. (REMOTEST) ____________________________________________________ 5. There don’t seem to be many talented athletes at the moment.

(PAUCITY)

____________________________________________________ Part 3: II. In about 200 words, write a paragraph about the changes that information technology bring to your study.(30 points) Người ra đáp án Dương Thị Bích Ngọc


SĐT: 0915094719 Trường THPT Chuyên Vĩnh Phúc Đáp án KỲ THI HỌC SINH GIỎI CÁC TRƯỜNG THPT CHUYÊN KHU VỰC DUYÊN HẢI VÀ ĐỒNG BẰNG BẮC BỘ LẦN THỨ XII, NĂM 2019 ĐỀ THI MÔN: TIẾNG ANH LỚP 10 SECTION I: LISTENING (50/200) Part 1: Write NO MORE THAN TWO WORDS for each answer. (10 points) 1. EDWINARI 2. Newzealander 3. play centre 4. a professor 5. first aid Part 2: Choose the correct letter A,B,C.( 10 points) 1. B 2. C 3. A 4. C 5. A Part 3: Listen to part of a radio programme about a psychological condition known as prosopagnosia. Decide whether the statements are true (T) or false (F). (10 points) 1. F


2. T 3. F 4. T 5. T Part 4: Write NO MORE THAN THREE WORDS.(20 points) 1. local charity 2. industry 3. lawyer 4. chocolate 5. passport 6. concert 7. gold medal each 8. solving problems 9. social worker 10. information pack SECTION II: LEXICO – GRAMMAR (50 points) Part 1. Choose the best answer A, B, C or D to complete each of the following sentences.(20 points) 1. A

3. C

5. D

7. D

9. C

11. D

13. A

15. B

17. C

19. D

2. C

4. D

6. C

8. B

10. A

12. C

14. D

16. A

18. B

20. B

Part 2: The passage below contains 10 mistakes. Find out and correct them.(10 points)


1. using -> used

6. what -> how

2. at -> in

7. himself -> itself

3. exclusive -> exclusively

8. crude -> raw

4. distinct -> distinction

9. should -> would

5. where -> whereas/while

10. to -> for

Part 3. Fill in each blank with a suitable preposition. Write your answers in the corresponding numbered boxes. (10 points) 1. to 6. into

2. with 7. down

3. over 8. with

4. with 9. off

5. round 10. off

Part 4. Supply the correct form of the words in bracket. Write your answers in the corresponding numbered boxes. (10 points) 1. analysts 2. shelved

3. materialized 4. irreparably

5. spacious 6. residents

7. 9. densely habitable/inhabitable 8. objections 10. architectural

III: READING (50 points) Part 1: Read the text below and decide which answer best fits each space. Write your answers in the corresponding numbered boxes. (10 points) 1. B

2. A

3, C

4, A

5. B

6. B

7. D

8. C

9. C

10. A

Part 2: Fill in each blank space with an appropriate word. Write your answers in the corresponding numbered boxes. (15 points)


1. all 6. so

2. where

3. answer/reply 7. which/that 8. for/ about

4. 5. happened/occurred someone/somebody 9. be/prove 10. one/theirs

Part 3: Read the passage and choose the best answer for each of the following questions. Write your answers in the corresponding numbered boxes. (10 points) 1. A

2. A

3. C

4. C

5. B

6. B

7. A

8. C

9. D

10. B

Part 4: Read the text below and do the following tasks.(15 points) 1. B 2. D 3. E

4. A 5. C 6. A

7. D 8. F 9. D

10. books

V. WRITING (50 points) Part 1: Question 1-5 (10 points – 2 points/correct answer) 1. It never occured to him to tell her. 2. The students deserved severe punishment for their riotous behaviour. 3. It crossed his mind that he might have misunderstood her. 4. It’s just a question/matter of waiting long enough to get what we want. 5. It came as no surprise (to me) that Harry had failed his driving test. Part 2: Question 1-5 (10 points – 2 points/correct answer) 1. Having to get up early is a pain in the neck. 2.It’s likely they were held up in a traffic jam./ They might have been held up in a traffic jam. 3. I’m not in my dad’s good books/ I’m in my dad’s bad books. 4.We haven’t/ don’t have the remotest idea how we are going to afford a new car.


5. We seem to be suffering from a paucity of talented athletes at the moment. Part 3: Paragraph writing (30 points) 1. Convincing ideas: 6 points 2. Good organization: 6 points 3. Logical cohesion: 6 points 4. Accurate grammar and spelling: 6 points 5. Wide range of vocabulary and clear expression: 6 points TRANSCRIPT Part 1:



Part 2



Part 3

Presenter: this week’s All in the Mind examines an unusual condition you may never have heard of

before prosopagnosia. Here’s Professor Alexander Scharma to explain. Professor: Hello. Well, let’s start with an image some of you may be familiar with a painting called The Son of Man, by the surrealist artist Rene Magritte. In the picture, an apple floats in front of a man’s face,

covering the features that would normally allow him to be recognized. The painting perfectly illustrates the concept of prosopagnosia, or face-blindness. To people with this condition, as soon as someone leaves their sight the memory of that person’s face is blank- or, at best, a set of jumbled features. Faceblindness is a little like tone-deafness: the tone can be heard, or the face seen, but distinguishing

between different tones or faces is nearly impossible. The effects of prosopagnosia can be so bad that people severely affected can recognize their own parents or children. If we understood how the normal brain recalls faces, we’d be well on the way to und understanding erstanding this strange disorder. It might also help us to understand human evolution, since the ability to recognize individuals. This ability helps to hold society together and has enabled human beings to develop a complex culture which is unique in the animal kingdom.


Part 4 I’m here today to talk about a wonderful project I’ve been involved in this summer called the Children’s University. It’s run every day for local children aged from seven to fourteen by a team of parents, teachers, students and local employers. It was originally set up by a local charity and has been running for the last five years. The aim of the Children’s University is to inspire children and foster a love of learning. It sounds rather grand I know, but it was amazing to watch the children working so enthusiastically on all the various projects. This year the topic and overall theme was industry, building on previous years’ themes such as culture and science. Each volunteer on the scheme was partnered with another with a different background. I was paired up with Mark, who was fantastic at motivating the children. He was a natural teacher and the children really loved him. Not exactly what you’d expect from a lawyer! I’m a student at university doing biology so we couldn’t have been more different! The workshop material was written by the volunteers. I was in a group of six who taught a series of workshops on manufacturing in our city called Making Chocolate. The workshop included taking the children to visit the local factory and we even had a tasting session! Other groups did things like making their own furniture and baking bread. Each child had their own booklet which was stamped when they’d successfully finished a project. This was known by everybody as their ‘passport’. The children were very competitive and were always comparing to see who’d collected the most stamps. It really seemed to be much more effective in encouraging the children to do more workshops than last year when group leaders simply signed their notebook each time they did a workshop. At the end of the Children’s University we held a graduation ceremony in the city’s real university. This was held in the Concert Hall and was very exciting for the children. They all wore graduation caps and gowns. And afterwards, everyone went to the dining hall for the official photos and refreshments. It was just like a proper graduation ceremony and the children and their parents loved it. Every child that attended the Children’s University was awarded a certificate. In addition, the children who had attended the most workshops received a gold medal each. Some had been to as many as forty workshops over the summer! The main benefit for me of volunteering for the Children’s University was learning from the other volunteers because many of them were older than me and more experienced. For example, I had ample opportunity to practise problem-solving and became much better at it. It’s a useful skill that I can use in the workplace when I graduate. Another benefit is that it gave me the opportunity to work with children. I’d been undecided about whether to become a teacher or a social worker. While I loved helping the children learn, the experience has confirmed that what I most want to do is help children and their families in our community who are having difficulties. I think that having worked on the Children’s University should definitely help me after I graduate and want to start my career because I can show that I’ve got experience of working closely with children. I’m definitely going to do it again next summer. If you think this kind of work is for you, and would like to register for next summer, come and ask me for an information pack after this talk. When you’re sure it’s what you want to do, you can download an application form from the Children’s University website.

Người ra đáp án


Dương Thị Bích Ngọc SĐT: 0915094719


Issuu converts static files into: digital portfolios, online yearbooks, online catalogs, digital photo albums and more. Sign up and create your flipbook.